Williams Obstetrics 23rd Edition Study Guide

Share Embed Donate


Short Description

Williams Obstetrics Study Guide...

Description

Williams OBSTETRICS

STUDY GUIDE HOFFMAN • HORSAGER • ROBERTS ROGERS • SANTIAGO-MUNOZ • WORLEY

CONTENTS xi

Preface

1. Obstetrics in P e rs p e c tiv e .....................................2

2. M aternal A n a t o m y ..................................................8

, 4. Fetal G rowth and D e v e lo p m e n t.....................21

3. Im plantation , Embryogenesis, and Placental D e v e lo p m e n t........................... : ........ 14

5. M aternal P h y s io lo g y ...........................................27 & pa r , u rjt io n ................................................... ........... 3 3

M

M

SECTION 3 ANTEPARTUM

7. Preconceptional Counseling............................ 40

12.

Genetics.................................................................... 6 6

. Prenatal C a r e .......................... .............................45

13.

Prenatal Diagnosis and Fetal The rap y........... 72

8

9. A b o rtio n ..................................................................50 10. Ectopic P regnancy...............................................56 11. Gestational Trophoblastic D isease..................61

14. Teratology and M edications That Affect the F e tu s ......................................................77 15.

A n te p a rtu m A s s e s s m e n t................................... 82

16.

Fetal Im a g in g ..........................................................8 8

v iii

Contents

SECTION 4

I:

LABOR AND DELIVERY

17. N o rm al Labor and D e liv e r y ..................... .......96

23.

18. In trapa rtum A s s e s s m e n t..........................

....101

24.

19. Obstetrical A n e s th e s ia ............................... . .. .1 0 6

25.

20

. A b n orm al L a b o r...........................................

21

. Disorders of A m nionic Fluid Volum e ... . .. .1 1 7

22

. Labor In d u c tio n .............................................

H y s te re c to m y ....................................................... 137

....112

....122

132

26.

IL Cord, and M e m b r a n e s ......................................147

SECTION 5

S|9?

FETUS A N D NEWBORN 28. The N e w b o rn Infant

.1 5 6

29. Diseases and Injuries of the Fetus and N e w b o r n ....................................................... 161

PUERPERIUM 30. The P u e rp e riu m ....

,168

32. Contraception.

.178

31. Puerperal Infection.

.173

33. S te riliz a tio n ...

,184

, i,

jt*

SECTION 7

-y

-v

-

,

$ 6.

OBSTETRIC COMPLICATIONS 190

38. Fetal G row th D is o rd e rs ........................... .......216

35. Obstetrical Flemorrhage

196

39. M ultifetal G e s ta tio n ................... .............. ........ 2 2 1

36. Preterm B i r t h ....................

205

40. Reproductive Tract A b n o r m a lit ie s ....... .......226

210

Contents

41. General Considerations and M aternal E va luation.........................................................

234

42. Critical Care and T raum a.............................

239

43. O b e s ity ..............................................................

244

44. Cardiovascular D iseases.............................

249

45. Chronic H y p e rte n s io n ..................................

254

46. Pulm onary Disorders..........................: ........

259

47. T h ro m b o e m b o lic D iso rd e rs......................

265

48. Renal and Urinary Tract Disorders

270

51. Hem atological Disorders...............................

786

52. Diabetes.............................................................

791

53. Thyroid and Other Endocrine D isorders.. ..2 9 6 54. Connective-Tissue Disorders........................

301

55. Neurological and Psychiatric Disorders...

307

56. Dermatological Disorders.............................

31?

57. Neoplastic Diseases.......................................

317

58. Infectious Diseases.........................................

3??

59. Sexually Transmitted Diseases...................

328

49. Gastrointestinal D isorders.......................... ...2 7 6 50. Hepatic, Gallbladder, and Pancreatic D isorders.......................................................... ...2 8 1 Index

335

ix

SECTION 1

OVERVIEW

2

CHAPTER 1

Obstetrics in Perspective 1 -1 . Which o f the following is an example o f an indirect

1 -5 . What statistic is defined by the red line in this figure?

maternal death?

,0

a. Septic shock following abortion b. Aspiration following eclampsia seizure C. Hemorrhage following uterine atony

,+

d . Aortic rupture secondary to Marfan syndrome 1-2. The fertility rate is the number oflive births per 1,000 females aged?

a. 8-39

-

N e o n a ta l m o r ta lity

CD

Q.

0

05 oc

0 F e ta l m o r ta lity

b. 11-59 c. 15-44 d. 1 8 ^ 9 1-3. A death o fa newborn at 5 days o f life due to congen ital heart disease would be classified as which o f the following?

Year Reproduced, with permission, from Cunningham F G , Leveno KJ, Bloom SL, er al: W illiams Obstetrics, 23rd ed. New York, M cGraw-Hill, 2010.

a. Stillbirth rate

a. Fetal death

b. Infant death rate

b. Infant death

C.

C.

Late neonatal death

d. Early neonatal death 1-4. Which o f the following correctly identifies the trends

-++0

,44+

Perinatal mortality rate

d. None o f the above 1-6. Most infant deaths occur in which o f the following groups?

in fertility and live birth rates in the United States since

a. Low-birthweightinfants

2 002 ?

b. Infants o f diabetic mothers

a . There has been no change in either rate.

C.

b. Fertility and live birth rates have increased. C. Fertility and live birth rates have decreased.

d . Fertility rate has decreased but live birth rate is increased.

Infants with congenital infections

d. Infants with congenital anomalies 1-7. Approximately what percentage o f pregnant women experience severe maternal morbidity?

a. 0.1% b. 0.5% C.

2%

d. 5% 1-8. Which o f the following events has contributed to the rise in the U . S. cesarean delivery rate?

a. Increase in deliveries performed by midwives b. Decline in vaginal births following cesarean delivery C. Reduction in reimbursement by insurance companies for cesarean delivery

d. Improved outcomes in infants with congenital abnor malities delivered by cesarean delivery

Obstetrics in Perspective 1-9. O n the basis o f an American College o f Obstetricians and Gynecologists survey, which o f the following is the most common citation for obstetric malpractice claims?

1 -1 6 . Very low-birthweight newborns are defined as those whose birthweight lies below which o f the following thresholds?

3 . Shoulder dystocia

a. 5 0 0 g

b. Electronic fetal monitoring

b. l,0 0 0 g

C. Stillbirth or neonatal death

c. 1,500 g

d . Neurologically impaired neonate

d.

1 -1 0 . Adolescent pregnancy accounts for what percentage o f the total number o f unintended pregnancies in the United States?

2 ,0 0 0 g

1 -1 7 . Which o f the following is defined as the number of maternal deaths that result from the reproductive process per 1 0 0 , 0 0 0 live births?

a. 5%

a. Maternal mortality rate

b. 10!

b. Maternal mortality ratio

c. 25%

C. Direct maternal death rate

d. 33%

d. Pregnancy-related death rate

1 -1 1 . W hat percentage o f the gross domestic product currently goes toward health care? 3 . 6%

b. 16% C. 26%

d. 36%

1 -1 8 . Including spontaneous abortions, induced abortions, and live births, what was the approximate number o f pregnancies in 2006?

a. 4 million b.

5

million

C.

6

million

d. 7 million 1 -1 2 . The field o f obstetrics encompasses all E X C E P T which o f the following?

a. Prenatal care b. Management o f labor C. Infertility treatments

d. Immediate newborn care

1 -1 9 . Which racial/ethnic group has the highest fetal death rate in the United States?

a. White b. Black C. Asian

d. Hispanic 1 -1 3 . “Preterm” defines neonates born before what gestational age?

a. 34 completed weeks b. 36 completed weeks C. 37 completed weeks

d. 38 completed weeks

1 -2 0 . Fetal death in the United States is defined as which o f the following?

a . Death following conception b. Death after 20 weeks’ gestation C. Death with a birthweight greater than 500 g

d. All o f the above 1 -1 4 . W hat is the leading indication for hospitalization during pregnancy not related to delivery?

a. Influenza b. Diabetes mellitus C. Chronic hypertension

d. None o f the above

1 -2 1 . Registration o f live births is currently assigned to which national agency? a . Bureau o f the Census

b. National Institutes of Health C. Department o f Health and Human Services

d . National Center for Health Statistics 1 -1 5 . The current number o f abortions performed annually in the United States approximates which o f the following?

a. 100,000 b. 500,000 C. 1,000,000

d. 2 ,000,000

1 -2 2 . Which o f the following is N O T among the top three causes o f pregnancy-related death?

a. Embolism b. Hemorrhage C. Cardiomyopathy

d. Hypertensive disorders

4

Overview 1 -2 3 . What is the average number o f live births per woman in

1 -2 6 . Goals o f Healthy People 2010 and 2020 include all

the United States?

E X C E P T which o f the following?

a. l

a. Reduce maternal mortality rates

b. 2

b. Reduce nonmaternal death rates

c. 3

C. Reduce the rate o f preterm delivery

d. 4

d . Decrease the disparity o f outcomes between whites and ethnic/racial minorities

1 -2 4 . What was the cesarean delivery rate in the United States in 2006?

a. 21 ! b. 28% c. 31% d. 36% 1 -2 5 . Which o f the following contributed to the trend on this graph?

Year Reproduced, with permission, from Cunningham F G , Leveno K J, Bloom SL, et al: "Williams Obstetrics, 23rd ed. New York, McGraw-Hill, 2010

3. Contraception

b. Antibiotic treatment C. Availability o f blood products

d. All o f the above

Obstetrics in Perspective

CHAPTER 1 ANSWER KEY Question num ber

Question num ber

Letter answer

Page cited

Header cited

Definitions

1-14

d

p. 4

Pregnancy-Related Care

Definitions

1-15

c

p. 4

Table 1-1

Letter answ er

Page cited

Header cited

1-1

d

p. 3

1-2

c

p. 3

1-3

d

p. 3

Definitions

1 -16

c

p. 3

Definitions

1-4

c

p. 4

Pregnancy Rates

1 -1 7

b

p. 3

Definitions

1-5

c

p. 5

Figure 1-1

1-18

b

p. 4

Pregnancy Rates

1-6

a

p. 5

Infant Deaths

1-19

b

Figure 1-2

1-7

b

p. 6

Severe Maternal M orbidity

1-20

d

P -5 p. 3

1-8

b

p. 7

Rising Cesarean Delivery Rate

1-21

d

p. 2

Vital Statistics

1-9

d

p. 9

Table 1-5

1-22

c

p. 6

Table 1-2

1-1 0

c

p. 10

Figure 1-5

1-23

b

p. 4

Pregnancy Rates

1-11

b

p. 11

Healthcare Reform

1 -2 4

c

p. 7

Rising Cesarean Delivery Rate

1-12

c

1-25

d

p. 6

Figure 1-3

1-13

c

P- 2 p. 3

Introduction Definitions

1-26

b

p. 4

Healthy People 2010 and 2020

Definitions

Maternal Anatomy

+18/32 = 56%

2 - 1 . Which o f the following in general is true o f vertical abdominal incisions?

2 -3 . Which dermatome approximates the level o f the umbilicus?

a . Heal better than Pfannenstiel incisions

a. T 7-T 1 1

b. Follow vertically oriented dermal fibers (Langer lines)

b. T12

C. Yield inferior cosmetic results compared with trans verse incisions

d. LI

c. T10

d. Sustain decreased lateral tension 2 - 2 . The femoral artery gives rise to all EX C E PT which o f the following vessels?

2 -4 . Which vulvar structure is homologous to the male scrotum?

a. Labia minora b. Urethra C.

Vestibule

d. Labia majora 2 - 5 . With regard to the vestibule, which of the following statements is correct?

a. It is derived from embryonic urogenital membranes. b. The posterior portion between the fourchette and the vaginal opening is called the fossa annulares. C.

It is perforated by four openings,

d. The fossa navicularis is usually observed only in nulliparous women. 2 - 6 . The internal pudendal artery supplies which o f the following? 3. Proximal vagina

b. Posterior vaginal wall C.

Distal vaginal wall

d. Bladder trigone

-cec; with permission, from Cunningham F G , Leveno K J, Bloom SL , et al: Williams :cs. lo rd ed. New York, M cGraw-Hill, 2010.

a. superficial circumflex iliac artery b. External pudendal artery C. !r.:;r:or epigastric artery d. ' _rrrf_cial epigastric artery

M a te rn a l A n a to m y

2 -7 . The vagina and its investing musculature are supplied by all E X C E P T which of the following arteries?

2 -1 1 . Which o f the following is true regarding the uterus? 3 _ j t measures 8 to 9 cm in length in nulliparous

women.

b. It weighs 50 to 70 g in the parous woman. C.

In the nullipara, the fundus and the cervix are of equal length,

d. The cervix is composed mainly of muscle. 2 -1 2 . The cervix contains little of which of the follow compo nents?

a. Smooth muscle b. Collagen C.

Proteoglycans

d. Elastin 2 -1 3 . Concerning the endometrium, which of the following is true? U te ru s lu m e n Reproduced, with permission, from Cunningham FG , Leveno K J, Bloom SL, et al: Williams Obstetrics, 23rd ed. N ew York, McGraw-Hill, 2010.

E p ith e liu m

a. Inferior rectal artery

C a p illa rie s

b. Perineal artery

U te rin e g la n d s

C. Pudendal artery

V e n o u s s in u s o id s

d . Posterior labial artery

F u n c tio n a lis la ye r

2 -8 . Which of the following is true concerning the anal sphincters?

S p ira l a rte ry

a. The external anal sphincter receives blood supply from the superior rectal artery.

b. The internal anal sphincter contributes the bulk of anal canal resting pressure. C. The external anal sphincter measures 3 to 4 cm in length.

B a s a l a rte ry B a s a lis la ye r E n d o m e tria l v e in M y o m e triu m

d . The external sphincter remains in a state of constant relaxation.

2 -9 . The perineal body is formed partly by which of the fol lowing muscles?

a. Ischiocavernosusmuscle

R a d ia l a rte ry

A rc u a te a rte ry U te rin e a rte ry

b. Gluteus maximus muscle C.

Bulbocavernosus muscle

d. Levator ani muscle 2 -1 0 . Which of the following is the potential space between the anterior surface of the uterus and the posterior wall o f the bladder? a . Pouch o f Douglas

b. Rectovaginal septum C.

Morison pouch

d. Vesicouterine space

Reproduced, with permission, from Hoffm an BL: Abnormal uterine bleeding. In: Schorge JO , Schaffer JI, Halvorson L M , et al., eds. Williams Gynecology. New York, McGraw-Hill, 2008, Figure 8-2.

a . The basal artery comes directly from the arcuate artery.

b. Spiral arteries extend directly from radial arteries. C. The functionalis layer contains spiral arteries and radial arteries. d. The spiral arteries extend directly from the arcuate artery.

A na tom y and Physiology

2 -1 4 . During postpartum tubal sterilization, which of the

2 -1 7 . The right ovarian vein empties into which of the follow

following correct anatomical information may assist you?

ing veins?

a . The fallopian tube lies anterior to the round liga ment.

b. Renal vein

b. The fallopian tube lies posterior to the uteroovarian ligament. C. The round ligament lies anterior to the fallopian tube.

d. The uteroovarian ligament lies anterior to the round ligament.

2 -1 5 . The artery that runs through this structure (arrow) directly arises from which o f the following?

a. Vena cava C.

Internal iliac vein

d. External iliac vein 2 - 1 8. Ovarian vessels are found in which of the following liga ments? 3. Broad

b. Round C.

Uterosacral

d. Infundibulopelvic 2 -1 9 . The common iliac artery arises directly from which of the following?

< d b b d c ^a^V X

Reproduced, with permission, from Cunningham F G , Leveno KJ, Bloom SL, et al: Williams

M h f^c

Obstetrics, 23rd ed. N ew York, M cGraw-Hill, 2010.

Bc h fc V a

a. Sampson artery

e i Y c Y V a V fh fm

b. Internal iliac artery C.

V fh fm

Uterine artery

d. Obturator artery Reproduced, with permission, from Cunningham F G , Leveno K J, Bloom SL , et al: Williams

2 -1 6 . The ovarian artery is a direct branch ofwhich o f the following vessels?

a. Internal iliac artery b. Aorta C.

Uterine artery

d. External iliac artery

Obstetrics, 23rd ed. N ew York, McGraw-Hill, 2010.

a. Aorta b. External iliac artery C.

Internal iliac artery

d. None o f the above 2 -2 0 . The uterine artery is a main branch o f which o f the following vessels?

a. External iliac artery b. Internal iliac artery C.

C om m on iliac artery

d. Iliolumbar artery

M a te rn a l A n a to m y

2 -2 1 . The internal pudendal artery arises from which o f the

2 -2 5 . The shortest anteroposterior diameter o f the pelvic inlet

following vessels?

is which o f the following?

a. External iliac artery

a. True conjugate

b. Internal iliac artery

b. Obstetrical conjugate

C.

Vaginal artery

d. Inferior epigastric artery 2 -2 2 . From proximal (uterus) to distal (fimbria), the correct progression o f fallopian tube anatomy is which o f the following?

C.

Diagonal conjugate

d. None o f the above 2 -2 6 . The clinical evaluation o f the pelvic inlet requires direct measurement ofwhich diameter?

a. True conjugate b. Obstetrical conjugate C.

Diagonal conjugate

d. Pelvic inlet transverse diameter 2 -2 7 . Engagement occurs when the biparietal diameter of the fetal head descends below the level o f which o f the following?

a. Midpelvis b. Pelvic inlet C.

Pelvic floor

d. Ischial tuberosities 2 -2 8 . In this figure, which o f the following is demonstrated?

Reproduced, with permission, from Cunningham FG , Leveno KL, Bloom SL, et aJ: Williams Obstetrics, 22nd ed. New York, McGraw-Hill, 2006. http://www.accessrnedicine.com. Figure 1.

a. Isthmus, infundibulum, ampulla b. Isthmus, ampulla, infundibulum C.

Infundibulum, ampulla, isthmus

d. Ampulla, infundibulum, isthmus 2~2 3. Which o f the following bone(s) forms the pelvis?

a. Innominate b. Sacrum C.

Coccyx

d . All o f the above

Reproduced, with permission, from Cunningham FG , Leveno K J, Bloom SL, et al: Williams Obstetrics, 23rd ed. N ew York, McGraw-Hill, 2010.

2 -2 4 . Which of the following is true regarding relaxation of the pelvic joints at term in pregnancy? a . It allows for an increase in the transverse diameter of the midpelvis.

b. It results in marked mobility o f the pelvis at term because o f a downward gliding movement o f the sacroiliac (SI) joint. C.

Displacement of the SI joint increases outlet diame ters by 1.5 to 2.0 cm in the dorsal lithotomy position,

d . It is permanent and not accentuated in subsequent pregnancies.

a. Important pelvic outlet diameters b. Ischial tuberosities C.

Midpelvis ischial spines

d. An inadequate obstetrical conjugate

A n a to m y and Physiology

2 -2 9 . What is the narrowest pelvic dimension that must be navigated by the fetal head?

2 -3 1 . Which o f the following is the most common pelvic shape?

a. Inferior strait

a. Gynecoidpelvis

b. Obstetrical conjugate

b. Anthropoid pelvis with gynecoid tendency

C.

Interspinous diameter

d . Transverse diameter o f the pelvic inlet

2 -3 0 . Which o f the following is true o f the midpelvis? a. It is measured at the level o f the ischial tuberosities. b. It usually measures no more than 9 cm. C.

It is measured at the level o f the ischial spines,

d . It usually is the largest pelvic diameter.

C.

Android pelvis

d. Gynecoid pelvis with android tendency

2 -3 2 . The pubococcygeus muscle is now preferably called which o f the following?

a. Puborectalismuscle b. Iliococcygeus muscle C.

Puboperinealis muscle

d. Pubovisceral muscle

M a te rn a l A n a to m y

CHAPTER 2 ANSWER KEY Q u e s tio n num ber

L e tte r answ er

Page c ite d

H e a d e r c ite d

Skin

2 -1 7

a

p. 26

B loo d Vessel

B lood S upply

2 -1 8

d

p. 26

B lood Vessel

In n e rv a tio n

2 -19

a

p. 27

Figure 2-16

p. 15

Labia M a jo ra

2 -2 0

b

p. 27

Figure 2-16

a

p. 16

V estibule

2-21

b

p. 27

Figure 2-16

2 -6

c

p. 18

Vascular and L y m p h a tic S upply

2 -22

b

p. 28

Fallopian Tubes

2 -7

a

p. 20

Figure 2-5

2 -23

d

p. 29

P elvic Bones

2 -8

b

p. 20

A nal S ph in cte r

2 -2 4

c

p. 29

P elvic Joints

2 -9

c

p. 21

P erineal B ody

2 -25

b

p. 31

P elvic In le t

2 -1 0

d

p. 21

Uterus

2-26

c

p. 31

P elvic In le t

2-11

c

p. 21

Size and Shape

2 -2 7

b

p. 31

P elvic in le t

c

Q u e s tio n num ber

L e tte r answ er

Page c ite d

H e a d e r c ite d

2-1

c

p. 14

2 -2

c

p. 14

2 -3

c

p. 15

2 -4

d

2 -5

2 -1 2

a

p. 23

Cervix

2 -28

p. 31

P elvic In le t

2 -1 3

b

p. 24

E n d o m e triu m

2 -29

c

p. 31

M id p e lv is

2 -1 4

c

p. 25

L ig a m e n ts

2 -3 0

c

p. 32

M id p e lv is

2 -1 5

c

p. 26

Figure 2-15

2-31

a

p. 32

P elvic Shapes

2-32

d

p. 32

M u scula r S upp o rt

2 -1 6

b

p. 26

Blood Vessel

implantation, Embryogenesis, and Placental Development 3 - 1 . The normal length o f the average menstrual cycle ranges between which o f the following values?

3 -6 . What is the peak ovarian progesterone production during midluteal phase?

a . 14—28 days

a. 10-20 mg/d

b. 25 -3 2 days

b. 25 -50 mg/d

C.

28-35 days

d. 30-4 0 days 3 - 2 . O f the 2 million oocytes in the human ovary present at birth, approximately how many are present at the onset of puberty?

c. 60—80 mg/d d. 7 5 -1 0 0 mg/d 3 -7 . Which o f the following is the most biologically potent naturally occurring estrogen? a . Estriol

a. 200,000

b. Estrone

b. 300,000

C.

c. 400,000

d. 17|3-estradiol

d. 500,000 3 - 3 . Which hormone is required for the late-stage development o f antral follicles?

Estetrol

3 - 8 . Which o f the following is the endometrial layer that is shed with every menstrual cycle? a . Basalis layer

a. Luteinizing hormone (LH)

b. Decidual layer

b. Follicle-stimulating hormone (FSH)

C.

C. Estradiol

d. Functionalis layer

d. Androstenedione 3 -4 . During the follicular phase o f the menstrual cycle, estrogen is primarily produced by which cells of the dominant follicle?

a. Theca b. Decidual C.

Granulosa

d. Endometrial

Luteinized layer

3 -9 . Which o f the following is the first h istological sign of ovulation?

a. Cessation o f glandular cell mitosis b. Vacuoles at the apical portion o f the secretory nonciliated cells C.

Secretion o f glycoprotein and mucopolysaccharide by secretory nonciliated cells

d. Subnuclear vacuoles and pseudostratification in the basal portion o f the glandular epithelium

3 -5 . The process through which the corpus luteum develops from the remains o f the Graafian follicle is called which o f the following?

a. Luteinization b. Thecalization C. Decidualization

d. Graafian transformation

Im plantation, Em bryogenesis, and Placental D evelopm ent

15

3 -1 0 . In the following figure, the spiral arteries are identified by which letter?

a. A c. C

U terine lum en

d. D E p ith e liu m ----------— •—

''7

C a p illa rie s --------- -------------------------------- i f

i.

> U / i £T * \ \ j

V e n o u s s in u s ---------------

| f it I E nd om etrial g la n d ----------------------------- > 4 l

f

(

JK J y J r

S m w y S u rl n T r ) B ?g " rH ~ t \ J) Ws

F un ctio nalis laye r

A rte ry A -

A rte ry B -

B asalis laye r

A rte ry C M yom e triu m A rte ry D U te rin e a rte ry ------------ -------------------Reproduced, with permission, from Cunningham F G , Leveno KJ, Bloom SL, et al. Williams Obstetrics, 23rd ed. New York: McGraw-Hill, 2010.

3 -1 1 . Among the following histological slides, which one is consistent with the menstrual phase?

a. A b. B c. C d. D

Reproduced, with permission, from Cunningham FG , Leveno KJ, Bloom SL, et al: Williams Obstetrics, 23rd ed. New York, McGraw-Hill, -+,+.

CHAPTER 3

b. B

16

A n a to m y and P h y s io lo g y 3 - 1 2 . Which prostaglandin plays a role in vasoconstriction o f the spiral arteries, leading to menstruation?

3 - 1 5 . The placenta does not perform which o f the following functions for the fetus?

SECTION 2

a. PG Ej

a. Renal

b. P G E2

b. Hepatic

c. P G D 2

C.

d. P G F2a

d. Pulmonary

3 -1 3 . In sequence from letters A to C, please identify the three types o f deciduas in the figure below:

Adrenal

3 - 1 6 . At 5 days postfertilization, the blastocyst is released from which surrounding structure?

a. Basalis, capsularis, parietalis

a. Morula

b. Capsularis, basalis, parietalis

b. Chorion laeve

C.

Parietalis, basalis, capsularis

C.

Zona pellucida

d. Trophectoderm

d. Parietalis, capsularis, basalis

Qda gVX = X^YiV : >b Wfmd ^c Vb c^dc^X gVX < df^dc^X j^aa^

< df^dc^X j^aa^ = X^YiV <

= X^YiV ;

>ldXd adb ^X XVj^hm

Mh f^c

< fj^XVa XVcVa

XVj^hm

Reproduced, with permission, from Cunningham F G , Leveno K J, Bloom SL, er al: W illiams Obstetrics, 23rd ed. New York, M cGraw-Hill, 2010.

3 -1 4 . W hat is the Nitabuch layer? a . A layer o f the decidua composed o f large, distended glands b. An area o f the decidua with large, closely packed epithelioid, polygonal cells C. A zone o f fibrinoid degeneration in which the invad ing trophoblasts meet the decidua d. An area o f superficial fibrin deposition at the bottom o f the intervillous space and surrounding the anchor ing villi

3 - 1 7 . Which of the following gives rise to the chorionic struc tures that transport oxygen and nutrients between fetus and mother?

a. Villous trophoblast b. Interstitial trophoblast C.

Extravillous trophoblast

d. Endovasculartrophoblast

Im plantation, Em bryogenesis, and Placental D evelopm ent

3 -1 8 . Which of the labeled structures in the following figure

3 -2 2 . Remodeling o f maternal spiral arteries by invading tro-

will eventually become the umbilical cord?

phoblasts is completed by which week(s) o f pregnancy?

a.

A

a.

b.

B

b.

12

c.

C

c.

12-16

d.

D

d.

16-18

8

Reproduced, with permission, from Streeter G L. A human embryo [Mateer] o f the presomite period. Contrib Embryol 1920; 9:389.

3 -1 9 . Which o f the following statements is accurate regarding

3 -2 3 . End diastolic blood flow can be identified in the fetal

the chorion frondosum?

umbilical artery by the end of which week o f pregnancy?

a . Is the same as the chorion laeve

a. 10

b. Is the maternal component o f the placenta

b. 12

Is the avascular area that abuts the decidua parietalis

c. 14

d. Is the area o f villi in contact with the decidua basalis

d. 16

C.

3 -2 0 . Maternal regulation o f trophoblast invasion and vascular

3 -2 4 . Regarding the orientation of spiral bloodvessels in rela

growth is mainly overseen by which o f the following?

tionship to the uterus, which o f the following is true?

a. Decidual natural killer (dNK) cells

a . Both arteries and veins are parallel to the uterine wall

b. C D 4 + T cells

b. Both arteries and veins are perpendicular to the uter

C.

d. Cellular adhesion molecules 3 -2 1 . All E X C E P T which o f the following is true regarding fetal fibronectin?

a. Is a matrix metalloproteinase b. Is also known as “trophoblast glue” C.

ine wall

Progesterone

Is a prognostic indicator of preterm labor

d. Plays a role in trophoblast invasion o f the en dometrium

C.

Arteries are perpendicular, and veins are parallel to the uterine wall

d. Veins are perpendicular, and arteries are parallel to the uterine wall

17

18

A n a to m y and P h y s io lo g y

3 -2 5 . The phenomenon that describes how fetal cells can be

SECTION 2

come engrafted in the mother during pregnancy and then be identified decades later is called which o f the following?

a. Microchimerism b. Histocompatibility C.

3 -2 7 . Which o f the following is accurate regarding Meckel diverticulum?

a. Is the allantoic duct remnant b. Is a failure of the right umbilical vein to involute C. Is an extraabdominal remnant o f the umbilical vesicle

d. Is a failure o f the intraabdominal portion o f the um

Hemochorial invasion

bilical vesicle to atrophy

d. Immunological neutrality

3 -2 8 . As shown in this figure, blood coming from the placenta 3 -2 6 . The amnion is composed of all E X C E P T which o f the following?

to the fetus travels from the umbilical vein into which of the following?

a. Lymphatics

a . The portal vein

b. Interstitial collagen

b. The hepatic vein

C.

Basement membrane

d. Acellularzonaspongiosa

C.

The ductus venosus

d. The inferior vena cava

O x y g e n a te d

M ix e d

D e o x y g e n a te d Reproduced, with permission, from Cunningham FG, Leveno KJ, Bloom SL, et al: W illiams Obstetrics, 23rd cd. N ew York, McGraw-Hill, 2010.

m p lan ta tion , Em bryogenesis, and Placental D evelopm ent 3 - 2 9 . The amino acid sequence of die a-subunit of human chorionic gonadotropin (hCG) is identical in all E X C E P T which o f the following?

a. LH b. Corticotropin-releasing hormone (CRH) C.

FSH

3 -3 4 . Which o f the following is true o f bilateral oophorectomy at 9 weeks’ gestation?

a. Will cause miscarriage b. Will cause a significant drop in progesterone levels C.

Will not alter the excretion of urinary pregnanediol

d. None o f the above

d. Thyroid-stimulating hormone (TSH) 3 ~ 3 5 . What is the precursor of placental progesterone? 3 -3 0 . Abnormally low levels ofh C G may be found in which of the following?

a. Down syndrome b. Ectopic pregnancy

a. Fetal high-density lipoprotein (H D L) cholesterol

b. Fetallow-density lipoprotein (LDL) cholesterol C.

Maternal H D L cholesterol

d. Maternal L D L cholesterol

C. Erythroblastosis fetalis

d. Gestational trophoblastic disease 3 - 3 1 . Known biological actions ofh C G include all EX C E PT which o f the following?

3 -3 6 . Besides being anX-linked disorder that affects male fetuses, true statements regarding ichthyosis include which of the following? a . It is seen with fetal adrenal hypoplasia.

a. Maternal thyroid stimulation

b. It is seen with fetal adrenal hyperplasia.

b. Inhibition o f relaxin secretion

C.

C. Sexual differentiation o f the male fetus

d . Rescue and maintenance o f the corpus luteum

It is associated with fetal placental sulfatase deficiency.

d . It is associated with fetal placental aromatase deficiency.

3 - 3 2 . Which of the following has the greatest production rate of any known human hormone?

a. Progesterone b. Human placental lactogen C.

Chorionic adrenocorticotropin

d. hC G 3 - 3 3 . Among placental peptide hormones, which o f the fol lowing has shown a correlation with birthweight? 3. Leptin

b. Activin C.

Inhibin

d. Neuropeptide Y

3 -3 7 . Which of the following conditions is associated with elevated estrogen levels in pregnancy? a. Down syndrome b. Fetal erythroblastosis C.

Glucocorticoid therapy

d . Fetal placental aromatase deficiency

A na tom y and Physiology

CHAPTER 3 ANSWER KEY Q u e s tio n num ber

Q u e s tio n num ber

L e tte r answ er

P age c ite d

The O varian Cycle

3 -1 9

d

p. 52

F o llicu la r o r P re o v u la to ry O varian Phase

D e v e lo p m e n t o f C horion and D ecidua

3 -2 0

a

p. 53

M a te rn a l R e gula tion o f T ro p h o b la st Inva sio n and Vascular G ro w th

3-21

a

p. 53

T roph oblast Invasion o f th e E n d o m e triu m

3 -2 2

c

p. 54

Invasion o f Spiral A rte rie s

3 -2 3

a

p. 56

Fetal C irculation

3 -2 4

c

p. 57

M a te rn a l C irculation

3 -2 5

a

p. 58

Breaks in th e P lacental "B a rrie r"

3 -2 6

a

p. 59

The A m n io n /S tru c tu re

3 -2 7

d

p. 62

Cord D e v e lo p m e n t

3 -2 8

c

p. 62, 90

Cord S tru ctu re and F u n c tio n /F ig u re 4-12

3 -2 9

b

p. 63

hC G /C h em ical C haracteristics

3 -3 0

b

p. 64

S ig n ifica n ce o f a b n o rm a lly hig h or lo w hCG le ve ls

L e tte r answ er

Page c ite d

H e a d e r c ite d

3-1

b

p. 37

3 -2

t

p. 37

3 -3

b

p. 38

F o llicu la r o r P re o v u la to ry O varian Phase

3 -4

c

p. 39

F o llicu la r o r P re o v u la to ry O varian Phase

3 -5

a

p. 40

Luteal or P o s to vu la to ry O varian Phase

3 -6

b

p. 40

Luteal o r P o s to vu la to ry O varian Phase

3 -7

d

p. 41

E strogen and P ro g e ste ro n e A ctio n

3 -8

d

p. 41

The E n d o m e tria l C y c le /P ro life ra tiv e and P re o v u la to ry E nd o m e tria l Phase

H e a d e r c ite d

3 -9

d

p. 42

S ecre to ry o r P o s to vu la to ry E nd o m e tria l Phase

3 -1 0

a

p. 42

Figure 3 -5

3-11

d

p. 38

D ating o f E n d o m e triu m , Figure 3-2

3-31

b

p. 64

K no w n B io lo g ica l F unctions of hCG

3 -1 2

d

p. 44

P ro sta g la n d in s and M e n s tru a tio n

3 -3 2

b

p. 65

3 -1 3

d

p. 45

D ecidual S tru c tu re /F ig u re 3-8

H u m an P lacental L a c to g e n /C h e m ic a l C haracteristics

3 -1 4

c

p. 46

D ecidual H isto lo g y

3 -3 3

a

p. 67

Leptin

3 -1 5

c

p. 47

Im p la n ta tio n , P lacental F o rm a tio n and M e m b ra n e D e v e lo p m e n t

3 -3 4

c

p. 67

P lacental P ro g e ste ro n e P ro d u ctio n

3 -3 5

d

p. 68

P ro g e s te ro n e P ro d u ctio n Rates

3 -3 6

c

p. 71

Fetal P lacental S ulfatase D e fic ie n c y

3 -3 7

b

p. 71

M a te rn a l C ond ition s That A ffe c t P lacental E strogen P ro d u ctio n

3 -1 6

c

p. 47

O vum F e rtiliza tio n and Z ygote C leavage

3 -1 7

a

p. 49

T ro p h o b la st D iffe re n tia tio n

3 -1 8

a

p. 51

Figure 3-15, Early T ro p h o b la st Invasion

Feta! Growth and Development 4 - 1 . Apatient reports that the first day ofher last menstrual period was April 10. O n the basis o f Naegele rule, her due date is which of the following?

4 -4 . At 28 weeks’ gestation, what is the chance o f survival without physical or neurological impairment?

a. 50%

a. January 17

b. 65%

b. July 17

c. 90%

C.

January 3

d. 100%

d. July 3

4 -5 . Which o f the following fetal head diameter measure 4 -2 . The length o f the embryonic period is which o f the following?

a. 6 weeks b. 8 weeks C.

12 weeks

ments is the largest?

a. Bitemporal b. Biparietal C.

Occipitofrontal

d. Occipitomental

d. 14 weeks

4 -6 . The shifting or sliding o f the fetal head bones to permit 4 - 3 . The image below is o f a human embryo at 22 days’ gestation. The area marked “A ” represents which o f the following?

accommodation o f the fetal head to the maternal pelvis is referred to as which o f the following? a. Caput

b. Yielding C.

Molding

d. Flexion 4 -7 . Uteroplacental blood flow at term measures approxi mately which o f the following?

a. 50-100 mL/min b. 100-300 mL/min C.

7 00-900 mL/min

d. 1,200-1,400 mL/min

Reproduced, with permission, from Cunningham FG, Leveno KJ, Bloom SL, etal: Williams Obstetrics, 23rd ed. New York, M cGraw-Hill, 2010.

a. Amnion b. Yolk sac C.

Chorion

d. Bladder

A n a to m y and Physiology Thefollowing image applies to questions 4—8 and 4—9.

4 -1 0 . Which o f the following sequences correctly names the labeled fetal head diameters?

Reproduced, with permission, from Cunningham FG , Leveno K J, Bloom SL, etal: Williams Obstetrics, 23rd ed. N ew York, M cGraw-Hill, 2010.

4 -8 . Which o f the following sequences correctly names the labeled fetal head bones?

a. A is the frontal bone, B is the parietal bone, and C is the occipital bone.

b. A is the parietal bone, B is the occipital bone, and C is the frontal bone. C.

A is the temporal bone, B is the frontal bone, and C is the occipital bone,

d. A is the frontal bone, B is the temporal bone, and C is the parietal bone.

4 - 9 . The suture that is bordered by the bones labeled A and B is which of the following?

Reproduced, with permission, from Cunningham FG , Leveno K J, Bloom SL, et al. Williams Obstetrics, 22nd ed. N ew York: McGraw-Hill, 2005.

a. A is suboccipitobregmatic, B is occipitofrontal, and C is occipitomental.

b. A is occipitofrontal, B is suboccipitobregmatic, and C is occipitomental. C.

A is occipitomental, B is occipitofrontal, and C is suboccipitobregmatic.

d. A is occipitofrontal, B is occipitomental, and C is suboccipitobregmatic.

4 -1 1 . Fetal blood oxygen stores are sufficient to sustain the

a. Frontal suture

fetus for what time length?

b. Coronal suture

3. 30 seconds

C.

Sagittal suture

d. Lambdoid suture

b. 60 -1 20 seconds C.

3—4 minutes

d. 5—7 minutes 4 -1 2 . Carbon dioxide (C O 2 ) transfer from the fetus to the mother is favored due to which o f the following mecha nisms?

a. Oxygen traverses the chorionic villus more rapidly than carbon dioxide.

b. Mild maternal hypoventilation enhances rapid C O 2 transfer. C.

Partial pressure o f C O 2 in the umbilical arteries is less than in maternal intervillous blood,

d . Fetal blood has less affinity for C O 2 than does mater nal blood.

Fetal Grow th and D evelopm ent 4 -1 3 . Oxygen, carbon dioxide, and water are transferred across the placenta via which mechanism?

4 -1 9 . Which o f the following statements about fetal circula tion is correct?

a. Facilitated diffusion

a . Ventricles o f the fetal heart work in series.

b. Simple diffusion

b. The oxygen content o f the blood entering the left ventricle is less than that entering the right ventricle.

C. Active transport

d. Carrier-mediated diffusion 4 -1 4 . Which o f the following hormones blocks maternal pe ripheral uptake and use o f glucose while promoting mo bilization and use o f free fatty acids by maternal tissues? a . Human chorionic gonadotropin (hCG)

b. Human placental lactogen (HPL) C. Relaxin

d. Leptin 4 -1 5 . Which o f the following statements about placental transport is true? 3. The placenta concentrates a large number o f amino acids.

b. Large proteins are freely transferred across the pla centa. C. Copper levels are higher in fetal plasma than mater nal plasma because o f carrier-mediated transfer,

d. Vitamin C crosses the placenta by simple diffusion. 4 -1 6 . Which o f the following immunoglobulins crosses the placenta in large amounts?

C. The right atrium directs entering blood to the left

atrium or right ventricle depending on the oxygen content o f the blood.

d. Most o f the right ventricular output enters the pul monary vasculature.

4 -2 0 . What percentage o f right ventricular output goes to the lungs?

a. 15% b. 30% c. 45% d. 60% 4 -2 1 . When do the distal hypogastric arteries undergo atrophy and obliteration postnatally? 3. D ay o f life 1

b. 3—4 days after birth C. 1 week after birth

d.

6

months o f age

4 -2 2 . The umbilical arteries become which o f the following? 3. Ligamenturn teres

a. Immunoglobulin A

b. Ligamentum venosum

b. Immunoglobulin E

C. Uterine ligaments

C. Immunoglobulin M

d. Umbilical ligaments

d. Immunoglobulin G 4 -2 3. Hemoglobin content o f fetal blood at term is which o f 4 -1 7 . Amnionic fluid is composed largely o f fetal urine starting at what gestational age?

the following? 3.

6

g/dL

a.

1 0

weeks

b.

2 0

weeks

C. 18g/dL

C. 30 weeks

d. 2 4 g/dL

b. 12g/dL

d. 40 weeks 4 -2 4 . Fetal hemopoiesis is first seen in which o f the following? 4 -1 8 . Fetal kidneys start producing urine at what gestational age?

a.

2 0

weeks

b. 16 weeks C. 12 weeks

d.

8

weeks

a. Bone marrow b. Liver C. Yolk sac

d. Kidneys 4 -2 5 . Fetoplacental blood volume at term is which o f the fol lowing? 3. 125 mL/kg

b. 100 mL/kg C. 75 mL/kg

d. 50 mL/kg

A na tom y and Physiology

4 -2 6 . Fetal hemoglobin F is produced at which of the follow

4 -3 2 . Ninety percent of surfactant (dry weight) is which of the

ing sites?

following?

a. Yolk sac

3. Protein

b. Kidney

b. Lipid

C.

Liver

d. Bone marrow 4 -2 7 . Which o f the following statements about immunoglob

C.

Carbohydrate

d. Water

4 -3 3 . Which o f the following statements regarding the sex-

ulin is correct?

determining region is true?

a . The bulk o f maternal IgG transport to the fetus oc

a. It is located on the long arm o f the Y chromosome.

curs in the second trimester.

b. Increased levels o f IgM are found in newborns with congenital rubella, cytomegalovirus infection, or toxoplasmosis. C. The newborn acquires significant passive immunity from the absorption o f humoral antibodies ingested in colostrum.

d. Preterm neonates have an adequate amount o f pro tective maternal antibodies.

b. It directs development o f the testis. C.

d . It is located on chromosomes 4 and 8.

4 -3 4 . Which o f the following stimulates the fetal testes to secrete testosterone?

a. Follicle-stimulating hormone (FSH) b. 5a-dihydrotestosterone C.

4 -2 8 . WTtich o f the following fetal activities, not present until

It is expressed in spermatozoa.

hCG

d. HPL

24 weeks, is the last to develop?

a. Opening the mouth b. Swallowing C. Breathing

d. Sucking

4 -3 5 . You are asked to see a newborn with genital ambigu ity. An image of the newborn is provided. Imaging and blood tests have confirmed that this baby is 46,X X and has a uterus, fallopian tubes, and upper vagina, No testes are present. The diagnosis is which o f the following?

4 -2 9 . In regard to the passage o f unconjugated bilirubin across the placenta, which o f the following is correct?

a . It only passes from the fetal to the maternal side. b . It only passes from the maternal to the fetal side. C. It is not exchanged to any significant degree between mother and fetus,

d. It is bidirectional. 4 -3 0 . W'hich o f the following is correct in regard to fetal urine? a. At term, the fetus produces 650 mL/d. b. It is hypertonic with respect to fetal plasma. C. Hemorrhage and hypoxia result in increased urine output.

d . Fetal kidneys start to produce urine at 6 weeks’ gesta tion.

4 -3 1 . The limits o f fetal viability are determined by which o f the following process? a . Hepatic development

b. Fetal immunocompetence C. Kidney formation

d. Pulmonary growth

Reproduced, with permission, from Cunningham FG , Leveno KJ, Bloom SL , et al: Williams O bstetrics, 23rd ed. N ew York, McGraw-Hill, 2010.

a. Dysgenetic gonads b. True hermaphroditism C.

Male pseudohermaphroditism

d. Female pseudohermaphroditism

Fetal Grow th and D evelopm en 4 - 3 6 . In the case o f 21 -hydroxlase deficiency, which o f the following drugs can be administered to the mother to reduce genital virilization in the fetus?

a. Dexamethasone b. Prednisone

4 -3 7 . A 25-year-old nulligravida presents with a history of primary amenorrhea. She has an extremely short vagina, no axillary hair, and Tanner stage 5 breast development. A i image o f the patient’s perineum is provided below. Which o f the following is the likely diagnosis?

C. Estradiol d. Progesterone

Reproduced, with permission, from Cunningham FG, Leveno KJ, Bloom SL, et al: Williams Obstetrics, 22nd ed. N ew York, McGraw-Hill, 2005.

a . Congenital adrenal hyperplasia

b. Female pseudohermaphroditism C. Androgen insensitivity syndrome

d. Turner syndrome (46,X)

A n a to m y and Physiology

CHAPTER 4 ANSWER KEY Q u e s tio n num ber

L e tte r answ er

Page c ite d

H e a d e r c ite d

4 -2 0

a

p. 89

Fetal C irculation

4-21

b

p. 90

Fetal C irculation

4 -2 2

d

p. 90

Feta! C irculation

4 -2 3

c

p. 91

Fetal B lood

4 -2 4

c

p. 91

Fetal Blood

4 -2 5

a

p. 91

Fetal Blood

4 -2 6

c

p. 92

Fetal B lood

4 -2 7

b

p. 93

O n to g e n y o f th e Fetal Im m u n e R esponse

4 -2 8

d

p. 93

N ervo u s S ystem and S ensory Organs

Figure 4 -9

4 -2 9

d

p. 95

G a stro in te s tin a l S ystem

p. 86

P lacental Transfer

4 -3 0

a

p. 95

U rinary S ystem

d

p. 86

P lacental Transfer

4-31

d

p. 96

Lungs

4 -1 3

b

p. 86

P lacental T ransfer

4 -3 2

b

p. 96

Lungs

4 -1 4

b

p. 87

Fetal N u tritio n

4 -3 3

b

p. 101

Sexual D iffe re n tia tio n

4 -1 5

a

p. 88

Fetal N u tritio n

4 -3 4

c

p. 101

Sexual D iffe re n tia tio n

4 -1 6

d

p. 88

Fetal N u tritio n

4 -3 5

d

p. 102

Figure 4-19

4 -1 7

b

p. 88

Fetal N u tritio n

4 -3 6

a

p. 102

4 -1 8

c

p. 88

Fetal P hysio lo g y

G enital A m b ig u ity o f th e N e w b o rn

4 -1 9

c

p. 89

Fetal C irculation

4 -3 7

c

p. 103

G enital A m b ig u ity o f th e N e w b o rn

Q u e s tio n num ber

L e tte r answ er

Page c ite d

4-1

a

p. 78

D e te rm in a tio n o f G e statio n a l A ge

4-2

b

p. 79

M o rp h o lo g ic a l G ro w th

4 -3

b

p. 79

Figure 4-2

4 -4

c

p. 80

M o rp h o lo g ic a l G ro w th

4 -5

d

p. 81

M o rp h o lo g ic a l G ro w th

4 -6

c

p. 82

M o rp h o lo g ic a l G ro w th

4 -7

c

p. 84

The P lacenta and Feta! G ro w th

4 -8

a

p. 84

Figure 4 -9

4 -9

b

p. 84

Figure 4 -9

4 -1 0

a

p. 84

4-11

b

4 -1 2

H e a d e r c ite d

CHAPTER

Maternal Physiology

+30/32 = 94%

5 -1 . In the nonpregnant woman, the uterus is an almost

5 -4 . In pregnancy, the midline o f the abdominal skin be

solid structure weighing approximately which of the following?

comes especially pigmented and is called which o f the following?

a. 40 g

a. Melasma gravidarum

W) 70 g

W) Lineaalba

c. lOOg

C.

Y) 130 g

d. Linea nigra

5 -2 . Uterine blood flow near term is which o f the following? a. 50-100 mL/min W) 100-300 mL/min

c. 450 to 600 mL/min Y) 1,000-1,400 mL/min

5 -3 . In the image below, cervical eversion is demonstrated. What kind o f epithelium makes up this portion of the cervix?

Chloasma

5 -5 . By the third trimester, which o f the following describes the maternal basal metabolic rate?

a. Decreased by 5% b. Increased by 10% to 20% compared with the non pregnant state C.

Increased an additional 2 5 % above normal preg nancy values in women with twin gestations

d . None o f the above 5 -6 . The graphic illustrates which o f the following points? (LMP = last menstrual period; MP = menstrual period.) .++ (B 29 6 ~

-U X.)

29 2 -

|

28 8 -

w

O

•E

284 -

E tn

280 276 272 MP

MP

LM P

4

8

12

16

W eeks o f p regn ancy Reproduced, with permission, from Cunningham FG , et al: Williams Obstetrics, 22nd ed. N ew York, McGraw-Hill, 2005.

a . Maternal plasma osmolality increases overall in preg Reproduced, with permission, from Cunningham F G , Leveno KJ, Bloom SL, et al: Williams Obstetrics, 23rd ed. N ew York, McGraw-Hill, 2010.

a. Columnar

nancy. W) Maternal plasma osmolality decreases early in preg nancy.

W) Squamous

C. Maternal plasma osmolality is affected most by in creases in sodium.

C. Serous

d. Maternal plasma osmolality does not change during

Y) Brush border cells

pregnancy.

A n a to m y and Physiology 5 -7 . The products o f conception, maternal blood volume expansion, breast gland growth, and uterine growth account for how many grams o f protein accrued during pregnancy?

a. 250 g b. 500 g c. 750 g d. 1,000 g 5 -8 . The graphic concerning insulin and glucose levels dur ing pregnancy suggests which o f the following?

5 -1 0 . Which o f the following occurs during pregnancy? a . 500 m E q o f sodium and 200 m Eq o f potassium are retained.

b. Total serum calcium levels decline during pregnancy. C. The fetal skeleton accrues approximately 70 g of

calcium by term,

d. Serum magnesium levels increase. 5 - 1 1 . As illustrated by this graphic, which o f the following occurs during pregnancy?

Reproduced, with permission, from Cunningham F G , Leveno K J, Bloom SL , etal: Williams Obstetrics, 23rd ed. N ew York, McGraw-Hill, 2010.

a . Hematocrit increases during pregnancy.

b. Red cell volume does not increase until 20 weeks. C. Total blood volume increases 40% during pregnancy,

d. Hematocrit increases due to increased red cell volume relative to plasma volume. 5 - 1 2 . This graphic suggests which o f the following? Reproduced, with permission, from Cunningham F G , Leveno K J, Bloom SL, et al: Williams Obstetrics, 23rd ed. New York, McGraw-Hill, 2010.

a. Mild fasting hyperglycemia b. Postprandial hypoglycemia C. Hypoinsulinemia

d. Hyperinsulinemia 5 - 9 . Concerning fat metabolism during pregnancy, which o f the following is true? a . Increased fat stores are stored peripherally.

b. Triacylglycerol and cholesterol levels are increased, but low-density lipoproteins (LDL) and very lowdensity lipoproteins (V LDL) are decreased. C. Mechanisms responsible for changes in fat levels include lipolytic and decreased lipoprotein lipase activities in adipose tissue,

d. None o f the above.

Reproduced, with permission, from Cunningham FG , Leveno K J, Bloom SL , etal: Williams Obstetrics, 23rd ed. N ew York, M cGraw-Hill, 2010.

a. Serum iron is unchanged in the first trimester. b. Serum ferritin is increased in the second trimester. C. Serum transferrin is increased by the end o f preg nancy.

d. Serum ferritin is increased by the end o f pregnancy.

Maternal Physiology ■Supine

5 - 1 3 . Concerning iron requirements in pregnancy, which o f the following is true?

a. 500 mg o f iron is required for normal pregnancy.

d )>

o E

b. O f the iron requirement, 300 gm is used for the in creased total volume of circulating erythrocytes.

o

C. Iron requirements o f pregnancy are usually not avail

able from iron stores in most women,

d. 500 mg o f iron is transferred to the placenta and fetus. 5 -1 4 . Concerning immunological function during pregnancy, which of the following is true?

a. T h l response is suppressed. b. There is up regulation ofT-cytotoxic cells.

v\

fd CL

2 )- + (

7

2 ), + (

7.05 -

--------1--------1--------1--------1--------1--------1 10 8 6 4 2 0 hV a W ^d e m g ^X V a e fd [^a

gXdf

This article was published in American Journal o f Obstetrics & Gynecology, Vol. 169, N o . 4, FA Manning, R Snijders, C R Harm an, et al: Fetal biophysical profile score. VI. Correlation with antepartum umbilical venous fetal pH , pp. 7 5 5 -7 6 3 , © Elsevier, 1993.

a . Score o f 8/10 is associated with an umbilical venous pH o f 7.30 or below.

b. Score o f 6/10 is indication to proceed with delivery. C.

1 5 - 1 7 . Which o f the following is true of Doppler velocimetry, as applied to fetus surveillance?

Score o f 2/10 is associated with acidemia.

d. Score o f 4/10 is similar to 6/10 in regard to umbili cal venous pH.

It focuses primarily on the systolic waveform of blood flow.

d . It is recommended by the American College o f O b stetricians and Gynecologists (ACOG) in the pres ence of other abnormal fetal tests.

Antepartum Assessment 1 5 - 1 8 . O f the three umbilical artery velocimetry waveforms in this image, which is normal?

a. A

1 5 -2 1 . Concerning ductus venosus Doppler velocimetry, which o f the following is true?

b. R E D F or absent end diastolic flow (AEDF) is an

c. C

early finding in fetal hypoxemia.

d. None o f the above C.

R E D F or A ED F is associated with profound fetal metabolic collapse,

d . It is currently recommended for surveillance in all growth-restricted fetuses.

Reproduced, with permission, from Cunningham FG , Leveno KJ, Bloom SL, et al. W illiams Obstetrics, 23rd ed. New York: McGraw-Hill, 2010.

1 5 - 1 9 . In the figure in question 15-18, which waveform is associated with reversed end diastolic flow (RED F) ?

a. A

1 5 -2 2 . Concerning uterine artery Doppler velocimetry, which o f the following is true?

a . Vascular resistance is decreased in the first half of pregnancy.

b. B c. C d. None o f the above 1 5 -2 0 . Concerning middle cerebral artery (MCA) Doppler velocimetry, which o f the following is true?

a. It is superior to the modified BPP in forecasting pregnancy outcomes.

b. It was found inferior to amniocentesis and amnionic fluid spectral analysis for predicting fetal anemia. C.

It is useful for detection and management o f fetal anemia o f any cause.

d. In those with brain sparing, decreased blood flow from reduced cerebrovascular impedance is de tected.

b. It is not helpful in assessing pregnancies at high risk due to uteroplacental insufficiency. C.

Low-resistance patterns are associated with preeclampsia.

d . Low-resistance patterns have been linked to a vari ety o f pregnancy complications. 1 5 -2 3 . Concerning antenatal fetal testing, which of the follow ing is true?

a. It is clearly beneficial. b. It should be considered experimental. C.

Despite its widespread use, the N S T is inferior to umbilical artery Doppler velocimetry at predicting marginally poor cognitive outcomes.

d. A C O G recommends contraction stress testing as the best test to evaluate fetal well-being.

CHAPTER 15

a . It is inferior to umbilical artery Doppler velocimetry in predicting fetal outcome.

b. B

85

86

Antepartum 1 5 - 2 4 . During acoustic stimulation testing, which fetal response is measured?

1 5 -2 5 . W hat controls fetal heart rate accelerations? a . Autonomic function at brainstem level

SECTION 3

a . Breathing

b. Aortic baroreceptor reflexes

b. Heart rate

C. Carotid baroreceptor reflexes

C. Eye movement

d. Humeral factors such as atrial natriuretic peptide

d. Body movement

Antepartum Assessment

87

Chapter 15 Answer Key L e tte r answ er

Page cite d

H e a d e r c ite d

1 5 -1 4

c

p. 342

Figure 1 5 -9

1 5 -1 5

a

p. 342

M o d ifie d B io p h ysica l Profile

C linical A p p lic a tio n

1 5 -1 6

d

p. 343

D o p p le r V e lo c im e try

p. 335

Figure 15-2

1 5 -1 7

b

p. 343

D o p p le r V e lo c im e try

a

p. 336

Fetal B re a th in g

1 5 -1 8

a

p. 344

Figure 15-11

1 5 -7

b

p. 337

C ontraction Stress Testing

1 5 -1 9

c

p. 344

Figure 15-11

1 5 -8

b

p. 337

C on tra ctio n Stress Testing

1 5 -2 0

c

p. 344

M id d le Cerebral A rte ry

1 5 -9

b

p. 338

N o rm a l N onstress Tests

15-21

c

p. 345

Ductus Venosus

1 5 -1 0

b

p. 338

Figure 15-5

1 5 -2 2

a

p. 345

U te rin e A rte ry

15-11

c

p. 339

A b n o rm a l N onstress Tests

1 5 -2 3

b

p. 345

S ig n ifica n ce o f Fetal Testing

1 5 -1 2

d

p. 340

F a lse -N o rm a l N onstress Tests

1 5 -2 4

b

p. 340

A coustic S tim u la tio n

1 5 -1 3

c

p. 341

B io p h ysica l P rofile

1 5 -2 5

a

p. 338

Fetal H e a rt A cc e le ra tio n

Page c ite d

H e a d e r c ite d

15-1

b

p. 3 3 4

In tro d u c tio n

15-2

d

p. 3 3 4

In tro d u c tio n

1 5-3

b

p. 335

Fetal M o v e m e n ts

1 5 -4

c

p. 335

1 5-5

c

1 5 -6

CHAPTER 15

Q u e s tio n num ber

L e tte r answ er

Q u e s tio n num ber

88

CHAPTER 16

Fetal Imaging 1 6 -1 . Which o f the following statements is correct with re gard to the relationship between tissue penetration and image resolution in sonography? a . Higher-frequency transducers penetrate tissue more effectively.

b. Higher-frequency transducers yield better image resolution. C.

Lower-frequency transducers yield better image resolution,

d. None o f the above.

1 6 -5 . A 39-year-old multipara with chronic hypertension is noted to have lagging fundal growth at 29 weeks’ gesta tion and is referred for sonographic examination. Fetalgrowth restriction is suspected based on a sonographic fetal weight estimate o f less than the third percentile for this gestational age. After which interval is it appropri ate to repeat the sonographic examination to evaluate for interval fetal growth? a . 2 days b. 1 week C.

1 6 -2 . In the first trimester of pregnancy, the most accurate predictor o f gestational age is a measurement o f which o f the following?

a. Gestational sac b. Yolk sac C.

Crown rump length

3 weeks

d. 5 weeks 1 6 -6 . This sonographic image is taken from a pregnancy in which hydramnios is suspected. I f measuring the largest vertical pocket for confirmation, the distance between the two calipers must exceed which value?

d. Fetal abdominal circumference 1 6 -3 . W ith transvaginal sonography, cardiac motion is usu ally observed when the embryo reaches what length?

a. 3 mm b. 5 mm C.

10 mm

d. 15 mm 1 6 -4 . Which o f the following biometric parameters most accurately predicts gestational age in the second trimester?

a. Biparietal diameter b. Head circumference C.

Abdominal circumference

d. Femur length

Reproduced, with permission, from Cunningham F G , Leveno K J, Bloom SL, et al: W illiams Obstetrics, 23rd ed. N ew York, McGraw-Hill, 2010.

a. 6 cm b. 8 cm C.

10 cm

d. 12 cm 1 6 - 7 . Adequate visualization o f which o f the following ax ial views o f the fetal head is essential to exclude most anomalies o f the brain that can be identified in utero?

a. Transthalamic b. Transventricular C.

Transcerebellar

d. All o f the above

Fetal Imaging 1 6 - 1 0 . The following image demonstrates a lumbosacral meningomyelocele in the sagittal view. Associated cra nial abnormalities include all E X C E P T which of the following?

Reproduced, with permission, from Cunningham FG , Leveno K J, Bloom SL, etal: Williams Obstetrics, 23rd ed. N ew York, McGraw-Hill, 2010. Reproduced, with permission, from Cunningham FG , Leveno K J, Bloom SL, et al: Williams Obstetrics, 23rd ed. New York, McGraw-Hill, 2010.

a . The cerebellar diameter value is smaller than gesta tional age throughout pregnancy.

b. The cerebellar diameter value is larger than gesta tional age throughout pregnancy. C.

The cerebellar diameter value is approximately equivalent to gestational age until approximately 22 weeks, after which it is smaller than the number o f weeks gestation.

d . The cerebellar diameter value is approximately equivalent to gestational age until approximately 22 weeks, after which it is larger than the number o f weeks gestation.

a. Efifacement of the cisterna magna b. Scalloping o f the frontal bones C.

d. Ventriculomegaly 1 6 - 1 1 . A 2 1-year-old primigravida undergoes routine sono graphic examination at 18 weeks’ gestation. The lateral ventricular atrial measurements are 12 mm and 13 mm on the right and left sides, respectively. The evaluation of this condition should include which o f the follow ing?

a. Amniocentesis for fetal karyotype b. Targeted sonography for associated abnormalities C.

1 6 - 9 . In what region o f the fetal skull are cephaloceles most commonly identified?

a. Occipital b. Frontal C.

Ethmoidal

d. Parietal

Dandy-Walker malformation

Testing for congenital infections

d. All of the above 1 6 - 1 2 . In the patient described in question 16-11, an evalu ation is performed but no etiology is identified. The patient asks you what neurologic outcomes can be ex pected. If this finding is isolated, correct statements during your counseling include which o f the following? a . Cognitive development will be completely normal.

b. M ost have developmental delay. C. Mental retardation is expected and can be severe. d. The prognosis is highly variable, limiting the ability to predict outcome accurately.

CHAPTER 16

1 6 - 8 . The following sonographic image demonstrates the normal posterior fossa structures at approximately midpregnancy. Which of the following statements is accurate regarding the relationship between the trans verse cerebellar distance measured in millimeters (C) and gestational age in weeks? CM = cisterna magna.

89

Antepartum 1 6 - 1 3 . If cystic hygromas are diagnosed in the first trimester, what is the most commonly associated aneuploidy? 3. Monosomy X

b. Trisomy 18 C.

1 6 - 1 5 . The following image is a transverse view o f the fetal ab domen demonstrating a gastroschisis defect. Which of the following statements is true regarding this anomaly? (Arrow points to the umbilical cord insertion site.)

Trisomy 21

d. Trisomy 13 1 6 - 1 4 . The following sonographic image demonstrates a nor mal four-chamber view o f the fetal heart. Which of the following cardiac malformations may not be detected when only this view is obtained? RV = right ventri cle; LV = left ventricle; RA = right atrium; LA = left atrium; FO = foramen ovale; A = aorta.

Reproduced, with permission, from Cunningham F G , Leveno K J, Bloom SL, et al: Williams Obstetrics, 23rd ed. N ew York, M cGraw-Hill, 2010.

a . It is more commonly seen with advanced maternal age.

b. It is associated with an increased risk for aneuploidy. C. There is an increased risk for fetal-growth restric tion.

d. The survival rate is less than 50%. 1 6 - 1 6 . Compared with gastroschisis defects, omphaloceles are more likely to have which o f the following?

a. Involve the fetal liver b. Be associated with aneuploidy C.

Be a component o f a genetic syndrome

d. All of the above Reproduced, with permission, from Cunningham FG , Leveno KJ, Bloom SL, et al: Williams Obstetrics, 23rd ed. N ew York, M cGraw-Hill, 2010.

a. Transposition ofthe greatvessels b. Hypoplastic left heart C.

Atrioventricular septal defect (endocardial cushion defect)

d. Ebstein anomaly

Fetal Imaging 1 6 - 1 9 . The fetal kidneys seen in this sonographic image are enlarged and echogenic, consistent with a diagnosis o f infantile polycystic kidney disease. What pattern of inheritance does this condition follow?

Reproduced, with permission, from Cunningham F G , Leveno KJ, Bloom SL, et al: Williams

Reproduced, with permission, from Cunningham FG, Leveno K J, Bloom SL, et al: Williams

Obstetrics, 23rd ed. New York, McGraw-Hill, 2010.

Obstetrics, 23rd ed. New York, McGraw-Hill, 2010,

a. It is usually an isolated finding.

a. Autosomal dominant

b. There is no increased risk for fetal aneuploidy.

b. Autosomal recessive

C. It is usually not detected prior to 24 weeks’ gesta tion.

C.

X-linked recessive

d. Polygenic

d. Hydramnios is uncommon in this setting. 1 6 -1 8 . The production o f amnionic fluid is largely from the placenta and membranes until the fetal kidneys assume this role at what gestational age? a . 12 weeks

b. 14 weeks C.

18 weeks

d. 22 weeks

1 6 - 2 0 . What is the most common cause of neonatal hy dronephrosis?

a. Ureteropelvic junction obstruction b. Collecting system duplication C.

Ureterovesical junction obstruction

d. Bladder outlet obstruction

CHAPTER 16

1 6 - 1 7 . A 39-year-old multipara who had a normal fetal sono graphic examination at 18 weeks is referred for a second examination at 28 weeks for increased fundal height. The following image of the fetal abdomen is obtained. Which o f the following statements is accurate regarding this finding?

91

Antepartum 1 6 - 2 1 . An ultrasound performed at 20 weeks’ gestation demonstrates the following finding in the fetal kidneys. In addition to bilateral pyelectasis, the bladder and proximal urethra are dilated in this male fetus. What is the most likely diagnosis?

1 6 -2 3 . The umbilical artery systolic to diastolic ratio (S/D ratio) changes in what way throughout pregnancy?

a. It increases with advancing gestation. b. It decreases with advancing gestation. C. It remains approximately constant. d . It follows a variable and unpredictable pattern. 1 6 - 2 4 . A 21-year-old primigravida at 34 weeks’ gestation is re ferred for suspected fetal-growth restriction. An umbili cal artery Doppler study is performed and the following waveform is obtained. What is the most appropriate management plan?

Reproduced, with permission, from Cunningham F G , Leveno K J, Bloom SL, et al: Williams Obstetrics, 23rd ed. New York, McGraw-Hill, 2010.

a . Urethral atresia

Reproduced, with permission, from Cunningham FG , Leveno K J, Bloom SL , etal: Williams Obstetrics, 23rd ed. N ew York, McGraw-Hill, 2010.

b. Posterior urethral valves C.

Ureterovesical junction obstruction

d. Ectopic ureterocele

a . Outpatient follow-up in 1 week

b. Biophysical profile C.

1 6 -2 2 . Compared with traditional 2-D sonography, 3- and 4-D sonography are superior in which o f the following regards?

a. Improved detection o f neural-tube defects

Amniocentesis for fetal lung maturity

d. Expedited delivery 1 6 -2 5 . Middle cerebral artery velocimetry is most useful as an adjunct to sonography in which clinical situation?

b. More accurate for fetal echocardiography

a. Suspected fetal anemia

C. Better characterization o f placenta and umbilical cord anomalies

b. Fetal-growth restriction

d. None ofthe above

d. All o f the above

C.

Intracranial anomalies

Fetal Imaging

93

Chapter 16 Answer Key L e tte r answ er

Page c ite d

1 6 -1 5

c

p. 358

N o rm a l and A b n o rm a l Fetal A n a to m y

1 6 -1 6

d

p. 359

N o rm a l and A b n o rm a l Fetal A n a to m y

1 6 -1 7

c

p. 359

N o rm a l and A b n o rm a l Fetal A n a to m y

S on o g ra p h y in O bstetrics

1 6 -1 8

c

p. 360

N o rm a l and A b n o rm a l Fetal A n a to m y

N o rm a l and A b n o rm a l Fetal A n a to m y

1 6 -1 9

b

p. 360

N o rm a l and A b n o rm a l Fetal A n a to m y

N o rm a l and A b n o rm a l Fetal A n a to m y

1 6 -2 0

a

p. 360

N o rm a l and A b n o rm a l Fetal A n a to m y

N o rm a l and A b n o rm a l Fetal A n a to m y

16-21

b

p. 361

N o rm a l and A b n o rm a l Fetal A n a to m y

N o rm a l and A b n o rm a l Fetal A n a to m y

1 6 -2 2

d

p. 361

N o rm a l and A b n o rm a l Fetal A n a to m y

3- and 4 -D im e n s io n a l S onog raph y

1 6-23

b

p. 363

D o p p le r

1 6 -2 4

d

p. 363

D o p p le r

1 6 -2 5

a

p. 365

D o ppler

P age c ite d

H e a d e r c ite d

16-1

b

p. 3 4 9

S on o g ra p h y in O bstetrics

1 6 -2

c

p. 3 5 0

S on o g ra p h y in O bstetrics

1 6 -3

b

p. 351

S on o g ra p h y in O bstetrics

1 6 -4

a

p. 352

S on o g ra p h y in O bstetrics

1 6 -5

c

p. 353

S on o g ra p h y in O bstetrics

1 6 -6

b

p. 353

1 6 -7

d

p. 354

1 6 -8 1 6 -9 1 6 -1 0 1 6-11

d a c d

p. 354 p. 3 5 4 p. 355 p. 355

1 6 -1 2

d

p. 355

N o rm a l and A b n o rm a l Fetal A n a to m y

1 6 -1 3

c

p. 356

N o rm a l and A b n o rm a l Fetal A n a to m y

1 6 -1 4

a

p. 357

N o rm a l and A b n o rm a l Fetal A n a to m y

H e a d e r c ite d

CHAPTER 16

Q u e s tio n num ber

L e tte r answ er

Q u e s tio n num ber

H M t t : iliiffilSMBBMl v, l SH-

M ftS g O g H K i

« ^

I B I

iff*? .......... . •LT n r-TTrrrr^fm"

: s%

m

-

;.,

.

..................... .i l l I f t m b t ^

.,

PH I 1 I jail If I ki B * 1“ !&3

i f c »

I °

B B B B fll

'

-

____________

- v

-i-

* i . '. l hfV (V b c ^d c ^X g V a^c

b. Shorter induction-to-delivery time C. Less frequent meconium passage in utero

d . Ease o f placement and removal 2 2 - 1 9 . Most women who undergo membrane stripping at term enter labor within how many hours?

a. 12 b. 24 C.

72

d. 96 Reproduced, with permission, from Cunningham F G , Leveno K J, Bloom SL, et al: Williams Obstetrics, 23rd ed. N ew York, McGraw-Hill, 2010.

a . The cesarean delivery rate is increased compared with that with prostaglandins.

b. Chorioamnionitis develops in at least 25% of women so treated. C.

Induction-to-delivery times may be decreased com pared with prostaglandins.

d. The risk for preterm birth in subsequent pregnan cies is increased.

2 2-2 0. In general, oxytocin infusions should be discontinued if the number o f contractions in 1 0 minutes persists with a frequency greater than how much?

a. 3 b. 5

c. 7 d. 10 2 2 - 2 1 . What is the mean half-life o f oxytocin?

a.

2 minutes

b.

5 minutes

C.

10 minutes

d. 2 0 minutes

Labor Induction 2 2 - 2 2 . Potential benefits o f a high-dose oxytocin regimen (4.5-6 m U /m L) compared with a low-dose regimen (0.5—1.5 mU/m L) include which o f the following?

2 2 - 2 4 . On average, epidural analgesia prolongs the active phase oflabor by how long?

a.

1 hour

a . Decreased admission-to-delivery intervals

b. 90 minutes

b. Fewer failed inductions

C.

C.

Lower rates o f intrapartum chorioamnionitis

2 hours

d. 3 hours

d. All o f the above 2 2 - 2 3 . At what oxytocin infusion dosage does free-water clear ance begin to decrease markedly?

2 2 - 2 5 . Elective amniotomy increases the risk for which ofthe following? a . Late decelerations

a. 10 m U /m L

b. Placental abruption

b. 20 m U /m L

C.

C.

36 m U /m L

d. 48 m U /m L

Chorioamnionitis

d. Cesarean delivery

125

Labor and Delivery

Chapter 22 Answer Key L e tte r answ er

Page c ite d

H e a d e r c ite d

Q u e s tio n num ber

L e tte r answ er

Page c ite d

H e a d e r c ite d

22 -1

c

p. 500

In tro d u c tio n

2 2 -1 6

c

p. 503

P re in d u c tio n Cervical R ipening

2 2 -2

b

p. 5 0 0

Labor In d u c tio n

2 2 -1 7

c

p. 5 0 4

P re in d u c tio n Cervical R ipening

2 2 -3

d

p. 501

Labor In d u c tio n

2 2 -1 8

d

p. 504

P re in d u c tio n Cervical R ipening

2 2 -4

d

p. 501

Labor In d u c tio n

2 2 -1 9

c

p. 506

2 2 -5

c

p. 501

Labor In d u c tio n

Labor In d u c tio n and A u g m e n ta tio n w ith O xytocin

2 2 -6

a

p. 502

Table 22-1

2 2 -2 0

b

p. 506

Labor in d u c tio n and A u g m e n ta tio n w ith O xytocin

2 2 -7

a

p. 502

P re in d u c tio n C ervica R ipening

2 2-2 1

b

p. 506

2 2 -8

c

p. 502

P re in d u c tio n C ervica R ipening

Labor In d u c tio n and A u g m e n ta tio n w ith O xytocin

2 2 -9

b

p. 502

P re in d u c tio n C ervica R ip e ning

2 2 -2 2

d

p. 506

2 2 -1 0

c

p. 503

P re in d u c tio n Cervica R ipening

Labor In d u c tio n and A u g m e n ta tio n w ith O xytocin

2 2-1 1

a

p. 503

P re in d u c tio n Cervica R ipening

2 2 -2 3

b

p. 5 0 7

Labor In d u c tio n and A u g m e n ta tio n w ith O xytocin

2 2 -1 2

b

p. 503

P re in d u c tio n Cervica R ipening

2 2 -2 4

a

p. 508

2 2 -1 3

a

p. 503

P re in d u c tio n C ervica R ip ening

Labor In d u c tio n and A u g m e n ta tio n w ith O xytocin

2 2 -1 4

d

p. 503

P re in d u c tio n C ervica R ipening

2 2 -2 5

c

p. 508

2 2 -1 5

d

p. 503

P re in d u c tio n C ervica R ipening

Labor In d u c tio n and A u g m e n ta tio n w ith O xytocin

Q u e s tio n num ber

Forceps Delivery and Vacuum Extraction 2 3 -1 . The opening in this forceps blade serves which of the following functions?

2 3 -5 . Which ofthe following describes forceps that are applied to the fetal head with the scalp visible at the introitus without manual separation o f the labia?

a. Midforceps

b. Low forceps C.

Inlet forceps

d. Outlet forceps 2 3 -6 . Which o f the following describes forceps that are applied to the fetal head at + 1 station?

a. Midforceps

b. Low forceps C.

Inlet forceps

d. Outlet forceps a. Protects the fetal ears

b. Protects the fetal scalp C.

2 3 -7 . The pair of forceps shown below is ideally suited for which o f the following obstetrical situations?

Allows blades to grip the fetal head firmly

d . Offers a smaller metal surface area against the fetal skull

2 3 -2 . In current obstetrics, forceps deliveries are categorized into one o f the following three groups?

a . High forceps, midforceps, low forceps

b. Midforceps, low forceps, outlet forceps C.

Inlet forceps, midforceps, outlet forceps

d. Inlet forceps, low forceps, outlet forceps 2 3 -3 . Which o f the following is true ofhigh forceps delivery? a . Indicated for fetal distress

a. Rotation o f a fetus from O T to OA

b. Delivery o f a mentum posterior fetus

b. Forceps applied at + 1 station

C. Delivery o f a fetus with a molded head

C. N o role in modern obstetrics

d. Delivery o f a fetus with a rounded head

d . Forceps applied when head is engaged

2 3 -4 . In general, the categories offorceps delivery are defined primarily by which o f the following?

a. Fetal station b. Type o f forceps used C.

Maternal pelvic shape

d. Degree o f fetal head molding

128

Labor and Delivery 2 3 -8 . This pair o f forceps is ideally suited for which of the following obstetrical situations?

2 3 - 1 2 . Advantages to elective forceps delivery include which o f the following?

a . Lower rates o f fetal acidosis b. Prevention of perineal laceration C. Lower rates o f postpartum urinary retention

d. None o f the above 2 3 - 1 3 . Prerequisites for forceps application include all E X C E P T which of the following?

a. Head is engaged b. Membranes are ruptured C. Cervix is completely dilated

d. Late fetal heart rate decelerations are absent a. Rotation o f a fetus from O T to OA b. Delivery of a mentum posterior fetus C.

Delivery o f a fetus with a molded head

d . Delivery o f a fetus with a rounded head 2 3 - 9 . This forceps is ideally suited for which of the following obstetrical situations?

2 3 - 1 4 . Prior to low forceps delivery, which o f the following preparations are routinely implemented? 3. Mediolateral episiotomy

b. Enema to empty the rectum C. Catheterization to empty the bladder

d. All o f the above 2 3 - 1 5 . During forceps placement, positioning o f the opera tor’s hand, as shown, serves what role?

a. Rotation of a fetus from O T to OA b. Delivery o f a mentum posterior fetus C.

Delivery o f a fetus with a molded head

d . Delivery o f a fetus with a rounded head 2 3 - 1 0 . Maternal indications for forceps delivery include all E X C E P T which o f the following?

a. Heart disease b. Chorioamnionitis C.

Spinal cord injury

d. Pelvic floor protection 2 3 - 1 1 . Fetal indications for forceps delivery include which o f the following?

a. Fetal coagulopathy b. Fetal congenital heart block C. Nonreassuring fetal heart rate pattern d . Protection of fragile preterm infant head

Reproduced with permission from Cunningham F G , et al: W illiams Obstetrics, 22nd ed. N ew York: McGraw-Hill, 2005, Figure 23-7.

a. Protects the fetal ear b. Guides the forceps into position C.

Identifies the ischial spines

d . Provides fetal scalp stimulation during the proce dure

Forceps Delivery and Vacuum Extraction 2 3 - 1 6 . When correctly applied to a fetus in an occiput anterior position, forceps align along which fetal head diame ter?

2 3 - 2 2 . During forceps delivery of a fetus with a face presenta tion, blades should be swept upward when which of the following passes beneath the symphysis?

3. Bitemporal

3. Chin

b. Occipitofrontal

b. Brow

C. Occipitomental

C. Upper lip

d. Suboccipitobregmatic

d. Base o f the nose

2 3 - 1 7 . During operative delivery o f afetus from a + 2 station and ROA position, movements of the forceps should follow which sequence?

2 3 - 2 3 . Maternal morbidity with forceps delivery is most closely predicted by which o f the following?

a. Fetal station

a . Rotation, outward traction, upward traction

b. Maternal parity

b. Upward traction, rotation, outward traction

C. Degree o f fetal distress

C.

Downward traction, rotation, upward traction

d. Downward traction, rotation, outward traction 2 3 - 1 8 . Ideally, forceps traction should be applied in which o f the following manners?

a. Continuously b. Intermittently and with contractions C.

Intermittently and between contractions

d . Intermittently with cycles o f 10 seconds of traction followed by 1 minute o f rest 2 3 - 1 9 . Which o f the following is T R U E regarding manual rotation o f the fetal head from occiput posterior to occiput anterior positions?

a . It has no role in modern obstetrics. b. It should be attempted prior to membrane rupture. C. Disengagement o f the fetal head should be avoided,

d. Forceps are usually applied following successful rotation.

d. Degree offetal head molding 2 3 - 2 4 . Forceps delivery, compared with spontaneous vaginal delivery, is associated with higher short-term rates of which o f the following maternal complications?

a. Episiotomy b. Anal incontinence C. Urinary incontinence

d. All o f the above 2 3-2 5 . For the fetus, forceps delivery, compared with sponta neous vaginal delivery, is associated with higher rates of all E X C E P T which o f the following complications?

a. Facial palsy b. Impaired intelligence C. Brachial plexus injury

d. Intracranial hemorrhage 2 3 -2 6 . Factors associated with a failed trial o f forceps and need for cesarean delivery include which o f the following? a. Advanced parity

2 3 - 2 0 . Compared with forceps delivery from an occiput an terior position, which o f the following is T R U E o f delivery from an occiput posterior position?

a. Equal rates of episiotomy b. Higher rates offetal facial palsy C. Lower rates o f fetal Erb palsy

d. Equal rates ofvaginal laceration 2 3 - 2 1 . When correctly applied to a fetus with a face presenta tion, forceps align along which fetal head diameter?

a. Bitemporal b. Occipitofrontal C.

Occipitomental

d. Suboccipitobregmatic

b. Coexistent chorioamnionitis C. Poor maternal pushing efforts

d. Absence o f regional or general anesthesia 2 3 - 2 7 . With vacuum extraction, a metal cup compared with a soft cup is associated with significantly higher rates of which o f the following?

a. Cephalohematoma b. Birth canal trauma C. LowApgar scores

d. None o f the above 2 3 - 2 8 . In general, vacuum extraction would be contraindi cated in all E X C E P T which o f the following clinical settings?

a. 30-week fetus b. Fetal thrombocytopenia C. Occiput transverse presentation

d. Inability to assess fetal head position

130

Labor and Delivery 2 3 - 2 9 . With vacuum extraction, correct cup placement is described by which of the following?

2 3 - 3 0 . Compared with forceps delivery, vacuum extraction is associated with lower rates o f which o f the following?

a . Centered across the sagittal suture.

a. Neonatal jaundice

b. Placed over the posterior fontanel.

b. Cephalohematoma

C. I f ROA, the cup is placed on left fetal parietal bone,

C. Third-degree laceration

d. Traction axis is aligned with the suboccipitobreg

d . Neonatal retinal hemorrhage

matic diameter.

Forceps Delivery and Vacuum Extraction

Chapter 23 Answer Key Q u e s tio n num ber

L e tte r answ er

Page c ite d

H e a d e r c ite d

Forceps D esign

2 3 -1 7

a

p. 5 1 4

Traction

p. 511

C lassification o f Forceps D e liv e ry

2 3 -1 8

b

p. 5 1 4

Traction

2 3 -1 9

c

p. 516

M a n u a l R otation

c

p. 511

C lassification o f Forceps D e liv e ry

2 3 -2 0

b

p. 5 1 6

Forceps D e liv e ry o f O cciput P osterior

2 3 -4

a

p. 511

C lassification o f Forceps D e liv e ry

2 3-21

c

p. 518

Face P re se n ta tio n Forceps D e live ry

2 3 -5

d

p. 513

Table 23-1

2 3 -2 2

a

p. 518

2 3 -6

a

p. 513

Table 23-1

Face P re se n ta tio n Forceps D e live ry

2 3 -7

d

p. 512

Function o f Forceps

2 3 -2 3

a

p. 5 1 9

Lacerations and E p isio to m y

2 3 -8

c

p. 512

Function o f Forceps

2 3 -2 4

d

p. 519

2 3 -9

a

p. 517

O cciput Transverse R otation

U rinary and Fecal In c on tinen ce

2 3 -1 0

d

p. 512

In d ica tio n s fo r Forceps

2 3 -2 5

b

p. 520

P erinatal M o rb id ity

2 3 -1 1

c

p. 512

In d ica tio n s fo r Forceps

2 3 -2 6

d

p. 522

Trial o f Forceps and Failed Forceps

2 3 -1 2

d

p. 513

E lective and O u tle t Forceps

2 3 -2 7

d

p. 522

Vacuum Extraction

2 3 -1 3

d

p. 513

P re re q u isite s fo r Forceps A p p lic a tio n

2 3 -2 8

c

p. 522

In d ica tio n s and P re re q u isite s fo r V acuum D e live ry

2 3 -1 4

c

p. 513

P re p a ra tio n fo r Forceps D e liv e ry

2 3 -2 9

a

p. 523

Technique

2 3 -1 5

b

p. 513

Forceps A p p lic a tio n

2 3 -3 0

c

p. 5 2 4

C om parison o f V acuum Extraction w ith Forceps

2 3 -1 6

c

p. 513

Forceps A p p lic a tio n

Q u e s tio n num ber

L e tte r answ er

Page c ite d

H e a d e r c ite d

23-1

c

p. 511

2 3 -2

b

2 3 -3

Breech Presentation and Delivery 2 4 - 1 . W hat percentage o f singletons present breech?

2 4 -5 . This image shows which type o f breech presentation?

a. 1% b. 3-4% c. 9-10%

d. 15% 2 4 -2 . Factors associated with breech presentation include all E X C E P T which of the following?

a. Nulliparity b. Placenta previa C.

Fetal malformations

d . Amnionic fluid abnormalities 2 4 -3 . Which o f the following statements is accurate regard ing the prevalence o f breech presentation?

a . It is stable throughout pregnancy. b. It increases with gestational age. C. It decreases with gestational age.

d . It approximates 80% in the 24th week. 2 4 - 4 . This image shows which type ofbreech presentation?

Reproduced, with permission, from Cunningham FG , Leveno KJ, Bloom SL, et al: Williams Obstetrics, 23rd ed. N ew York, M cGraw-Hill, 2010.

a. Frank breech b. Complete breech C.

Single footling breech

d. Double footling breech 2 4 - 6 . A fetus with hips flexed and knees flexed is best described as which o f the following?

a. Frank breech b. Complete breech C. Single footling breech

d. Double footling breech Reproduced, with permission, from Cunningham FG , Leveno K J, Bloom SL, et al: Williams Obstetrics, 23rd ed. N ew York, McGraw-Hill, 2010.

2 4 -7 . Which breech presentation has the lowest risk o f cord prolapse?

a. Frank breech

a. Frank breech

b. Footling breech

b. Footling breech

C.

Complete breech

d. Incomplete breech

C. Complete breech d. Double fooding breech

Breech Presentation and Delivery 2 4 -8 . The best and most readily available way to confirm a suspected breech presentation is with which o f the following?

a. Sonography

2 4 - 1 4 . Which o f the following best describes a breech fetus delivered spontaneously as far as the umbilicus, but whose remaining body is delivered with operator trac tion?

b. Vaginal examination

a. Mauriceau maneuver

C. Leopold maneuvers

b. Total breech extraction

d. Computed tomography

C.

Partial breech extraction

d. Spontaneous breech delivery 2 4 -9 . What percent o f breech presenting term fetuses has congenital anomalies?

a. 1% b. 6% c. 10% d. 16% 2 4 - 1 0 . Which fetal injury is not associated with vaginal breech delivery?

a. Fracture o f clavicle

2 4 - 1 5 . Umbilical cord abnormalities seen in breech fetuses include which o f the following?

a. Short cord b. Straight cord C.

Hypercoiling

d. Lack o f Wharton jelly 2 4 - 1 6 . This figure shows the operator’s hands in proper position during which o f the following?

b. Fracture ofhumerus C. Brachial plexus injury

d. Congenital hip dislocation 2 4 - 1 1 . Based on the review by Cheng and Hannah, what is a correct statement regarding the overall risk o f perina tal injury and death in term breech fetuses delivered vaginally?

a . It is applicable to preterm breeches. b. It is the same as that for breeches delivered by cesarean. C. It is increased fourfold over that of fetuses delivered by cesarean.

d . It is applicable to delivery of an aftercoming breech presenting twin. 2 4 - 1 2 . Which o f the following is accurate regarding preterm vaginal breech deliveries?

a . They ate as risky as term breech deliveries. b. The risk o f death is greater than with delivery by planned cesarean. C. The risk o f intraventricular hemorrhage is greater than delivery by planned cesarean,

d . The best way to deliver a preterm breech fetus has not been evaluated in any randomized clinical trials. 2 4 - 1 3 . Contraindications to breech vaginal delivery include which o f the following?

a. Frank breech b. Complete breech C. Hyperextended head

d . Aftercoming breech presenting twin

Reproduced, with permission, from Cunningham FG , Leveno KJ, Bloom SL, et al: Williams Obstetrics, 23rd ed. N ew York, McGraw-Hill, 2010.

a. Pinardmaneuver b. Partial breech extraction C.

Release o f a nuchal arm

d . Zavanelli maneuver

134

Labor and Delivery 2 4 -1 7 . Which o f the following is accurate regarding the Pinard maneuver, shown here?

2 4 - 1 9 . What is the name o f the procedure shown here, used to resolve head entrapment?

Reproduced, with permission, from Cunningham F G , Leveno K J, Bloom SL, etal: Williams Obstetrics, 23rd ed. N ew York, McGraw-Hill, 2010.

a . It accomplishes fetal decomposition.

b. It replaces the fetal feet into the uterus. C. It is easier to perform through an intact amnionic sac. d . It helps convert a footling breech into a frank breech. 2 4 - 1 8 . Which o f the following may be necessary to deliver the aftercoming head if the fetal trunk fails to rotate anteriorly?

a. Piper forceps b. Laufe forceps C.

Prague maneuver

d. Mauriceau maneuver

Reproduced, with permission, from Cunningham FG , Leveno KJ, Bloom SL, et al: Williams Obstetrics, 23rd ed. New York, McGraw-Hill, 2010.

a. Symphysiotomy b. Pinard maneuver C.

Diihrssen incisions

d. Zavanelli maneuver 2 4 - 2 0 . Which o f the following is true o f successful external cephalic version?

a . It is linked to increasing parity. b. It is aided by an anterior fetal spine. C. It occurs in over 80% o f version attempts,

d . It normalizes the risk o f cesarean delivery to that of vertex presentations. 2 4 - 2 1 . Absolute contraindications to external cephalic version include which o f the following?

a. Obesity b. Placenta previa C.

Anterior placenta

d. Prior uterine incision 2 4 - 2 2 . For breech decomposition, the most adequate anesthe sia is likely to be provided by which of the following?

a. Spinal anesthesia b. General anesthesia C.

Epidural anesthesia

d. Pudendal anesthesia

Breech Presentation and Delivery 2 4 - 2 3 . Which o f the following is the only tocolytic agent shown in a randomized trial to increase the success rate o f external cephalic version? a. Ritodrine

2 4 -2 5 . Which o f the following risks are associated to external cephalic version? a . Uterine rupture

b. Placental abruption

b. Terbutaline

C. Fetomaternal hemorrhage

C. Nitroglycerin

d. All o f the above

d. Magnesium sulfate

2 4 -2 4 . Internal podalic version is usually reserved for which clinical settings? a . Frank breech deliveries

b. Complete breech deliveries C. Delivery o f an aftercoming twin d. Preterm breech deliveries, regardless o f presentation

136

Labor and Delivery

Chapter 2 4 Answer Key Q u e s tio n num ber

L e tte r answ er

Page c ite d

H e a d e r c ite d

In tro d u c tio n

2 4 -1 5

a

p. 532

Fetal M o n ito rin g

p. 5 2 7

A sso cia te d Factors

2 4 -1 6

b

p. 533

Figure 2 4 -6

c

p. 528

Figure 24-1

2 4 -1 7

a

p. 535

Figure 24 -1 0

2 4 -4

a

p. 528

Figure 24-2

2 4 -1 8

c

p. 5 3 7

2 4 -5

c

p. 529

Figure 2 4 -4

M o d ifie d P rague M aneuver

2 4 -6

b

p. 527

D e fin itio n s

2 4 -1 9

c

p. 538

2 4 -7

a

p. 532

Fetal M o n ito rin g

E n tra p m e n t o f th e A fte rc o m in g H e a d / Figure 2 4 -1 7

2 4 -8

a

p. 529

Im a g in g T e c h n iq u e s / S on o g ra p h y

2 4 -2 0

a

p. 540

Factors A sso cia te d w ith Successful V ersion

2 4 -9

b

p. 529

P e rin a ta l M o rb id ity and M o rta lity

2 4-2 1

b

p. 540

In d ica tio n s

2 4 -2 2

b

p. 538

2 4 -1 0

d

p. 530

Fetal In ju rie s

A n a lg e sia and A n e s th e s ia

2 4 -1 1

c

p. 530

Term Breech Fetus

2 4 -2 3

b

p. 541

Tocolysis

2 4 -1 2

d

p. 530

P re te rm B reech Fetus

2 4 -2 4

c

p. 542

In te rn a l P odalic Version

2 4 -1 3

c

p. 531

R e co m m e n d a tio n s fo r D e liv e ry

2 4 -2 5

d

p. 542

C o m plica tion s

2 4 -1 4

c

p. 532

M e th o d s o f V aginal D e liv e ry

L e tte r answ er

Page c ite d

H e a d e r c ite d

2 4-1

b

p. 527

2 4 -2

a

2 4 -3

Q u e s tio n num ber

Cesarean Delivery and Peripartum Hysterectomy 2 5 - 1 . Some have proposed that the origin of the word cesarean arose after Julius Caesar was born in this manner. Why is this explanation unlikely?

2 5 -6 . What is the most frequent indication for cesarean delivery in the United States?

a. Dystocia

a . Antiseptic technique was not yet available.

b. Malpresentation

b. Abdominal deliveries had been described well be fore this time.

C.

C. Julius Caesar was born preterm. d . His mother survived for many years after his birth. 2 5 -2 . The words caesarean and section are both derived from Latin verbs that mean what?

a. To cut

Nonreassuring fetal status

d. Cord prolapse 2 5 -7 . Increasing use of electronic fetal monitoring has been associated with a decrease in which adverse perinatal outcome?

a. Death b. Neonatal seizures

b. To heal

C.

C. To multiply

d. None of the above

Cerebral palsy

d . To assuage

2 5 -8 . Practical strategies aimed at reducing the cesarean de 2 5 -3 . The cesarean delivery rate has steadily increased over the past 30 years with the exception o f what epoch during which the vaginal birth after cesarean (VBAC) rate was increasing commensurate with a decreasing cesarean rate?

a. 1970-1978 b. 1980-1988

livery rate include all EX C E P T which o f the following?

a. Peer review of cases b . Increasing vaginal breech deliveries C. Encouraging vaginal birth after cesarean delivery

d. Applying strict criteria to the diagnosis of dystocia 2 5 -9 . Compared with vaginal delivery, the maternal risks o f

c. 1989-1996

cesarean delivery include which of the following?

d. 2000-2008

a. Increased morbidity and mortality rates

2 5 - 4 . Reasons for increasing use o f cesarean delivery include which of the following?

a. An increasing percentage o f births to multiparas b. Declining average maternal age C. Widespread use of electronic fetal monitoring d. Higher rates of labor induction in women with preeclampsia 2 5 -5 . Elective cesarean deliveries are increasingly being performed for what indication?

a. Prevention ofpelvic floor injury

b. Increased morbidity but equivalent mortality rates C. Increased morbidity but decreased mortality rates

d. Equivalent morbidity and mortality rates 2 5 -1 0 . Although controversial, cesarean delivery on maternal request should only be considered as an option when which o f the following criteria have been met? a. The patient is concerned about inadequate pain control in labor.

b. The pregnancy has reached at least 39 weeks’ gestation.

b. Medically indicated preterm birth

C. The mother plans to have several subsequent pregnancies,

C. Maternal request

d. There is a history o f cerebral palsy in a previous

d. All of the above

child.

138

Labor and Delivery 2 5 - 1 1 . During Pfannenstiel incision, which vessels should be anticipated halfway between the skin and fascia, several centimeters from the midline? a. Inferior epigastric

2 5 - 1 5 . In deciding where to make the incision shown in this figure, when is it imperative to incise higher on the uterus to avoid laceration o f the uterine vessels or unintended entry into the vagina?

b. External pudendal C.

Superficial epigastric

d. Superficial circumflex iliac 2 5 - 1 2 . W hat benefit does Pfannenstiel incision offer over a midline incision? « 5g

a. Improved cosmetic result

b. Less postoperative pain C. Decreased rates o f incisional hernia

d. All o f the above 2 5 - 1 3 . Transverse uterine incisions are generally preferred to vertical incisions for all > P < > H L which of the follow ing reasons? a . Decreased risk of rupture in subsequent pregnan cies

b. Lower rates o f postpartum metritis C.

Ease of closure

d . Less likely to result in incisional adhesions to bowel 2 5 - 1 4 . The separadon o f the bladder from the lower uterine segment, as shown in the figure below, should not ex ceed what distance? a. 1 cm

Obstetrics, 23rd ed. New York, McGraw-Hill, 2010.

a . When the cervix is completely dilated

b. When the fetus is in breech presentation

b. 3 cm C.

Reproduced, with permission, from Cunningham FG, Leveno K J, Bloom SL, et al: Williams

C.

5 cm

When the mother is anemic

d. When the cesarean is done prior to the onset of

d. 7 cm

labor

N g ^X d i h f^c g fd g V

; aV Y Y f D d k f i h f^c g \b ch

Reproduced, with permission, from Cunningham FG , Leveno KJ, Bloom SL, et al: Williams Obstetrics, 23rd ed. New York, McGraw-Hill, 2010.

Cesarean Delivery and Peripartum Hysterectomy

D d k f i h f^c g \b ch M h f^c

^c X ^g ^d c

H aV X c hV

Reproduced, with permission, from Cunningham F G , Leveno KJ, Bloom SL , et al: Williams Obstetrics, 23rd ed. New York, McGraw-Hill, 2010.

2 5 - 1 6 . Spontaneous delivery o f the placenta with fundal massage, as shown in the figure above, compared with manual extraction has been shown to reduce the risk of what complication?

a. Deep venous thrombosis b. Amnionic fluid embolism C.

Postpartum infection

2 5 - 2 0 . All > P < > H L which o f the following would be con sidered potential indications for a classical (vertical) hysterotomy? a . Densely adherent bladder

b. Cervical cancer C.

“Back-up” transverse fetal lie

d. Significant maternal obesity

d. Retained placenta 2 5 - 1 7 . Which o f the following is a disadvantage to exterioriza tion o f the uterus to repair the hysterotomy?

2 5 - 2 1 . What is the most common complication in women who undergo peripartum hysterectomy? a. Urinary tract injury

a . Increased blood loss

b. Infection

b. Increased postoperative infection

C.

C.

Increased operative injury

d. Increased nausea and vomiting 2 5 - 1 8 . What is a potential advantage o f parietal peritoneum closure prior to fascial closure?

a. Decreased postoperative pain b. Avoidance o f distended bowel C.

Less adhesion formation

d. Shorter operative times 2 5 - 1 9 . Subcutaneous tissue greater than what depth should be closed with suture to avoid wound disruption? a. 2 cm

b. 4 cm C. 6 cm d . 1 0 cm

Blood transfusion

d. Venous thromboembolism

139

140

Labor and Delivery 2 5 - 2 2 . When performing the step shown in the image as a part o f peripartum hysterectomy, particular care must be taken to avoid injury to what structure?

SECTION 4

2 5 - 2 3 . When cystotomy complicates cesarean delivery, the bladder should be closed with a two- or three-layer running closure. Which layer is being closed in the image shown?

Mh f^c ^cX^g^dc ; aV Y Y f

iviS.- Z r -

C. Pronounced vascularity

d . Insertion into the uterine cornua

3 3-8 . With the Parkland method, to allow adequate separa



S it

identified by which o f its following attributes?

iU

$

'

:

0

tion of tubal stumps, it is recommended that a section o f approximately which length be excised?

; +

3. 0.5 cm

r*

’It "

* -

*

_

b . 1 cm C. 2 cm d . 4 cm Reproduced, with permission, from Cunningham FG , Leveno KJ, Bloom SL, et al: Williams Obstetrics, 23rd ed. N ew York, McGraw-Hill, 2010.

3. Appendix b. Fallopian tube C.

Round ligament

d . Utero-ovarian ligament

1

Puerperium 3 3 - 1 0 . Puerperal sterilization failures result most commonly from which o f the following?

3 3 - 1 4 . Cause for interval tubal ligation failure includes which o f the following?

3. Ligation o f incorrect structure

a. Fistula formation

b. Postoperative acute salpingitis

b. Intercourse too soon after the procedure

C.

Use o f absorbable suture for ligation

d. Intercourse too soon following ligation 3 3 - 1 1 . In the United States, interval tubal sterilization is most commonly completed by which o f the following meth ods?

a. Colpotomy

3 3 - 1 5 . Amenorrhea following tubal sterilization should prompt which o f the following? a . Basal body temperature charting C. (3-Human chorionic gonadotropin testing

Hysteroscopy

d. Progesterone administration and withdrawal

d. Minilaparotomy 3 3 - 1 2 . O f the following interval sterilization methods, which is least commonly employed in the United States? a . Laparoscopic clip placement

b. Distal fimbriectomy via colpotomy C. Laparoscopic electrosurgical coagulation d . Hysteroscopic placement of tubal occlusive inserts 3 3 - 1 3 . With this method ofinterval sterilization shown below, which o f the following energy sources is preferred? a. Harmonic ultrasound

b. Bipolar electrocoagulation C.

d. All o f the above

b. Intramuscular methotrexate administration

b. Laparoscopy C.

C. Surgery scheduled in the follicular phase

Unipolar electrocoagulation

d. Nd:YAG (neodymium-dopedyttrium aluminum

3 3 - 1 6 . Although an effective method o f contraception, preg nancy in a patient following tubal sterilization is often complicated by which o f the following?

a. Twinning b. Placenta previa C.

Placenta accreta

d. Ectopic implantation 3 3 - 1 7 . Studies support that tubal ligation is associated with which o f the following postprocedural side effects?

a. Regret b. Menorrhagia C.

Breast cancer

d. Decreased libido

garnet) laser

Reproduced with permission from Schorge JO , Schaffer JI, Halvorson LM , et al (eds). Williams Gyne cology. N ew York, McGraw-Hill, 2008, Figure 41-29.1.

Sterilization 3 3 - 1 8 . This device achieves sterilizadon by which ofthe following methods?

187

3 3 - 2 1 . During vasectomy, which o f the following structures is ligated? a. Epididymis C. Ductus deferens

d. Efferent ductule 3 3 - 2 2 . Compared with vasectomy, which ofthe following is higher with female tubal sterilization? a. Cost

b. Failure rate C.

Surgical complication rate

d . All o f the above

a . It lies within the cervical canal to secrete spermi cide.

b. It wraps around the fallopian tube to occlude the lumen. C. It is placed within the tubal ostia to promote occlu sive tissue ingrowth, d. It is placed within the endometrial canal to aggluti nate the endometrium. 3 3 - 1 9 . With the Essure method, which o f the following imag ing procedures is recommended to document steriliza tion following surgery?

3 3 - 2 3 . To avoid conception following vasectomy, an alterna tive form o f contraception should be used until semen analysis documents aspermia. Complete sperm expul sion from the reproductive tract takes approximately how long?

a.

1 week

b. 4 weeks C. 8

d.

weeks

12

weeks

3 3 - 2 4 . Vasectomy failures may result from which o f the following? a. Recanalization

a. K U B radiograph

b. Incomplete surgical occlusion

b. Hysterosalpingography

C. Intercourse too soon after the procedure

C. Magnetic resonance imaging

d. All o f the above

d. 3-D transvaginal sonography 3 3 - 2 0 . With the Essure method, an imaging procedure to document sterilization should be performed how long after surgery?

3 3 - 2 5 . A long-term complication following vasectomy includes which o f the following?

a. Regret b. Atherogenesis

a . 1 week

C. Testicular cancer

b. 4 weeks

d. All o f the above

C. 8

d.

weeks

1 2 weeks

CHAPTER 33

b. Spermatic cord

Puerperium

Chapter 33 Answer Key L e tte r answ er

Page c ite d

H e a d e r c ite d

Q u e s tio n num ber

L e tte r answ er

Page c ite d

H e a d e r c ite d

3 3-1

b

p. 699

Failure Rates

3 3 -1 4

a

p. 700

Failure Rates

3 3 -2

a

p. 699

Failure Rates

3 3 -1 5

c

p. 701

Ectopic P reg nancy

3 3 -3

c

p. 698

P ue rp e ra l Tubal S te riliz a tio n

3 3 -1 6

d

p. 701

Ectopic P regna ncy

3 3 -4

a

p. 698

P uerperal Tubal S te riliz a tio n

3 3 -1 7

a

p. 701

O th e r Effects

3 3 -5

c

p. 7 0 0

Figure 33-2

3 3 -1 8

c

p. 702

In tra tu b a l D evices

3 3 -6

d

p. 699

Figure 33-1

3 3 -1 9

b

p. 702

In tra tu b a l D evices

3 3 -7

a

p. 698

S urgical T e ch n iq u e

3 3 -2 0

d

p. 702

In tra tu b a l D evices

3 3 -8

c

p. 699

Figure 33-1

3 3-2 1

c

p. 702

M a le S te riliz a tio n

3 3 -9

b

p. 699

Failure Rates

3 3 -2 2

d

p. 703

M a le S te riliz a tio n

3 3 -1 0

a

p. 699

Failure Rates

3 3 -2 3

d

p. 703

M a le S te riliz a tio n

3 3 -1 1

b

p. 700

S urgical A p p ro a ch e s

3 3 -2 4

d

p. 703

M a le S te riliz a tio n

3 3 -1 2

b

p. 700

S urgical A pp ro a ch e s

3 3 -2 5

a

p. 703

L o n g -te rm Effects

3 3 -1 3

b

p. 700

Laparoscopic M e th o d s o f Tubal In te rru p tio n

Q u e s tio n num ber

SECTION 7

OBSTETRICAL COMPLICATIONS

CHAPTER 34

Pregnancy Hypertension 3 4 -1 . Which o f the following is accurate regarding the diag nosis o f gestational hypertension? a . It is appropriate if patients demonstrate only 1+ protein on urine dipstick.

b. It can be made only if hypertension resolves by 1 2 weeks postpartum.

3 4 -3 . What is the term for placental vasculature changes shown here, which include endothelial damage and subsequent lipid accumulation in myointimal cells?

: f V d[ c Y d h Y ^g fi e h^d c

a^V a

C. It is made if the systolic blood pressure increases by 30 mm H g over the patient’s baseline blood pres sure. d. All o f the above. ^W f^c *e aV g b V efdh ^c

3 4 - 2 . The figure given here demonstrates which o f the fol lowing proposed etiologies for preeclampsia? )(: cX df^c\ j^aa^

> cYdh mc X^h^dhfd e dWaVgh

a^V a X aag

D ^e ^Y (aV Y c b V X fd e V \ g

< mhdhfd e dWaVgh Modified, with permission, from BB Rogers, S L Bloom, and KJ Leveno: Atherosis revisited: Current concepts on the pathophysiology o f implantation site, Obstet Gynecol Survey > lhfVj^aadig hfde dWaVghg

19 9 9 ;5 4(3):189-195.

Bch fgh^h^Va lhfV j^aadig hfde dWaVgh

a . Atherosis

b. Vasospasm C. Angiogenesis

lhfVj^aadig hfde dWaVgh

NORM AL

HJ > > < D: E H B:

Reproduced, with permission, from Cunningham FG , Leveno K J, Bloom SL, et al: Williams Obstetrics, 23rd ed. N ew York, McGraw-Hill, 2010.

a . Increased branching o f the chorion frondosum

b. Incomplete trophoblastic invasion o f uterine arteri oles C. Development o f blocking antibodies to placental antigenic sites d . Increased activity o f decidual natural killer cells leading to reduced placental vascularity

d. Endotheliosis 3 4 - 4 . What is the risk that a daughter o f a woman who had preeclampsia will develop this complication herself?

a. 5% b. 10% c. 65% d. 20-40%

Pregnancy Hypertension 3 4 -5 . What is the incidence o f gestational hypertension in this type o f pregnancy?

3 4 - 8 . Disturbances in cardiovascular function related to preeclampsia include all E X C E P T which o f the follow ing?

b. Increased cardiac afterload C.

Decreased left ventricular mass

d. Extravasation o f intravascular fluid into extracellu lar space 3 4 -9 . Preeclamptic patients, with hemodynamic function shown in the hatched area o f part A, are more likely to develop which of the following complications? 120 -i

Hyperdynam ic

7 E E b)

b. 25% c. 33% d. 0.5%

Depressed

3 4 -6 . Risk factors for the development o f preeclampsia in clude all E X C E P T which o f the following?

a. Obesity

A

b. Nulliparity C.

h-j

_l------ —

------------r ------- 1

10

20

15

25

30

PCW P (mm Hg)

0

B

5

10

15

20

25

30

PCW P (mm Hg)

Reproduced, with permission, from Cunningham FG , Leveno KJ, Bloom SL, et al: Williams

Hispanic ethnicity

Obstetrics, 23rd ed. New York, McGraw-Hill, 2010.

d. Maternal age > 3 5 years a. Pulmonary edema 3 4 -7 . Soluble fms-like tyrosine kinase 1 (sFlt-1) causes in activation o f placental growth factor and leads to en dothelial dysfunction. In the graph here, the red line represents which group o f pregnant women?

b. Myocardial infarction C.

Acute tubular necrosis

d. Supraventricular tachycardia 3 4 - 1 0 . Which ofthe following is true of thrombocytopenia secondary to preeclampsia? a . It is an indication for cesarean route o f delivery.

b. It is an indication for delivery if levels drop below 140,000/jxL. C. It is frequently accompanied by fetal thrombocy topenia.

d. It may not reach a nadir until 48-72 hours after delivery.

ghVh^dcVa V\

k

Reproduced, wida permission, from Cunningham FG , Leveno K J, Bloom SL, et al: Williams Obstetrics, 23rd ed. New York, McGraw-Hill, 2010.

3 4 - 1 1 . Clinical evidence o f hemolysis includes which of the following?

a. Decrease in hematocrit b. Presence ofspherocytes in peripheral blood

a. Women who develop preeclampsia

C. Elevation o f serum lactate dehydrogenase level

b. Women whose fetuses are growth restricted

d. All o f the above

C.

Women who are normotensive during pregnancy

d. Women who ultimately suffer placental abruption

CHAPTER34

a . Reduced cardiac preload

a. 13%

191

192

Obstetrical Complications 3 4 - 1 2 . What is the mortality rate when this complication occurs during pregnancy?

3 4 - 1 5 . This image demonstrates evidence o f which o f the following complications that may be present in preeclampsia?

SECTION 7 From H ay JE , Liver disease in pregnancy. Hepatology, vol. 47, no. 3, pp. 1067—1076. © 2008 American Association for the Study o f Liver Diseases. Reproduced, with permission, o f John W iley &£ Sons, Inc.

a. 5% b. 10% c. 30% d. 50%

Reproduced, with permission, from Knoop KJ, Stack LB, Storrow AB, et al: Atlas o f Emer gency Medicine, 3rd ed. New York, McGraw-Hill, 2010:849. Photo contributor: Jam es P. Elrod, M D , PhD.

a. Hemolysis b. Leukocytosis C. Thrombocytopenia

3 4 - 1 3 . Which of the following laboratory studies may aid in the differentiation between severe preeclampsia and acute fatty liver? a . Glucose

b. Creatinine

d. Iron-deficiency anemia 3 4 - 1 6 . Which o f the following therapies is paramount in treat ing the complication shown in this computed tomog raphy scan when it occurs during pregnancy?

C. Platelet count

d. Transaminases 3 4 - 1 4 . Which ofthe following is accurate regarding the com plaints o f headache and scotomata in a patient with preeclampsia? 3. These are secondary to reduced cerebral perfusion,

b. These are associated with a significant risk o f blind ness. C. These are concerning as they frequently precede eclamptic seizures,

d. None o f the above.

Reproduced, with permission, from Doherty G M : Current Diagnosis & Treatment: Surgery, 13th ed. N ew York, McGraw-Hill, 2010, Figure 36-7.

a. Prophylactic intubation b. Immediate ventriculostomy C. Antihypertensive therapy to normalize blood pres sure d . Platelet transfusions to keep platelet count >8 0 ,0 0 0/|x L

Pregnancy Hypertension 3 4 - 1 7 . Antihypertensive therapy for early mild gestational hypertension is associated with which of the following?

a. Increased birthweights

3 4 -2 2 . In women with preeclampsia, causes for the fetal heart rate pattern seen here include all E X C E P T which of the following?

CHAPTER 34

b. Pregnancy prolongation C.

Lower mean blood pressure

d . Reduced rates o f fetal-growth restriction 3 4 - 1 8 . Initial reports o f expectant management of severe preeclampsia showed which o f the following?

a. Low risk o f eclampsia b. Increased maternal death rate C.

Perinatal mortality rates approached 90%

d. A l-in-10 risk for placental abruption 3 4 - 1 9 . In the setting o f severe early onset preeclampsia, de layed delivery allows for an average pregnancy prolon gation o f what length?

a. 48 hours b. 5-10 days C.

20 days

d. 32 days 3 4 -2 0 . A pregnant patient who has received a 6 g loading dose o f magnesium sulfate and who is currently on a 2.5 g/h infusion has a seizure. Which o f the following medica tions is appropriate to use to control the seizure? a . Midazolam

b. Amobarbital C.

Magnesium sulfate

d. All o f the above

Reproduced, with permission, from Cunningham FG , Leveno K J, Bloom SL, et al: Williams Obstetrics, 23rd ed. New York, McGraw-Hill, 2010.

a. Cord compression b. Placental abruption C.

Fetal-growth restriction

d . Maternal hypotension following hydralazine administration 3 4 - 2 3 . Appropriate antihypertensive therapy in the intra partum period include all E X C E P T which o f the fol lowing?

a. Labetalol 3 4 - 2 1 . Mild-to-moderate respiratory depression that accom panies magnesium sulfate toxicity is treated with which o f the following intravenous medications?

a.

0 . 5 mg atropine

b.

1 mg epinephrine

C.

20 mg furosemide

d.

1 g calcium gluconate

193

b. Nifedipine C.

Furosemide

d. Nitroprusside 3 4 - 2 4 . Which ofthe following statements accurately describes the use o f magnesium sulfate in preeclampsia? a. Phenytoin is equivalent to magnesium sulfate in the prevention o f eclampsia.

b. Only women who have had an eclamptic seizure should receive this medication. C.

Women with “nonsevere” disease should receive magnesium sulfate during labor.

d. Magnesium sulfate reduces the risk o f eclampsia by 58% in women with severe disease.

194

Obstetrical Complications 3 4 - 2 5 . In preeclamptic patients, which o f the following ad verse effects is seen with epidural analgesia during labor compared with intravenous analgesia?

3 4 -2 7 . For women diagnosed with preeclampsia before 30 weeks’ gestation in their first pregnancy, what is the recurrence risk in a subsequent pregnancy?

SECTION 7

a. Decreased pain relief

a. 4 -6 %

b. Increased risk o f pulmonary edema

b. 10%

C. Increased frequency o f hypotension

c. 40-60%

d. Increased risk o f cesarean delivery for dystocia

d. 90%

3 4 - 2 6 . Severe postpartum hypertension can be effectively treated with which of the following? 3. Diuretics

b. |3-Blockers C. Calcium-channel blockers

d. All of the above

Pregnancy Hypertension

Chapter 34 Answer Key Q u e s tio n num ber

L e tte r answ er

Page c ite d

3 4 -1 7

c

p. 731

A n tih y p e rte n s iv e Therapy fo r M ild to M o d e ra te H yp e rte n s io n

3 4 -1 8

c

p. 732

D elayed D e liv e ry w ith Early O nset Severe P reeclam psia

3 4 -1 9

b

p. 732

D elayed D e liv e ry w ith Early O nset Severe P reeclam psia

3 4 -2 0

d

p. 737

M a g n e s iu m S ulfate to C ontrol C onvulsions

3 4 -2 1

d

p. 738

P h a rm a c o lo g y and Toxicology

T h ro m b o c yto p e n ia

3 4 -2 2

a

p. 740

H ydralazine

p. 718

H em o lysis

3 4 -2 3

c

p. 741

D iuretics

c

p. 721

Liver

3 4 -2 4

d

p. 742

P re v e n tio n o f Eclam psia

3 4 -1 3

a

p. 721

Liver

3 4 -2 5

c

p. 746

A na lg e sia and A n e s th e s ia

3 4 -1 4

c

p. 722

Clinical M a n ife s ta tio n s

3 4 -2 6

d

p. 746

3 4 -1 5

a

p. 718

H em o lysis

Severe P ost-P artum H yp e rte n s io n

3 4 -1 6

c

p. 724

Cerebral Edem a

3 4 -2 7

c

p. 747

C ounseling fo r Future P regnancy

L e tte r answ er

Page c ite d

H e a d e r c ite d

34-1

b

p. 707

Table 34-1

34-2

b

p. 711

Figure 34-2

3 4 -3

a

p. 710

A b n o rm a l T ro p h o b la stic Invasion

3 4 -4

d

p. 712

G enetic Factors

3 4 -5

a

p. 709

Incidence and Risk Factors

3 4 -6

c

p. 709

Incid e n ce and Risk Factors

3 4 -7

a

p. 714

A n g io g e n ic and A n tia n g io g e n ic P roteins

3 4 -8

c

p. 715

C ardiovascular S ystem

3 4 -9

a

p. 716

H e m o d y n a m ic Changes

3 4 -1 0

d

p. 717

3 4 -1 1

d

3 4 -1 2

Q u e s tio n num ber

H e a d e r c ite d

196

CHAPTER 35

Obstetrical Hemorrhage 3 5 -3 . Which o f the following is true regarding supracervical bleeding in the mid and third trimester?

3 5 - 1 . "Which o f the following is true of obstetrical hemor rhage?

a. Usually from vasa previa

a . Responsible for 17% o f pregnancy-related maternal deaths in the United States

b. Infrequently associated with adverse outcomes

b. Responsible for 12% o f maternal deaths in the U.S.

C. M ay involve separation o f the placenta

private sector

d. None o f the above

C. Single most important cause o f maternal death worldwide

3 5 -4 . Loss o f more than what blood volume after comple tion o f third-stage labor in vaginal deliveries defines postpartum hemorrhage?

d. All o f the above 3 5 - 2 . The figure below indicates that most maternal deaths are due to which o f the following?

a. 400 cc

b. 500 cc

a. Placental abruption

C. 600 cc

b. Uterine atony

d.

C. Coagulopathy

d. Placenta accreta

1 , 0 0 0 cc

3 5 -5 . What is the blood volume o fa 5 feet, 120lbw om an during pregnancy?

a. 3,000 cc b. 4,000 cc c. 4,500 cc d. 5,000 cc

J hV ^c Y e aV X c hV

M h f^c V hd c m

R E d fW ^Y ^hm

(n = , 1 , 0 V(W

B E d fhV a^hm

(n = 2 1 .

D V X fV h^d c ' fi e hi f < Vig

d[

H f j ^V ' V X X f hV ' e fX f hV

H aV X c hV a < d V \ i ad e V h m V W fi e h^d c

b d ff V \

Reproduced, with permission, from Cunningham F G , Leveno K J, Bloom SL, et al: W illiams Obstetrics, 23rd ed. New York, McGraw-Hill,

2010 .

Obstetrical Hemorrhage 3 5 - 6 . W hat is the amount o f blood at term that flows through the intervillous space?

3 5 - 1 0 . This image indicates which o f the following? a . Incidence o f previa decreases with age.

a. lOOcc/min

b. Incidence o f placental abruption is not age depen

b. 300 cc/min

dent. Incidence o f placental abruption decreases with age.

C. 600 cc/min

C.

d.

d . Incidence o f placental abruption increases with age.

1 , 2 0 0 cc/min

E V h fc V a V \

mf

e V h^ c hg

Reproduced, with permission, from Cunningham F G , Leveno KJ, Bloom SL, et al: Williams Obstetrics, 23 rd ed. N ew York, McGraw-Hill,

2010)

3 5 -7 . The mean increase in blood volume in women with preeclampsia is which of the following?

3 5 - 1 1 . What is the percentage o f women with chronic hyper tension who suffer placental abruption?

a. 5-10%

a. 1.5%

b. 10-29%

b. 10%

c. 30-3 7% d. 38-5 0%

C.

3 5 - 8 . W ith a firm, well-contracted uterus, postpartum brisk bleeding is most likely due to which o f the following?

25%

d. 50% 3 5 - 1 2 . A woman who has suffered previous placental abrup tion during pregnancy has what recurrence risk?

a. Lacerations

a. 5%

b. Retained placenta

b. 7%

C.

Coagulopathy

d . Lower uterine segment atony 3 5 - 9 . What is the average frequency o f placental abruption?

a. 1/50 b. 1/100 C.

1/150

d. 1/200

C.

12%

d. 15%

198

Obstetrical Complications 3 5 -1 3 . What is the approximate age o f the clot in this image?

3 5 -1 6 . This image demonstrates which o f the following?

SECTION 7 Reproduced, with permission, from Cunningham F G , Leveno K J, Bloom SL , et al: Williams Obscetrics, 23rd ed. N ew York, McGraw-Hill, 2010.

3. Fresh

b. Several hours old C. At least 24 hours old

d. Several days old 3 5 - 1 4 . W ith placental abruption severe enough to kill the fetus, what is the usual maternal blood volume lost?

Reproduced, with permission, from Cunningham FG , Leveno K J, Bloom SL , et al: Williams Obstetrics, 23rd ed. N ew York, McGraw-Hill, 2010.

3. Myometritis

b. Placenta accreta

3. 500 cc

C. Couvelaire uterus

b.

d. Uterine didelphys

1 , 0 0 0 cc

C. One-half o f pregnancy blood volume

d. None o f the above 3 5 - 1 5 . Placental abruption is linked to what percentage of pregnancy-associated acute renal failure? a . 10%

3 5 - 1 7 . This fetal monitor tracing is from a case o f placental abruption with fetal demise. Features include which of the following? a . Maternal EC G

b. Increased resting tone C. Tachysystole

b. 25%

d. All o f the above

c. 33%

d. 50%

Reproduced, with permission, from Cunningham FG , Leveno K J, Bloom SL , et al: W illiams Obstetrics, 23rd ed. New York, McGraw-Hill, 2010.

Obstetrical Hemorrhage 3 5 - 1 8 . This fetal monitor tracing demonstrates which o f the following?

199

3 5 - 2 0 . What is the incidence of placenta previa?

a. 1/10 b. 1/100 1/200

d. 1/300 3 5 - 2 1 . Risk factors for placenta previa include which o f the following? 3. Young age

b. Smoking Reproduced, with permission, from Cunningham FG , Leveno K J, Bloom SL, et al: Williams Obstetrics, 23rd ed. New York, M cGraw-Hill, 2010.

a. M aternalEC G

C.

Nulliparity

d. Low maternal serum a-fetoprotein levels 3 5 - 2 2 . This image illustrates which o f the following?

b. Increased resting tone C.

Less than 100 Montevideo units in 10 minutes

d. Repetitive variable decelerations 3 5 - 1 9 . Which o f the following is illustrated in this image o f an opened cesarean hysterectomy specimen?

Reproduced, with permission, from Cunningham FG , Leveno KJ, Bloom SL, et al: Williams Obstetrics, 23rd ed. New York, McGraw-Hill, 2010.

a. Transabdominal sonography b. Complete placenta previa C.

Partial placenta previa

d. Low lying placenta

a . Complete placenta previa

b. Partial placenta previa C. Marginal placenta previa

d. Low lying placenta

CHAPTER 35

C.

200

Obstetrical Complications 3 5 - 2 3 . This image illustrates which o f the following points regarding placenta previa?

3 5 - 2 7 . Which o f the following is an effective uterotonic agent used to treat uterine atony and postpartum hemor rhage? 3. Oxytocin

b. Carboprost C.

Misoprostol

d. All o f the above 3 5 -2 8 . This image illustrates which o f the following?

g hV h^d c V a V \ k

k

c g d c d \ fV e m e f[d fb Y

Reproduced, with permission, from Cunningham F G , Leveno K J, Bloom SL , etal: Williams Obstetrics, 23rd ed. N ew York, M cGraw-Hill, 2010.

a . If found early in pregnancy, the persistence rate increases. b. Rates increase with a history o f prior cesarean deliv ery. C. Diagnosed placenta previa has a higher persistence rate if there is a prior cesarean delivery,

Reproduced, with permission, from Cunningham F G , Leveno K J, Bloom SL, etal: Williams Obstetrics, 23rd ed. N ew York, McGraw-Hill, 2010.

d. None ofthe above. a . Transverse imbricating compression sutures 3 5 - 2 4 . Which of the following is true o f low birth weight asso ciated with placenta previa?

a . Most likely from fetal-growth restriction

b. B-lynch technique C.

Hysterotomy closure

d. None o f the above

b. Confined to third-trimester fetuses C.

Related to preterm birth

d. None o f the above

3 5 - 2 9 . In 2006, what was the reported incidence o f placenta accreta?

a. 1/2,500 3 5 - 2 5 . WTiich of the following is the most common cause of postpartum hemorrhage? a . Uterine atony

b. Lacerations C.

Retained placenta

d. Coagulopathy 3 5 - 2 6 . Which of the following are risk factors for uterine atony?

b. 1/535 c. 1/210 d. 1/125 3 5 - 3 0 . Which o f the following is diagnosed when placental villi attach abnormally to, but do not invade, the my ometrium?

a. Placentaincreta b. Placenta percreta

a. High parity

C. Placenta accreta

b. Overdistended uterus

d. None o f the above

C.

Vigorous uterine activity

d. All o f the above

Obstetrical Hemorrhage 3 5 - 3 1 . The photograph demonstrates which o f the following?

3 5 - 3 3 . With extensive placental involvement and hem orrhage, successful management o f placenta acc reta/ percreta usually includes which o f the following? b. Immediate blood replacement C. Full development of the bladder flap prior to hys terotomy

d. All of the above 3 5 - 3 4 . Uterine inversion occurs with what frequency?

a. 1/10,000 deliveries b. 1/5,000 deliveries C. 1/3,000 deliveries

d.

Obstetrics, 23rd ed. New York, McGraw-Hill, 2010.

1 / 1 , 0 0 0 deliveries

3 5-3 5 . Complete uterine inversion after delivery is almost always associated with which of the following?

a. Placenta accreta

a. Placenta accreta

b. Short umbilical cord

b. Placenta increta

C. Posterior placenta

C. Placenta percreta

d. Strong traction on the umbilical cord

d. None o f the above 3 5 - 3 2 . Using Doppler flow color mapping, which two factors are highly predictive of placenta accreta?

3 5 - 3 6 . Initial management o f uterine inversion includes which of the following?

a. Call for help b. Replace uterus C. Establish large bore IV fluid systems

d. Call for blood 3 5 - 3 7 . Concerning vaginal lacerations involving the middle or upper third o f the vagina, which o f the following is true?

a. These are often the result o f a forceps delivery. b. These result from uterine overdistension. C.

These are usually associated with injuries to the levator ani muscles.

d. All o f the above.

Reproduced, with permission, from Cunningham F G , Leveno KJ, Bloom SL, et al: Williams Obstetrics, 23rd ed. New York, McGraw-Hill, 2010.

a. < 1 mm between uterine serosa-bladder interface and retroplacental vessels; lateral placenta W) Large intraplacental lakes; fundal placenta C. < 1 mm between uterine serosa-bladder interface and retroplacental vessels; large placental lakes Y) None o f the above

CHAPTER 35

a. Hysterectomy

Reproduced, with permission, from Cunningham FG , Leveno K J, Bloom SL, et al: Williams

201

202

Obstetrical Complications 3 5 - 3 8 . This image illustrates which of the following?

3 5 - 4 3 . After fetal death and delayed delivery, which is true regarding significant disruption o f the maternal coagu lation system?

SECTION 7

a. Rarely develops within 2 weeks after fetal death b. Frequently develops within 1 month after fetal death C.

Develops in 40% o f women after 1 month follow ing fetal death

d. All o f the above 3 5 - 4 4 . This photomicrograph illustrates which ofthe follow ing?

Reproduced, with permission, from Cunningham F G , Leveno K J, Bloom SL, et al: Williams Obstetrics, 23rd ed. N ew York, McGraw-Hill, 2010.

a. Vulvar hematoma b. Vulvovaginal hematoma C. Paravaginal hematoma

d. Retroperitoneal hematoma 3 5 - 3 9 . Which of the following is the best treatment for vulvar hematomas that are extremely painful, but stable in size?

Reproduced, with permission, from Cunningham F G , Leveno K J, Bloom SL, et al: Williams Obstetrics, 23rd ed. N ew York, McGraw-Hill, 2010.

a. Analgesics b. Ice compress C. Incision and drainage

d. Angiographic embolization

a. Tuberculosis b. Pulmonary embolism C.

Fetal squames in small pulmonary artery

d. Neonatal pneumonia 3 5 - 4 0 . Which o f the following is the most common cause o f uterine rupture?

a. Traumatic injury b. Internal podalic version C. Separation o f a previous cesarean hysterotomy scar

d. Labor induction

3 5 - 4 5 . Serious disruption o f the coagulation mechanism as the consequence o f abortion may develop in which o f the following circumstances? a . Prolonged retention o f a dead fetus

b. Sepsis syndrome C. Labor induction with a prostaglandin

3 5 - 4 1 . Pregnancy induces appreciable increases in amounts of which of the following?

a . Factors VII and VIII b. Fibrinogen

d. All o f the above 3 5 - 4 6 . At least 70% o f total blood volume in pregnant women is contained in which o f the following?

C. Factors IX and X

a. Venules

d. All of the above

b. Large veins C.

3 5 - 4 2 . To promote clinical coagulation, fibrinogen levels must be maintained at approximately what level?

a. 50 mg/dL b. 100 mg/dL C. 150 mg/dL

d. 200 m g/dL

Arterioles

d. Capillary beds

Obstetrical Hemorrhage 3 5 - 4 7 . During hemorrhage, there is relative maintenance of blood flow to which organ?

3 5 - 5 4 . O f the following viral infections, which has the highest risk o f transmission with blood products?

a. Kidney

a. Human immunodeficiency virus-1

b. Adrenal gland

b. Hepatitis C

C. Uterus

C. Human immunodeficiency virus-2

d. Splanchnic beds

d. Hepatitis B

3 5 - 4 8 . Which o f the following is one o f the most important signs to follow in a woman with obstetrical hemor rhage?

3 5-5 5 . Which o f the following is used primarily for surgical management o f hemorrhage in lateral hysterotomy extensions?

a. Pulse

a. Uterine artery ligation

b. Blood pressure

b. Uterine compression sutures

C. Respiratory rate

C. Internal iliac artery ligation

d. Urine output

d. Hysterectomy

3 5 - 4 9 . For the woman acutely bleeding, at what hematocrit should rapid blood transfusion begin?

3 5 - 5 6 . The equipment in this image is used in which clinical situation?

a. 30% b. 27% c. 25%

d. 20% 3 5 - 5 0 . The use o f whole blood as opposed to packed red blood cells for replacement during obstetric hemorrhage is associated with which of the following?

a . Decreased respiratory distress syndrome b. Decreased renal failure C. Decreased pulmonary edema

d. All o f the above 3 5 - 5 1 . Each unit o f do nor platelets transfused should raise the platelet count by:

a. 15,000/uL b. 12,000/uL C. 5,000/uL

d. 3,000/uL 3 5 - 5 2 . Which o f the following correctly describes a unit o f cryoprecipitate?

a. It contains 200 mg of fibrinogen. b. It is available in 50 cc solutions.

Reproduced, with permission, from Cunningham FG , Leveno K J, Bloom SL, et al: Williams Obstetrics, 23rd ed. N ew York, McGraw-Hill, 2010.

C. It contains platelets as well as factor XIII.

d. All o f the above. 3 5 - 5 3 . "Which o f the following is true o f transfusion-related lung injury (TRALI)?

a. It usually develops within 3 hours. b. It involves massive coagulopathy. C. It is characterized by noncardiogenic pulmonary edema.

d. It complicates I in 2,000 transfusions.

a . It is used to pack the uterus to treat hemorrhage.

b. It is used to pack the vagina to treat hemorrhage. C. It is used to pack the pelvis to treat hemorrhage following hysterectomy,

d. None o f the above.

204

Obstetrical Complications

Chapter 35 Answer Key

SECTION 7

Q u e s tio n num ber

L e tte r answ er

Page c ite d

H e a d e r c ite d

In tro d u c tio n

3 5 -3 0

c

p. 776

D e fin itio n s

p. 758

Figure 35-1

3 5 -3 1

c

p. 778

Figure 35-20

c

p. 759

A n te p a rtu m H e m o rrh a g e

3 5 -3 2

c

p. 779

Figure 35-21

3 5 -4

b

p. 759

D e fin itio n

3 5 -3 3

d

p. 7 7 9

D e liv e ry o f th e P lacenta

3 5 -5

c

p. 761

Table 35-3

3 5 -3 4

c

p. 780

Inversion o f th e U terus

3 5 -6

c

p. 7 6 0

H e m ostasis at th e P lacental Site

3 5 -3 5

d

p. 780

Inversion o f th e U terus

3 5 -3 6

a

p. 781

M anagem ent

3 5 -7

b

p. 760

C linical C haracteristics

3 5 -3 7

a

p. 782

V aginal Lacerations

3 5 -8

a

p. 761

D iagnosis

3 5 -3 8

a

p. 783

3 5 -9

d

p. 761

S ig n ifica n ce and F re q u e n cy'

Figure 35-2 6 , P ue rp e ra l H e m a to m a s

3 5 -3 9

c

p. 783

T re a tm e n t

3 5 -1 0

d

p. 763

Figure 3 5 -6

3 5 -4 0

c

p. 784

R upture o f th e U terus

3 5 -1 1

a

p. 764

H yp e rte n s io n

3 5-41

d

p. 785

3 5 -1 2

c

p. 764

R e cu rre n t A b ru p tio n

P regnancy H y p e rc o a g u la b ility

3 5 -1 3

b

p. 763

Figure 3 5 -5

3 5 -4 2

c

p. 7 8 7

H y p o fib rin o g e n e m ia

3 5 -1 4

c

p. 766

Shock

3 5 -4 3

a

p. 787

3 5 -1 5

c

p. 766

Renal Failure

Feta! D eath and D elayed D e liv e ry

3 5 -1 6

c

p. 767

Figure 35-7

3 5 -4 4

c

p. 7 9 0

Figure 3 5 -5 0

3 5 -1 7

d

p. 768

Figure 3 5 -9

3 5 -4 5

d

p. 790

C oa gu lo p a th y

3 5 -1 8

b

p. 7 6 9

Figure 3 5 -1 0

3 5 -4 6

a

p. 791

H y p o v o le m ic S hock

3 5 -1 9

a

p. 769

Figure 3 5 -1 1 , P lacenta Previa

3 5 -4 7

b

p. 791

H y p o v o le m ic S hock

3 5 -4 8

d

p. 791

E stim a tio n o f B lo o d Loss

3 5 -4 9

c

p. 792

B lood R e p la ce m e n t

3 5 -5 0

d

p. 793

W h o le B lood and B lood C o m pone nts

3 5-51

c

p. 793

P la te le ts

3 5 -5 2

a

p. 794

C ry o p re cip ita te

3 5 -5 3

c

p. 794

Transfusion R elate d Lung In ju ry (TRALI)

3 5 -5 4

d

p. 795

Viral Transm ission

3 5 -5 5

a

p. 795

S urgical M a n a g e m e n t o f H e m o rrh a g e

3 5 -5 6

c

p. 7 9 7 - ■8

P elvic U m b re lla Pack, Figure 35-34

Q u e s tio n num ber

L e tte r answ er

P age c ite d

H e a d e r c ite d

3 5 -1

d

p. 7 5 7

3 5 -2

a

3 5 -3

3 5 -2 0

d

p. 770 ^In cid e n ce

35-2 1

b

p. 7 7 0

-A ssociated Factors

3 5 -2 2

c

p. 772

Figure 3 5 -1 4

3 5 -2 3

c

p. 773

Figure 35 -1 5

3 5 -2 4

c

p. 773

M a te rn a l a n d P erin a ta l O u tco m e s

3 5 -2 5

a

p. 774

U te rin e A to n y

3 5 -2 6

d

p. 774

U te rin e A to n y

3 5 -2 7

d

p. 775

U te ro to n ic A ge n ts

3 5 -2 8

b

p. 7 7 7

Figure 35 -1 8

3 5 -2 9

c

p. 7 7 7

In cid e n ce

205

CHAPTER 36

Preterm Birth 3 6 - 1 . The term appropriatefor gestational age designates new borns whose weights are between what percentiles?

a. 2.5% and 97.5% b. 5% and 95%

3 6 - 6 . Much o f the increase in the singleton preterm birth rate in the United States is explained by increases in which o f the following?

a. Spontaneous preterm labor

C. 10% and 90%

b. Premature rupture of fetal membranes

d. 15% and 85%

C. Pelvic infection rates d. Indicated (iatrogenic) preterm births

3 6 -2 . Late preterm births, defined as those between 34 and 36 weeks’ gestation, compose what percentage o f all preterm births?

a. 35%

3 6 - 7 . All E X C E P T which o f the following lifestyle factors have been associated with spontaneous preterm labor and preterm birth?

b. 50%

a. Obesity

c. 70%

b. Inadequate maternal weight gain

d. 85%

C. Poverty d. Cigarette smoking

3 6 -3 . After achieving a birthweight of at least 1,000 g, neona tal survival rates reach 95% at approximately what gestational age with regard to infant sex?

a. 28 weeks for females and 30 weeks for males

3 6 - 8 . In the United States, women classified in what racial group are consistently reported to be at higher risk o f preterm birth?

b. 30 weeks for both males and females

a. Hispanic

C. 30 weeks for females and 28 weeks for males

b. White (non-Hispanic)

d . 28 weeks for both males and females

C. African American d. American Indian

3 6 - 4 . According to the published data from the National Institute o f Child Health and Human Development Neonatal Research Network, what is the single most prevalent complication that will be recognized at the time o f hospital discharge in very-low-birth-weight neonates?

a. Severe intraventricular hemorrhage b. Necrotizing enterocolitis C. Hydrocephalus

d. Bronchopulmonary dysplasia 3 6 - 5 . Compared with newborns at term, the risks to new borns between 34 and 36 weeks’ gestation include which o f the following?

a . Increased serious morbidity but equivalent mortality rates

b. Increased serious morbidity and mortality rates C. Increased serious morbidity but decreased mortality rates

d. Equivalent serious morbidity and mortality rates

3 6 - 9 . Intervals shorter than how many months between pregnancies have been associated with an increased risk for preterm birth?

a. 18 months b. 24 months C. 36 months d. 48 months 3 6 - 1 0 . For women who have one prior spontaneous preterm delivery at or before 34 weeks, what is the risk o f recur rence?

a. 5% b. 9% c. 16% d. 25%

206

Obstetrical Complications 3 6 - 1 1 . Antimicrobial treatment to prevent preterm labor due to microbial invasion has been demonstrated to have what effect? 3. Reduced rate o f spontaneous preterm birth

b. Reduced rate o f indicated preterm birth for chorioamnionitis C.

Reduced rate o f premature rupture of membranes

3 6 - 1 5 . When measured by transvaginal sonography, ashortened cervix has been associatedwith an increased rate of spontaneous preterm birth. This measurement is demonstrated in the ultrasound image as the distance between the two arrows (AF = amnionic fluid). In a population o f normal women who are not at increased risk for preterm birth, what is the approximate mean cervical length at 24 weeks’ gestation?

d . N o change in the rate o f preterm birth 3 6 - 1 2 . A 22-year-old G 2 P 1 at 14 weeks’ gestation complains o f a malodorous vaginal discharge. A wet mount is per formed with the findings illustrated in the image. You recommend antimicrobial treatment for this condition for what principal reason?

Reproduced, with permission, from Cunningham F G , Leveno K J, Bloom SL, et al: Williams Obstetrics, 23rd ed. N ew York, McGraw-Hill, 2010.

a. 25 mm b. 30 mm c. 35 mm

Reproduced, with permission, from Fauci A S, Braunwald E, Kasper D L: Harrison’s Princi

d. 40 mm

ples o f Internal Medicine, 17th ed. N ew York, M cGraw-Hill, 2008, Figure 124-3.

a. Prevention o f preterm birth b. Resolution o f symptoms C.

Avoidance o f spontaneous abortion

d . Treatment o f in tra-amnionic infection 3 6 - 1 3 . Characteristics o f Braxton Hicks contractions can include all E X C E P T which o f the following?

a. Irregular pattern b. Painful C.

Cervical dilatation

d. Nonrhythmical 3 6 - 1 4 . Which o f the following statements is true regarding the relationship between parity and asymptomatic cervical dilatation after midpregnancy? a . Risk for preterm delivery is increased only in nulliparous women.

b. Risk for preterm delivery is increased in nulliparous and parous women. C. Risk for preterm delivery is increased only in parous women.

d . There is no increased risk for preterm delivery.

36-16

Recurrent midtrimester pregnancy loss in the setting of painless cervical dilatation is termed:

a. Cervical incompetence b. Cervical ineptitude C.

Cervical deficiency

d. Cervical ineffectiveness 3 6 - 1 7 . Home uterine activity monitoring has been associated with which o f the following outcomes? a . Decreased rates o f preterm premature rupture of membranes

b. Decreased rates o f spontaneous preterm birth C.

Decreased rates o f midtrimester pregnancy loss

d. None o f the above

Preterm Birth 3 6 - 1 8 . Potential indications to perform the procedure demon strated in the image include which of the following?

a. Amoxicillin b. Amoxicillin-clavulanate C.

Ampicillin

d. Erythromycin 3 6 - 2 2 . Based on the recommendations by the American Col lege o f Obstetricians and Gynecologists, what is the most appropriate management of preterm premature rupture o f membranes at or beyond 34 weeks’ gesta tion? a . Expectant management Reproduced, with permission, from Cunningham FG , Leveno K J, Bloom SL, et al: Williams

b. Expectant management unless fetal lung maturity is confirmed

Obstetrics, 23rd ed. N ew York, McGraw-Hill, 2010.

C.

a . Recurrent midtrimester losses

A course o f corticosteroids followed by delivery

d. Expedited delivery

b. Short cervix identified during sonography C.

Threatened preterm labor with cervical dilatation

d. All o f the above 3 6 - 1 9 . Based on the known natural history o f preterm pre mature ruptured membranes, approximately what percentage o f women will be delivered within 48 hours o f membrane rupture when this complication occurs between 24 and 34 weeks’ gestation?

a. Greater than 90% b. 75% c. 50%

d. Less than 25% 3 6 - 2 0 . W hat is the only reliable indicator o f clinical chorioam nionitis in women with preterm ruptured membranes? 3. Leukocytosis

b. Fever C.

Fetal tachycardia

d . Positive cervical or vaginal cultures

3 6 - 2 3 . Corticosteroids administered to women at risk for preterm birth have been demonstrated to decrease rates o f neonatal respiratory distress if the birth is delayed for at least what amount o f time after the initiation of therapy?

a.

1 2 hours

b. 24 hours C.

36 hours

d. 48 hours

CHAPTER 36

3 6 - 2 1 . A number o f antibiotic regimens have been used to prolong the latency period in women with preterm ruptured membranes who are attempting expectant management. Which antibiotic should be avoided in this setting since it has been associated with an in creased risk o f necrotizing enterocolitis in the new born?

207

208

Obstetrical Complications

SECTION 7

3 6 - 2 4 . A21-year-old G1P0 presents at 28 weeks’ gestation in active preterm labor and intravenous terbutaline is administered for tocolysis. Approximately 2 hours after this therapy is initiated, she begins to cough, and her pulse oximetry reading is 80% on room air. The fol lowing chest radiograph is obtained. In which o f the following clinical settings is the risk for this complica tion increased?

Reproduced, with permission, from Chen MYM, Pope J r T L , O tt D J. Basic Radiology. N ew York, M cGraw-Hill, 2 0 0 4 , Figure 4-20.

a. Maternal sepsis b. Concurrent administration o f corticosteroids C. Twin pregnancy d. All ofthe above

3 6 - 2 5 . When used as a tocolytic agent, indomethacin admin istration should generally not exceed 24 to 48 hours to prevent what complication?

a. Oligohydramnios b. Fetal-growth restriction C. Placental abruption d. Necrotizing enterocolitis

Preterm Birth

Chapter 36 Answer Key L e tte r answ er

Page c ite d

H e a d e r c ite d

Q u e s tio n num ber

L e tte r answ er

Page c ite d

H e a d e r cite d

36-1

c

p. 804

In tro d u c tio n

3 6 -1 4

b

p. 814

D iagnosis

3 6 -2

c

p. 804

M o rta lity Rates o f P re te rm In fa n ts

3 6 -1 5

c

p. 815

D iagnosis

3 6 -1 6

a

p. 815

D iagnosis

3 6 -3

a

p. 805

M o rb id ity in P re te rm Infants

3 6 -1 7

d

p. 816

D iagnosis

3 6 -4

d

p. 807

Table 3 6 -3

3 6 -1 8

d

p. 8 1 7

D iagnosis

3 6 -5

b

p. 809

M o rb id ity in P re te rm Infants

3 6 -1 9

a

p. 818

M a n a g e m e n t o f P re te rm R uptured M e m b ra n e s and P reterm Labor

3 6 -6

d

p. 811

Reasons fo r P re te rm D e liv e ry

3 6 -2 0

b

p. 819

3 6 -7

a

p. 811

A n te c e d e n ts and C o n trib u tin g Factors

M a n a g e m e n t o f P re te rm R uptured M e m b ra n e s and P reterm Labor

3 6-2 1

b

p. 820

3 6 -8

c

p. 812

A n te c e d e n ts and C o n trib u tin g Factors

M a n a g e m e n t o f P re te rm R uptured M e m b ra n e s and P re te rm Labor

3 6 -9

a

p. 812

A n te c e d e n ts and C o n trib u tin g Factors

3 6 -2 2

d

p. 820

Table 3 6 -8

3 6 -2 3

b

p. 821

3 6 -1 0

c

p. 812

Table 3 6 -6

P re te rm Labor w ith In ta c t M e m b ra n e s

3 6-11

d

p. 812

A n te c e d e n ts and C o n trib u tin g Factors

3 6 -2 4

d

p. 823

P re te rm Labor w ith In ta ct M e m b ra n e s

3 6 -1 2

b

p. 813

A n te c e d e n ts and C o n trib u tin g Factors

3 6 -2 5

a

p. 825

P re te rm Labor w ith In ta c t M e m b ra n e s

3 6 -1 3

c

p. 814

D iagnosis

Q u e s tio n num ber

CHAPTER 37

Postterm Pregnancy 3 7 -1 . What is the threshold o f completed weeks, after which a pregnancy is considered prolonged?

3. Labor was induced in 55% o f pregnancies complet ing 42 weeks,

a. 40 weeks W) 41 weeks C.

3 7 -5 . Concerning Parkland Hospital between 1988 and 1998, which o f the following is true?

W) The rate o f cesarean delivery and fetal distress was increased in those at 42 completed weeks.

42 weeks

Y) 43 weeks

C.

3 7 - 2 . Which o f the following is the most accurate way to confirm postterm pregnancy? a . Precise last menstrual period (LMP) W) Sonography performed at < 1 2 weeks C.

Sonography performed at 13-24 weeks

Infants o f postterm pregnancies were admitted predominantly to special care nurseries,

d. The incidence o f neonatal seizures and deaths tripled at 42 completed weeks. 3 7 -6 . This image illustrates which ofthe following?

d . Sonography performed at 36—42 weeks

H f^c V hV a b d fhV a^hm fV h

3 7 -3 . Rare fetal-placental factors associated with postterm pregnancy include which o f the following? a . Autosomal recessive placental sulfatase deficiency W) Adrenal hypoplasia C.

Iniencephaly

Y) Wilms tumor 3 7 -4 . This image suggests which o f the following?

O

d [ W ^fh

Reproduced, with permission, from Cunningham F G , Leveno K J, Bloom SL , et al: Williams Obstetrics, 23rd ed. New York, McGraw-Hill, 2010.

a. Highest P M R occurs at 43 weeks W) Perinatal risk index, which accounts for risk o f all ongoing pregnancies, is highest at 36 weeks C.

Highest PM R occurs at 38 weeks

Y) None o f the above

O

gs \ ghVh^dc Vh Y a^j fm

Reproduced, with permission, from Cunningham F G , Leveno K J, Bloom SL, et al: Williams Obstetrics, 23rd ed. New York, McGraw-Hill, 2010.

a . Perinatal mortality rate (PMR) increases after 40 weeks. W) PM Rw ashigherin 1977-1978. C.

PM R increases after 41 weeks,

Y) P M R waslowestin 1943—1952.

Postterm Pregnancy 3 7 -9 . This figure demonstrates which o f the following?

syndrome afflicting this infant? ::

_J_ Z.

--

r 4 -i

I

-180-

I -4.1L

IVW

Vr

w-

N

h r *

-L ~

Sk

I yu -

_

-I5 0 -

-90

-



;G [^h _ /+ ,

r* f

■J.

L S0-

o

,



(C'

60

,

-

-120-

i

^(jN ,r

...

— 1—~

A -IIft

...

-180-

I

~~r20u

U p ^ D(C)

+

^

AC U

Cr

*4/L AS' \ , ,

RUCraU D

+

r

Reproduced, with permission, from Cunningham FG , Leveno K J, Bloom SL, et al: Williams Obstetrics, 23rd ed. New York, McGraw-Hill, 2010.

V) Variable deceleration

b. Late deceleration C. Prolonged deceleration

d. None o f the above

3 7 -1 0 . Which of the following is true regarding fetal-growth restriction in the postterm fetus? V) Rare occurrence Obstetrics, 23rd ed. N ew York, McGraw-Hill, 2010.

a. Its incidence in pregnancies between 41 and 43 weeks is 2 0 %.

b. Neurological deficits are found in 23% o f affected newborns. C. Associated oligohydramnios substantially increases

the likelihood at 42 weeks,

d. Features include simian crease and low set ears. 37 -8 . Which o f the following may be associated with placental dysfunction? V) Increased placental apoptosis b. Increased cord blood erythropoietin C. Postmaturity syndrome

d. All o f the above

b. Not an indication for delivery C. Associated with increased perinatal morbidity and

mortality rates

d. None o f the above 3 7 -1 1 . Concerning amnionic fluid, which of the following is true? a . Oligohydramnios is only associated with risk to the postterm fetus.

b. The amnionic fluid index (API) is more specific for abnormal fetal outcomes than the deepest vertical pocket. C. Normal amnionic fluid precludes all normal out

comes.

d. With oligohydramnios in postterm pregnancy, the incidence o f fetal distress in labor is increased.

CHAPTER 37

j*

__>_DU

"W

I

3 7 -7 . Which o f the following is true concerning the

Reproduced, with permission, from Cunningham FG , Leveno K J, Bloom SL, et al: Williams

211

212

Obstetrical Complications 3 7 -1 2 . This image illustrates which o f the following?

3 7 - 1 4 . In the presence o f macrosomia, which ofthe following is true?

a . Early induction decreases maternal and fetal morbidity rates.

b. Cesarean delivery should be performed for M e e f fV c \

estimated fetal weight greater than 4,000 g. C. Cesarean delivery is recommended for estimated fe tal weight greater than 4,500 g if there is prolonged second-stage labor,

d. None of the above. 3 7 - 1 5 . Concerning the management ofprolonged pregnan cies, which o f the following is true?

a . Some form o f intervention is indicated. b. O f American College of Obstetrician and Gynecol 38

39

40 O

41

ogists (A CO G ) members, 50% routinely induce women at 41 completed weeks.

42

g s \ g hV h^d c

Reproduced, with permission, from Cunningham F G , Leveno KJ, Bloom SL, et al: W illiams Obstetrics, 23rd ed. N ew York, McGraw-Hill, 2010.

a . The largest amount o f amnionic fluid is present before term.

C.

O f A C O G members, 50% routinely induce women at 42 completed weeks,

d. All ofthe above 3 7 -1 6 . Concerning an unfavorable cervix, research supports which o f the following statements?

b. Amnionic fluid decreases from term until 43 weeks.

a . O f women at 42 weeks, 92% have an unfavorable

C. The smallest amount o f fluid in the upper range is seen at 41 weeks,

b. The risk o f cesarean delivery is increased twofold in

cervix, when defined as a Bishop score o f less than 7. those with a closed cervix at 42 weeks undergoing labor induction.

d . The greatest amount o f fluid in the lower range is approximately 700 cc at 38 weeks. 3 7 -1 3 . This image illustrates which o f the following?

C. A cervical length o f 3 cm or less was predictive of successful induction,

d. All o f the above. 3 7 - 1 7 . Concerning membrane stripping, research supports which o f the following statements?

a. It increases maternal infection rates. b. It decreases the need for induction. C.

Its complications include bleeding and pain,

d. All o f the above.

O

gs \ ghVh^dc

Reproduced, with permission, from Cunningham F G , Leveno K J, Bloom SL, etal: Williams Obstetrics, 23rd ed. N ew York, M cGraw-Hill, 2010.

3. Fetuses lose weight after 40 weeks.

b. The peak of fetal growth occurs in the late midtrimester. C.

Fetal growth continues until at least 42 weeks.

d . Maternal and fetal morbidity associated with macrosomia would be mitigated with timely induction.

Postterm Pregnancy 3 7 -2 1 . From one observational study by Usher and colleagues (1988), which ofthe following perinatal death rates is correct?

3 7 -1 8 . This image illustrates which ofthe following?

o

a. 3.0/1,000 a t 4 2 weeks

,++%

,++

0

Q.

80

b.

1 .5 / 1 , 0 0 0 a t 4 l weeks

C.

0.7/1,000 a t 40 weeks

CHAPTER 37

\

d. 0.3/1,000 a t 39 weeks

60

3 7 -2 2 . Concerning this algorithm that summarizes A C O G recommendations, which o f the following is true?

CO 40 E o c n 20 <

C o m p le te d 41 w k N o o th e r c o m p lic a tio n s

6.7 X b (5th e fX c h^a

2 5 0 mg/d

a. Preterm births

C. > 3 0 0 mg/d

b. Pregnancy-associated hypertension

d. > 7 0 0 mg/d

C. Pyelonephritis

following?

d. Low birth weight

4 8 -: . Which o f the following statements about renal func tion in pregnancy is true?

4 8 -6 . The recurrence rate o f ASB during the same pregnancy

a. Pregnancy induces intrarenal vasoconstriction.

is which o f the following?

b. Glomerular filtration rate increases by 40-65% .

a. 10%

C. Serum creatinine levels increase.

b. 30%

d. Serum urea concentration increases.

c. 60% d. 90%

4 8 - . A 30-year-old G 2P 1 at 20 weeks’ gestation presents with a history o f nephrectomy after a car accident 4 years ago. The patient has no other medical his tory. She had one normal spontaneous vaginal delivery 7 years ago. In regard to the patient’s pregnancy, which o f the following counseling points is true?

4 8 -7 . A 17-year-old G2P1 at 20 weeks’ gestation presents with dysuria, frequency, and urgency. She is afebrile and has no costovertebral angle tenderness. Urinalysis indicates pyuria but her urine culture is negative. The likely offending organism is:

a . She is at increased risk for preeclampsia.

a . Escherichia coli

b. Renal failure is highly likely.

b. Pseudomonas aeruginosa

C. You will evaluate her renal function. If normal, she

should expect a normal pregnancy and one without renal complications, d. She is at high risk for spontaneous abortion because o f her kidney’s inability to filter properly.

4 8 -4 . A 15-year-old G1P0 presents for her first prenatal visit. She has no complaints. A urine culture done that day identifies > 1 0 0 ,0 0 0 gram-negative rods per mL in her urine. Which of the following statements about the patient’s management is true?

a . Treatment is not required because she is asymp tomatic. b. Treatment is not required because the count is not > 1 million gram-negative rods per mL. C. Treatment should be delayed until susceptibilities

are determined, d . Treatment should be prescribed at this visit to avert symptomatic infection.

C. Chlamydia trachomatis

d. Group B streptococcus

Renal and Urinary Tract Disorders 4 8 -8 . In cases o f pyelonephritis in pregnancy, the most com mon organism isolated is pictured here. The organism is which of the following?

4 8 -1 0 . A 17-year-old G1P0 at 20 weeks’ gestation presents

Contributed by Professor Shirley Lowe, University o f California, San Francisco School o f

a. Group B streptococcus C. Klebsiella pneumoniae

d. Proteus mirabilis

4 8 -9 . Which o f the following is the most common complica

Reproduced with permission from Cunningham FG , et al: W illiams Obstetrics, 22nd ed. New York, McGraw-Hill, 2005, Figure 42-2.

a. 0.5% b. 1 0 %

tion o f pyelonephritis in pregnancy?

c. 25%

a. Anemia

d. 50%

b. Renal dysfunction C. Respiratory insufficiency

d. Bacteremia

4 8 -1 1 . In a setting o f limited resources, which o f the following steps could be eliminated from the management of pyelonephritis in pregnancy to be more cost effective?

a. Urine culture b. Intravenous antibiotics C. Intravenous fluids

d. Blood culture

4 8 -1 2 . In pregnant women treated for pyelonephritis, 95% are afebrile by what time? a. 1 2 hours b. 24 hours C. 48 hours

d. 72 hours

CHAPTER 4

to the emergency department complaining o f fever, flank pain, and dysuria for 5 days. She is admitted with pyelonephritis and started on intravenous fluids and antibiotics. Eighteen hours later, the patient is com plaining o f shortness of breath. Her respiratory rate is 26, and her O 2 saturation is 89%. A chest radiograph is obtained, and it is provided here. What percentage o f women with pyelonephritis have this complication?

Medicine.

b. Escherichia coli

271

Medical and Surgical Complications 4 8 - 1 3 . A 22-year-old G IPO at20 weeks’ gestation is admitted for pyelonephritis. She is started on intravenous an tibiotics to which the offending organism is found to be susceptible. After 72 hours, the patient continues to spike temperatures o f 39 °C . Which o f the following is the next best management step?

a. Computed tomography (C T) scan

4 8 - 1 7 . A 24-year-old G1P0 at 15 weeks’ gestation presents with severe flank pain and hematuria. She is afebrile. You suspect she has a kidney stone, and a renal ultra sound is performed. An image from that study is pro vided here, and the arrow points to the area o f interest. A urinalysis is positive only for blood. Which o f the following is the next best management step?

b. Stop antibiotics C. Renal sonography

d. Renal biopsy 4 8 - 1 4 . Th epatientin Question 48 -13 eventually improves. After being afebrile for 24 hours, she is discharged home with a 7-day course o f oral antibiotics. Which of the following is the best way to prevent reinfection? a . Urine cultures at every prenatal visit

b. Nitrofurantoin 100 m g orally at bedtime for the remainder o f the pregnancy C. Recommend she consume 2 L o f water per day

d. Prescribe pelvic rest for the remainder o f the preg nancy 4 8 - 1 5 . Which o f the following promotes the formation of kidney stones?

a. Low calcium diet b. Low sodium diet C. Low protein diet

d. Thiazide diuretics 48-1 6. In pregnant women, which ofthe following is the most common symptom o f nephrolithiasis?

Reproduced, with permission, from Tanagho EA, McAninch JW: Sm iths General Urology, 17th ed. N ew York, McGraw-Hill, 2008, Figure 6-23.

a. C T scan to confirm nephrolithiasis followed by intravenous hydration and pain control

b. Intravenous hydration and pain control only C. Pain medication, antibiotics, and ureteral stenting

d. Intravenous pyelogram to confirm nephrolithia sis, followed by hydration, pain control, and laser lithotripsy

a. Hematuria b. Fever C. Frequency

d. Pain

4 8 - 1 8 . For renal disease to be considered chronic, it must be present for what length o f time?

a . 3 weeks b. 6 weeks C. 3 months

d.

6 months

4 8 - 1 9 . WTiich o f the following is the most common cause o f end-stage renal disease?

a. Hypertension b. Polycystic kidney disease C. Diabetes mellitus

d. Glomerulonephritis

Renal and Urinary Tract Disorders

273

4 8 - 2 0 . Which o f the following statements about physiologic adaptations during pregnancy is true?

CHAPTER 4

3. In cases o f mild renal insufficiency, pregnancy in creases glomerular filtration,

b. In cases o f severe renal insufficiency, pregnancy increases glomerular filtration. C. Women with mild renal dysfunction do not expand

their blood volume,

d . In women with severe renal dysfunction, pregnancy causes anemia to improve. 4 8 - 2 1 . Which o f the following is the most common complica tion in pregnant women with chronic renal disease?

a. Fetal-growth restriction b. Worsening renal function C. Preterm birth

d. Hypertension 4 8 - 2 2 . In managing women with chronic renal disease, which of the following is true? a . The patient should be placed on a low protein diet while pregnant.

b. These women are infertile so birth control does not need to be addressed. C. To avoid abruptvolume changes and hypotension

in those pregnant women undergoing dialysis, the frequency o f dialysis sessions should be increased,

d. Peritoneal dialysis is contraindicated in pregnancy. 4 8 - 2 3 . A 32-year-old nulligravida presents 1 year after a Iddney transplant for preconceptional counseling. Which o f the following is appropriate counseling?

Reproduced, with permission, from Fauci A S, Braunwald E, Kasper D L: Harrison’s Princi ples o f Internal Medicine, 17th ed. New York, McGraw-Hill, 2008, Figure 278-11.

a. Most cases are due to PKD1 gene mutation on chro mosome 16.

b. Hypertension is uncommon. C. Prenatal diagnosis is available if the mutation has been identified in a family member or if linkage has been established in the family, d . Ten percent o f patients with this disease die from rupture o f an associated intracranial berry aneurysm. 4 8 - 2 5 . Which ofthe following is N O T a cause o f nephritic syndrome?

a. Diabetes mellitus

a . She can start trying to get pregnant anytime.

b. Subacute bacterial endocarditis

b. The severity o f her hypertension is not important in

C. Systemic lupus erythematosus

regard to the pregnancy. C. Corticosteroids will not increase her chances of gestational diabetes,

d . Renal function should be stable with a serum creati nine o f < 2 m g/dL and preferably < 1 .5 mg/dL.

d. Poststreptococcal infection 4 8 - 2 6 . Which o f the following is the most common cause of minimal change disease? a . Nonsteroidal anti-inflammatory drugs

b. Viral infections C. Idiopathic

d. Allergies

2 74

Medical and Surgical Complications

8 N 0 ID 3 S

4 8 - 2 7 . In pregnant women with nephrotic syndrome, care should include which o f the following? a . Frequent patient visits for blood pressure evaluation because o f associated preeclampsia risk b. Surveillance o f fetal growth because o f associated growth-restriction risks C. Counseling regarding the associated increased risk for preterm birth d. All o f the above

4 8 - 2 8 . A 38-year-old G3P2 at 37 weeks’ gestation with a known history o f chronic hypertension presents with abdominal pain, vaginal bleeding, headache, and de creased fetal movement. She is diagnosed with placen tal abruption, fetal demise, and superimposed severe preeclampsia. To avoid acute tubular necrosis, which o f the following is the most appropriate management step? a . Reduce her blood pressure to a low normal level b. Administer diuretics if she develops oliguria C. Administer vasoconstrictors if she becomes hy potensive from blood loss d . Promptly and vigorously replace blood losses

Renal and Urinary Tract Disorders

275

Chapter 48 Answer Key L e tte r answ er

P age c ite d

H e a d e r c ite d

Q u e s tio n num ber

L e tte r answ er

Page c ite d

H e a d e r c ite d

4 8 -1

c

p. 1033

A sse ssm e n t o f Renal Function D uring P re g n a ncy

4 8 -1 6

d

p. 10 3 8

Stone D isease D uring P regnancy

4 8 -2

b

p. 1033

P re g n a n cy-In d u ce d U rinary Tract Changes

4 8 -1 7

b

p. 1039

Stone Disease D uring P regnancy

4 8 -3

c

p. 10 3 4

P regnancy a fte r U n ila te ra l N e p h re c to m y

4 8 -1 8

c

p. 1039

C hronic Renal Disease

4 8 -1 9

c

p. 10 3 9

C hronic Renal Disease

4 8 -4

d

p. 1035

U rinary Tract In fe c tio n s

4 8 -2 0

a

p. 1039

Chronic Renal Disease

4 8 -5

c

p. 1035

U rinary Tract In fe c tio n s

48-21

d

p. 10 4 0

Table 48-3

4 8 -6

b

p. 1035

U rinary Tract In fe c tio n s

4 8 -2 2

c

p. 1041

D ialysis D u rin g P regnancy

4 8 -7

c

p. 1036

Cystitis and U re th ritis

4 8 -2 3

d

p. 1042

4 8 -8

b

p. 10 3 6

A cu te P y e lo n e p h ritis

P regnancy a fte r Renal T ra nsp lanta tion

4 8 -9

a

p. 1 0 3 6

A cu te P y e lo n e p h ritis

4 8 -2 4

b

p. 1043

4 8 -1 0

b

p. 10 3 7

A cute P ye lo n e p h ritis

P olycystic K idney Disease and P reg nancy

4 8 -1 1

d

p. 1 0 3 7

A cu te P ye lo n e p h ritis

4 8 -2 5

a

p. 1043

Table 4 8 -5

4 8 -1 2

d

p. 10 3 7

A cute P ye lo n e p h ritis

4 8 -2 6

c

p. 1044

Table 4 8 -6

4 8 -1 3

c

p. 10 3 8

A cute P ye lo n e p h ritis

4 8 -2 7

d

p. 1045

A cute Renal Failure

4 8 -1 4

b

p. 10 3 8

A cu te P y e lo n e p h ritis

4 8 -2 8

d

p. 1 0 4 6

P re ve n tio n

4 8 -1 5

a

p. 10 3 8

Reflux N e p h ro p a th y

CHAPTER 4

Q u e s tio n num ber

Gastrointestinal Disorders 4 9 - 1 . Based on general experience, which o f the following diagnostic studies are safe to use in pregnancy?

4 9 - 6 . Which ofthe following is the most common complica tion o f total parenteral nutrition?

a. Lower endoscopy

a. Hemothorax

b. Flexible sigmoidoscopy

b. Catheter sepsis

C. Endoscopic retrograde cholangiopancreatography (ERCP)

C. Brachial plexus injury

d. All ofthe above 4 9 - 2 . All E X C E P T which o f the following are common indi cations for nonobstetrical surgery in pregnancy?

d . Fetal sub dural hematoma caused by vitamin K deficiency 4 9 - 7 . What organism has been implicated as a causative organism in hyperemesis gravidarum?

a. Appendicitis

a. Heliobacterpylori

b. Cholecystitis

b. Bacteroidesfragilis

C. Adnexal mass

C. Streptococcus viridans

d. Nephrolithiasis

d . Clostridium difficile

4 9 - 3 . Preference is given to which ofthe following when trying to maintain adequate nutrition in a pregnant patient with nausea/vomiting?

4 9 - 8 . This endoscopic view shows which ofthe following?

a. Parenteral feeding b. Hyperalimentation C. Enteral alimentation

d. Dextrose-containing solutions 4 9 - 4 . What is the incidence o f laparotomy and laparoscopy for nonobstetric indications in pregnancy?

a. 1:50 b. 1:100 C. 1:500

d. 1:1,000 4 9 - 5 . When counseling a patient who needs nonobstetric surgery in pregnancy, which o f the following should be disclosed?

a . There is increased risk of stillbirth. b . There is increased risk o f cerebral palsy.

Reproduced, with permission, from Fauci AS, Braunwald E, Kasper D L : H arrisons Princi ples o f Internal Medicine, 17th ed. New York, M cGraw-Hill, 2008, Figure 285-18.

C. There is increased risk o f preterm delivery,

d. There is no increase in long-term risks to the fetus or the mother.

a. H. pylori infection b. Mallory-Weiss tear C. Barrett esophagus

d . Inflammatory bowel disease

Gastrointestinal Disorders 4 9 - 9 . Which o f the following is true o f hyperemesis gravi darum?

a . It is very common in pregnancy up until the 16th

4 9 - 1 6 . The complication seen in this gastric endoscopic image is uncommon in pregnancy for which of the following reasons?

week o f gestation.

b. It can cause weight loss, dehydration, and hypokalemia. C. It is more likely to trigger hospitalization in obese patients. d . It is more common if carrying a male fetus.

4 9 - 1 0 . Wernicke encephalopathy is a life-threatening compli cation o f recalcitrant hyperemesis gravidarum associ ated to a deficiency o f which o f the following?

a. Thiamine b. Vitamin K C. (3-carotene d . Phenylalanine

4 9 - 1 1 . Which o f the following is first-line treatment in the management of hyperemesis gravidarum? a. Glucocorticoids

Reproduced, with permission, from Greenberger N J, Blumberg RS, Burakoff R: Current Diagnosis & Treatment: Gastroenterology, Hepatology, & Endoscopy. New York, McGrawHill, 2009, Plate 51.

b. Enteral nutrition C. Ondansetron (Zofran) d . IV hydration and antiemetics

a. H.pylori infection is uncommon in pregnancy.

b. Gastric acid production is decreased in pregnancy. C. Motility o f the gastrointestinal tract is decreased in

4 9 - 1 2 . Which vitamin deficiency is most commonly associ ated with hyperemesis gravidarum?

a. Vitamin A b. Vitamin D C. Vitamin E and alanine d . Vitamin K and thiamine 4 9 - 1 3 . Which o f the following is N O T safe to use in preg nancy for the treatment o f reflux esophagitis?

a. Cimetidine b. Omeprazole

pregnancy,

d. All of the above. 4 9 -1 7 . All E X C E P T which o f the following are adequate treatment choices for H. pylori in pregnancy? a. Amoxicillin

b. Tetracycline C. Clarithromycin

d. Metronidazole 4 9 -1 8 . Ulcerative colitis and Crohn disease share all E X C E P T

C. Misoprostol

which o f the following clinical features?

d . Calcium carbonate

a. Genetic predisposition

b. Cured by proctocolectomy 4 9 - 1 4 . W hat is the hospital readmission rate in patients with hyperemesis gravidarum?

a. 30% b. 45%

C. Periods o f exacerbation and remission

d. Can be associated with erythema nodosum 4 9 -1 9 . Which is a clinical feature typical of Crohn disease?

c. 60%

a . Rectum is usually spared.

d . 75%

b. Affected areas o f bowel are contiguous.

4 9 - 1 5 . What is the main mechanism behind the symptoms of reflux esophagitis in pregnancy?

a. Decreased peristalsis b. Excess gastric acid production C. Relaxation o f the upper esophageal sphincter d . Relaxation o f the lower esophageal sphincter

C. It is associated with antineutrophil cytoplasm anti body.

d . Risk o f cancer is greater than with ulcerative colitis.

Medical and Surgical Complications 4 9 - 2 0 . A 11EXCEPT which ofthe following are inflammatory bowel disease treatment options during pregnancy?

4 9 - 2 6 . What is the most common presentation o f bowel ob struction in pregnancy?

a. Infliximab

a. Diarrhea

b. Mesalamine

b. Nausea and vomiting

C. Methotrexate

C. Abnormal bowel sounds

d. Glucocorticoids

d. Continuous or colicky abdominal pain

4 9 - 2 1 . Subfertility associated with inflammatory bowel disease m ay be linked to which o f the following?

4 9 - 2 7 . This image shows which o f the following?

a. Rectovaginal fistulas b. Use o f immune modulators as treatment C. Sperm abnormalities caused by sulfasalazine

d . None o f the above 4 9 - 2 2 . Which o f the following statements is true about in flammatory bowel disease?

a . Relapses are usually mild. b . Pregnancy increases the likelihood of a flare. C. M ost treatments must be discontinued during preg nancy.

d. Active disease in early pregnancy increases the likeli hood o f poor pregnancy outcome. 4 9 - 2 3 . After proctocolectomy for the treatment o f ulcerative colitis, which o f the following are accurate counseling points?

a . The pacient can expect decreased fertility. b. The patient should always be delivered by cesarean. C. The patient can expect improved sexual function,

d. None o f the above. 4 9 - 2 4 . Which o f the following should be routinely supple mented in pregnant patients with inflammatory bowel disease?

a. Calcium b. Selenium C. Chromium

d. Potassium 4 9 - 2 5 . During pregnancy and the puerperium, what is the most common cause ofintestinal obstruction?

a. Volvulus b. Adhesions C. Carcinoma

d. Intussusception

Reproduced, with permission, from Cunningham F G , Leveno K J, Bloom SL, et al: Williams Obstetrics, 23rd ed. N ew York, McGraw-Hill, 2010.

a. Crohn disease b. Intussusception C. Colonic volvulus

d. Ulcerative colitis 4 9 - 2 8 . What is the approximate incidence o f appendectomy in pregnancy?

a. 1:500 b. 1:1,000 C. 1:5,000

d. 1:10,000 4 9 - 2 9 . Which o f the following is the most commonly observed finding o f appendicitis in pregnancy?

a. Fever b. Anorexia C. Nausea and vomiting

d. Persistent abdominal pain and tenderness

Gastrointestinal Disorders

2 79

4 9 - 3 0 . Which o f the following diagnostic methods provides the lowest negative appendectomy rate? a . Computed tomography (CT)

CHAPTER 4

b. Magnetic resonance imaging (MRI) C. Sonography and C T

d. Sonography and MRI 4 9 - 3 1 . Regarding the complication shown here, which of the following is true? a. The risk o f spontaneous abortions is increased. b. Surgical evaluation is best postponed until the sec ond trimester. C. Accuracy o f diagnosis improves with increasing

gestational age. d. Tocolytics decrease the risk o f preterm labor associ ated with this complication.

Reproduced from I Pedrosa, D Levine, A D Eyvazzadeh, et al: M R imaging evaluation in pregnancy, Radiology, 2 0 06 ;2 3 8:8 9 1 -8 9 9 , with permission from The Radiological Society o f N orth America and Dr. Ivan Pedrosa.

Medical and Surgical Complications

Chapter 49 Answer Key Q u e s tio n num ber

L e tte r answ er

Page c ite d

H e a d e r c ite d

Q u e s tio n num ber

L e tte r answ er

Page c ite d

H e a d e r c ite d

4 9 -1 7

b

p. 1053

P eptic U lc e r/M a n a g e m e n t

4 9 -1 8

b

p. 1054

Table 49-3

4 9 -1 9

a

p. 1055

C rohn D isease

4 9 -2 0

c

p. 1055

In fla m m a to ry B ow e l D is e a s e /M a n a g e m e n t

4 9 -1

d

p. 1 0 4 9

G eneral C o n s id e ra tio n s / E ndoscopy

4 9 -2

d

p. 1 0 5 0

L a p a ro to m y and Laparoscopy

4 9 -3

c

p. 1050

N u tritio n a l S up p o rt

4 9 -4

c

p. 1050

L a p a ro to m y and Laparoscopy

4 9 -5

d

p. 10 5 0

L a p a ro to m y and Laparoscopy

4 9-2 1

c

p. 1055

4 9 -6

b

p. 1050

P are n te ra l N u tritio n D uring P re g n a n cy

In fla m m a to ry B o w e l D isease and F e rtility

4 9 -2 2

d

p. 1055

H yp e re m e s is G ra v id a ru m / H. p y lo ri In fe c tio n

In fla m m a to ry B o w e l D isease and P re gnancy

4 9 -2 3

c

p. 1056

H yp e re m e s is G ra v id a ru m / C om p lica tio n s

U lce ra tive C olitis and P re g n a n cy

4 9 -2 4

a

p. 1056

U C /C rohn Disease and P re g n a n cy

4 9 -2 5

b

p. 1057

In te s tin a l O b s tru ctio n and Table 4 9 -4

4 9 -7 4 9 -8

a b

p. 1051 p. 1051

4 9 -9

b

p. 1050

H yp e re m e s is G ravida rum

4 9 -1 0

a

p. 1051

H yp e re m e s is G ra vida ru m

4 9-1 1

d

p. 1052

H yp e re m e s is G ra v id a ru m / M anagem ent

4 9 -2 6

d

p. 10 5 7

In te s tin a l O b s tru c tio n / E tio p a th o g e n e s is

4 9 -1 2

d

p. 1051

H yp e re m e s is G ra v id a ru m / C o m p lica tio n s

4 9 -2 7

c

p. 1057

Figure 4 9 -2

4 9 -1 3

c

p. 1052

R e flu x E sophagitis

4 9 -2 8

b

p. 1058

A p p e n d ic itis

4 9 -1 4

a

p. 1052

H yp e re m e s is G ra v id a ru m / M anagem ent

4 9 -2 9

d

p. 1058

A p p e n d ic itis /D ia g n o s is

4 9 -3 0

c

p. 10 5 9

A p p e n d ic itis /D ia g n o s is

4 9-31

a

p. 1059

A p p e n d ic itis /M a n a g e m e n t

4 9 -1 5

d

p. 1052

R e flu x E sophagitis

4 9 -1 6

d

p. 1053

P eptic Ulcer

Hepatic, Gallbladder, and Pancreatic Disorders 5 0 - 1 . O f the following findings that can be associated with liver dysfunction, which is a normal physiologic change in pregnancy? a . Elevation o f hepatic transaminases

b. Spider angiomas C.

Esophageal varices

5 0 - 6 . Which of the following is the safest and most effective treatment for cholestasis of pregnancy?

a. Ursodeoxycholic acid b. Benadryl C.

\

Topical emollients

d. Plasmapheresis

d. Asterixis 5 0 - 2 . A known risk factor for intrahepatic cholestasis o f preg nancy may include which o f the following?

a. Twin pregnancies b. Women o f Italian descent C.

Certain specific gene mutations

d. All ofthe above 5 0 - 3 . Which o f the following clinical features are characteris tic o f intrahepatic cholestasis?

a. Serum transaminases > 5 0 0 U/L b. Maculopapular rash C.

Generalized pruritus

d. Nausea and vomiting 5 0 - 4 . Which o f the following chronic viral infections has been associated with a marked increased risk for cholestasis o f pregnancy?

a . Hepatitis B b. Hepatitis C C.

Human immunodeficiency virus (HIV)

d . Cytomegalovirus (CMV) 5 0 - 5 . Liver biopsy in women who suffer from cholestasis o f pregnancy would be expected to show which o f the following findings?

a . Bile plugs in hepatocytes b. Inflammatory changes C.

Periportal necrosis

d. All o f the above

5 0 - 7 . Mutations in enzymes involved in fatty acid oxidation have been classically associated with maternal acute fatty liver of pregnancy when they occur in what pat tern? a . Homozygous mutation in the fetus and the mother

b. Homozygous mutation in the fetus; heterozygous mutation in the mother C.

Heterozygous mutation in the fetus and the mother

d. Heterozygous mutation in the fetus; homozygous mutation in the mother 5 0 - 8 . All E X C E P T which o f the following are clinical char acteristics that increase the risk for acute fatty liver of pregnancy?

a. Nulliparity b. Third trimester C.

Female fetus

d. Twin gestation

282

Medical and Surgical Complications Thefollowing case applies to Questions 5 0 - 9 through 50—11. A 33-year-old G 2P 1 presents at 35 weeks’ gestation with complaints o f nausea and vomiting. Laboratory analysis reveals elevated transaminases and creatinine levels and coagulopathy. A peripheral smear is per formed with the results shown. 5 0 - 9 . What is the underlying etiology o f these hemolyzed cells found on the blood smear in this patient?

5 0 -1 2 . Postpartum acute pancreatitis occurs in up to what percentage o f women who have acute fatty liver of pregnancy? a. 10 % b. 25% c. 50% d . 75%

5 0 -1 3 . All >PHL which o f the following acute liver dis eases o f pregnancy can be associated with thrombocy topenia? a. Viral hepatitis

b. Preeclampsia C. Acute fatty liver o f pregnancy d . Cholestasis o f pregnancy

5 0 - 1 4 . When associated with acute viral hepatitis, all >PHL which o f the following symptoms and signs are indications o f severe disease and should prompt hospitalization? a. Hyperglycemia Reproduced with permission from Lichtman MA, Beutler E, K ipps T J: Color Atlas III: Red

b. Coagulopathy Hypoalbuminemia

Cell Morphology. In W illiams Hematology, 7th ed. New York, M cGraw-Hill, 2006, Plate

C.

III-3.

d . Central nervous system symptoms

a . Increased destruction in the spleen

b. Intense vasospasm C. Occlusive pain crisis

d . Decreased cholesterol production

5 0 - 1 5 . Acute hepatitis that progresses to fulminant hepatic necrosis is most likely to be caused by which o f the following viruses? a. Hepatitis A virus b. Hepatitis B virus

5 0 - 1 0 . As a part o f the evaluation, you want to confirm the suspected diagnosis with imaging. Which of the fol lowing modalities is most appropriate?

a. Sonography b. Computed tomography

C. Hepatitis E virus

d. Any of the above

5 0 -1 6 . What is the most common complication ofhepatitis B infection?

C. Magnetic resonance imaging

a. Hepatocellular carcinoma

d. None o f the above

b. Esophageal varices

5 0 - 1 1 . After you stabilize the patient and correct her coagu lopathy, you induce labor and she has a vaginal deliv ery. O n postpartum day 2, she appears to be doing well, but you notice that her urine output has increased to approximately 800 cc per hour. What is the most likely cause o f this condition?

C. Chronic infection

d. Cirrhosis

5 0 -1 7 . In patients who require interferon pharmacotherapy for treatment o f chronic hepatitis, what is the approxi mate cure rate?

a. Hypothalamic dysfunction

a. 5-10%

b. Elevated serum vasopressinase concentrations

b. 10-20%

C. Acute tubular necrosis

d. Pituitary tumor

c. 30-40%

d. 60-70%

Hepatic, Gallbladder, and Pancreatic Disorders 5 0 -1 8 . W hat is the primary route o f transmission for the hep atitis Avirus?

283

5 0 -2 3 . In the general population, what is the most common cause o f the type o f liver injury seen here?

a. Fecal-oral

CHAPTER 5

b. Respiratory droplets C. Sexual

d. Injection drug use 5 0 - 1 9 . I f the genetic makeup of the viral particle shown in this sketch is DN A , which type o f hepatitis virus is represented here?

Reproduced, with permission, from Brunicardi F C , Anderson D K , Billiar T R , ec al: Schwartz’s Principles o f Surgery. 9 th ed. N ew York, McGraw-Hill, 2010, Figure 11-21.

a. Alcohol exposure

b. Hepatitis B Reproduced, with permission, from Ryan K, et al: Sherris M edical Microbiology, 3rd ed. Appleton and Lange, 1994, Figure 41-1.

C. Autoimmune hepatitis

d. Steatohepatitis a. Hepatitis A

b. Hepatitis B C. Hepatitis C d . Hepatitis E

5 0 -2 0 . After infection with hepatitis B, what is the first de

5 0 - 2 4 . A 43-year-old G5P4 at 30 weeks’ gestation presents complaining o f hematemesis. An emergent upper en doscopy is performed with the findings o f this study depicted in the image. If this condition is present in pregnancy, which o f the following associated condi tions would most increase her mortality risk?

tectable serological marker? a. H Be antigen b. IgM anti-HB core C. H Bs antigen

d. Anti-HBs 5 0 - 2 1 . Approximately what percentage o f infants who con tract hepatitis B will become chronically infected? a. 15%

b. 30% c. 50%

d. 85% 5 0 - 2 2 . W hat adverse effects does infection with hepatitis C have on pregnant women? a . Increased progression to cirrhosis

b. Higher average serum transaminase levels

Reproduced, with permission, from Fauci AS, Braunwald E, Kasper D L : H arrisons Princi ples o f Internal Medicine, 17th ed. New York, McGraw-Hill, 2008, Figure 285-16.

C. More fetal-growth restriction

d . None o f the above

a. H IV infection b.

Cirrhosis

C. Twin pregnancy ampsia

284

Medical and Surgical Complications 5 0 - 2 5 . A 3 1-year-old G2P1 at 20 weeks’ gestation presents complaining ofvomiting, fever, and right upperquadrant pain. An examination reveals right upperquadrant tenderness. Upper abdominal sonography is performed with the results depicted in the image. What is the most appropriate next management step? a . Discharge home with oral antibiotics b. Admission for serial abdominal examinations and intravenous antibiotics C. Endoscopic retrograde cholangiopancreatography d. Cholecystectomy

Reproduced, with permission, from Knoop K J, Stack LB, Storrow A B , et al: Atlas o f Emergency Medicine, 3rd ed. N ew York, McGraw-Hill, 2010, Figure 24.46. Photo contributor: Stephen J. Leech, M D , R D M S.

Hepatic, Gallbladder, and Pancreatic Disorders

285

Chapter 50 Answer Key L e tte r answ er

Page c ite d

H e a d e r c ite d

Q u e s tio n num ber

L e tte r answ er

1

Page c ite d

H e a d e r c ite d

50-1

b

p. 1063

H epatic D isorders

5 0 -1 4

a

p. 10 6 7

H e p a tic D isorders

5 0 -2

d

p. 1063

H epatic D isorders

5 0 -1 5

b

p. 106 7

H epatic D isorders

5 0 -3

c

p. 1064

H epatic Disorders

5 0 -1 6

c

p. 1068

H epatic D isorders

5 0 -4

b

p. 1064

H epatic D isorders

5 0 -1 7

c

p. 1068

H e p a tic D isorders

5 0 -5

a

p. 1064

H epatic D isorders

5 0 -1 8

a

p. 10 6 9

H e p a tic D isorders

5 0 -6

a

p. 1064

H epatic D isorders

5 0 -1 9

b

p. 10 6 9

H epatic D isorders

5 0 -7

b

p. 1065

H epatic D isorders

5 0 -2 0

c

p. 10 69

H epatic D isorders

5 0 -8

c

p. 1066

H epatic D isorders

5 0-21

d

p. 10 7 0

H epatic D isorders

5 0 -9

d

p. 1066

H epatic D isorders

5 0 -2 2

d

p. 1071

H epatic D isorders

5 0 -1 0

d

p. 1066

H epatic D isorders

5 0 -2 3

a

p. 1072

H epatic D isorders

5 0-1 1

b

p. 1067

H epatic D isorders

5 0 -2 4

b

p. 1072

H epatic D isorders

5 0 -1 2

c

p. 1067

H epatic D isorders

5 0 -2 5

d

p. 1 0 7 4

G a llb la d d e r D isorders

5 0 -1 3

d

p. 1 0 6 4

Table 50-1

CHAPTER 5

Q u e s tio n num ber

Hematological Disorders 5 1 -1 . Anemia in pregnancy or the puerperium is defined as a hemoglobin concentration o f less than what value?

a. l4g/dL b.

1 2 g/dL

C.

lO g/dL

d.

8 g/dL

5 1 -2 . At what point is the proportion o f plasma expansion to red cell expansion maximal?

a. First trimester b. Second trimester C.

Third trimester

d. Puerperium 5 1 -3 . O fthe 1,000 mg ofiron needed for a typical singleton pregnancy, how much does the fetus require?

a.

1 0 0 mg

b. 300 mg C.

500 mg

d. 700 mg 5 1 -4 . It can be anticipated that a neonate born to a mother with severe iron-deficiency anemia will have what hematological findings?

a. Normal hemoglobin count b. Iron-deficiency anemia C.

Reactive polycythemia

d. Pancytopenia 51 -5 . A 3 4 -year-old G3P2 presents at 30 weeks’ gesta tion to enroll for prenatal care and complains of frequent fatigue. Routine prenatal laboratory tests reveal a hemoglobin concentration o f 8 g/dL. A pe ripheral smear and red cell indices are consistent with hypochromic, microcytic cell changes. What is the most appropriate next step in management?

a . Transfusion with packed red cells b. Supplemental iron given intravenously C. Supplemental iron given orally

d. Recombinant erythropoietin

5 1 -6 . In a hemodynamically stable postpartum woman with acute blood loss anemia, transfusions are generally not indicated as long as the hemoglobin is at least at what level? a . 4 g/dL

b. 5 g/dL C. 6 g/dL

d . 7 g/dL 5 1 -7 . Women who are treated with recombinant erythropoi etin for anemia associated with chronic renal disease must be closely monitored for what side effect? a . Weight loss b.

Hypertension

C.

Hyperglycemia

d. Hyponatremia 5 1 -8 . What is the first sign o f folic acid deficiency? a . Low plasma folate levels b.

Hypersegmented neutrophils

C. Nucleated red cells

d. Macrocytosis 5 1 -9 . According to recommendations by the American Col lege of Obstetricians and Gynecologists, women of childbearing age should consume at least how much folic acid daily? a . 40 mg b.

4 mg

C.

0.4 mg

d. 0 .0 4 mg 5 1 -1 0 . What is the mainstay o f treatment for autoimmune hemolytic anemia? a . Simple transfusion b.

Exchange transfusion

C.

Hydroxyurea

d. Corticosteroids

Hematological Disorders Thefollowing clinical scenario applies to Questions 5 1 -1 1 and 51 - 12.

5 1 -1 5 . In affected patients, red cells may assume the configura tion seen in this photograph under what conditions?

A 23-year-old primigravida at 16 weeks’ gestation presents complaining o f fatigue, increased nausea and vomiting, and “yellow eyes.” The blood counts reveal severe anemia, and a peripheral blood smear is consis tent with spherocytosis. 5 1 -1 1 . This disorder is most commonly caused by a mutation in which of the following genes?

a. |3-Globin gene b. a-G lobin gene C.

Spectrin

d . Methylene tetrahydrofolate reductase 5 1 -1 2 . What additional test could be ordered to confirm the diagnosis?

a. Hemoglobin electrophoresis b. Osmotic fragility testing C.

Indirect Coom b’s test

d. Total iron binding capacity 5 1 -1 3 . What is the mode o f inheritance for glucose-6 phosphate dehydrogenase deficiency?

a. Autosomal recessive b. Autosomal dominant C.

X-linked

d. Mitochondrial 5 1 -1 4 . A 22-year-old primigravida presents at 34 weeks’ gestation with complaints of fever for 3 days. Physical examination is significant only for diffuse petechiae, and laboratory studies reveal pancytopenia. A bone marrow biopsy is performed, and a less than 25% cellularity is found. What is the maternal mortality in this setting?

a. 10% b. 25% c. 50% d. 75%

Reproduced, with permission, from Hail JB , Schmidt GA, Wood L D H : Principles o f Critical Care, 3rd ed. New York, McGraw-Hill, 2005, Figure 108-4.

a . Low-oxygen tension

b. Dietary deficiency C. Administration o f certain antibiotics

d. Hyperglycemia 5 1 -1 6 . What is the approximate incidence o f sickle cell anemia— an autosomal recessive disease— in African Americans if the carrier rate is 1/12?

a. 1/25 b. 1/150 c. 1/300 d. 1/600

287

288

Medical and Surgical Complications

SECTION 8

5 1 - 1 7 . An 18-year-old primigravida with known sickle cell disease presents at 19 weeks’ gestation with complaints o f fever, shortness o f breath, and chest pain. A pul monary consolidation is suspected by chest radiograph, and a C T scan o f the chest is obtained with the results shown. Which o f the following can precipitate this condition?

5 1 - 2 1 . Sickle cell trait (hemoglobin AS) has been associated with an increased risk for which o f the following?

a. Urinary tract infections b. Fetal-growth restriction C.

Placental abruption

d. Preeclampsia 5 1 -2 2 . A 25-year-old Asian primigravida undergoes sono graphic examination at 28 weeks’ gestation. This image o f the fetal abdomen is obtained. Amniocentesis is performed and sent for analysis o f the a-globin gene. What is the expected genotype o f this fetus?

Reproduced, with permission, from Hall JB , Schmidt GA, W o o dL D H : Principles o f Critical Care, 3rd ed. N ew York, McGraw-Hill, 2005, Figure 108-4.

a. Infection b. Bone marrow emboli C.

Atelectasis

d. All o f the above 5 1 - 1 8 . Pregnant women with sickle cell anemia require addi tional supplementation with which o f the following?

a. Vitamin B 1 2 b. Folate C.

Vitamin D

Reproduced, with permission, from Cunningham F G , Leveno K J, Bloom SL, et al: Williams Obstetrics, 23rd ed. N ew York, M cGraw-Hill, 2010.

d. Calcium

a. aal-a 5 1 - 1 9 . A 22-year-old G3P2 with sickle cell anemia presents with 2 days o f worsening bone pain that is typical o f her past crises. A complete blood count reveals hemoglobin o f 6.5 g/dL. Transfusion with red cells at this point will achieve what goal? a . Decrease duration o f the acute pain crisis

b. Improve her pain symptoms C.

Increase blood count

b. a -l- a C.

5 1 -2 3 . Which o f the following findings on hemoglobin elec trophoresis would be most consistent with a diagnosis o f (3-thalassemia minor?

a. Hemoglobin A2 greater than 3.5%; normal fetal

d. All of the above 5 1 - 2 0 . Which o f the following contraceptive choices may help prevent painful crises in women with sickle cell disease?

a. Combination oral contraceptives b. Depot medroxyprogesterone acetate C. Intrauterine device

d. Bilateral tubal ligation

-I-a

d. - - / -

hemoglobin level

b. Hemoglobin A2 greater than 3.5%; fetal hemoglobin greater than 2 % C.

Hemoglobin A2 less than 1 %; normal fetal hemoglobin level

d. Hemoglobin A2 less than 1%; fetal hemoglobin greater than 2 %

Hematological Disorders 5 1 -2 4 . W hat is the initial treatment of women with immune thrombocytopenia in pregnancy when therapy is indi cated? a. Azathioprine

5 1 -2 5 . Pregnancy outcomes are generally good in women who have von Willebrand disease. Which of the following pregnancy-related complications maybe encountered in up to 50% o f such cases?

b. Intravenous immunoglobulin

a. Placental abruption

C. Prednisone

b. Fetal growth restriction

d. Splenectomy

C. Preterm birth

d. Postpartum hemorrhage

Medical and Surgical Complications

Chapter 51 Answer Key Q u e s tio n num ber

L e tte r answ er

P age c ite d

H e a d e r c ite d

Q u e s tio n num ber

L e tte r answ er

Page c ite d

H e a d e r c ite d

51-1

c

p. 1079

A n e m ia s

5 1 -1 4

c

p. 1085

A n e m ia s

5 1 -2

b

p. 1079

A n e m ia s

5 1 -1 5

a

p. 10 8 5

H e m o g lo b in o p a th ie s

5 1 -3

b

p. 1080

A n e m ia s

5 1 -1 6

d

p. 1 0 8 6

H e m o g lo b in o p a th ie s

5 1 -4

a

p. 10 8 0

A n e m ia s

5 1 -1 7

d

p. 10 8 6

H e m o g lo b in o p a th ie s

5 1 -5

c

p. 1081

A n e m ia s

5 1 -1 8

b

p. 1 0 8 7

H e m o g lo b in o p a th ie s

5 1 -6

d

p. 1081

A n e m ia s

5 1 -1 9

c

p. 10 8 7

H e m o g lo b in o p a th ie s

5 1 -7

b

p. 1082

A n e m ia s

5 1 -2 0

b

p. 1 0 8 9

H e m o g lo b in o p a th ie s

5 1 -8

a

p. 1082

A n e m ia s

5 1-21

a

p. 1 0 8 9

H e m o g lo b in o p a th ie s

5 1 -9

c

p. 1082

A n e m ia s

5 1 -2 2

d

p. 1091

H e m o g lo b in o p a th ie s

5 1 -1 0

d

p. 1083

A n e m ia s

5 1 -2 3

b

p. 10 9 2

H e m o g lo b in o p a th ie s

5 1 -1 1

c

p. 1084

A n e m ia s

5 1 -2 4

c

p. 1 0 9 4

P la te le t D isorders

5 1 -1 2

b

p. 1 0 8 4

A n e m ia s

5 1 -2 5

d

p. 10 9 8

5 1 -1 3

c

p. 1084

A n e m ia s

In h e rite d C o agulation D efects

5 2 -1 . The age-adjusted prevalence o f diabetes complicating pregnancy has increased from 14.5/1,000 in 1991 to what value in 2003?

a. 20/1,000

52~4. At and above what fasting plasma glucose level is overt diabetes diagnosed?

a. 105 mg/dL b. 116 mg/dL

b. 32.3/1,000

C. 126 mg/dL

c. 47.9/1,000

d. 140 m g/dL

d. 75.2/1,000 5 2 -2 . Imprinting of the fetus in the diabetic mother suggests that exposure to hyperglycemia leads to hyperinsulinemia, causing an increase in which fetal cell type and leading to obesity and insulin resistance in childhood?

a. Pancreatic |3-cells b. Adrenal zona fasciculata cells C.

Adipocytes

d. Pituitary somatotrophs 5 2 -3 . According to the image shown here, the sharpest increase in the age-specific incidence o f gestational diabetes occurred in what age group between 2 0 0 1 and 2004?

H f^dY m Vfg Reprinted from American Journal o f Obstetrics & Gynecology, Vol. 198, No. 5, D Getahun, C N ath, C V Ananth, et al: Gestational diabetes in the United States: Temporal trends 1989 through 2004, pp. 525.e l —525.e5, © 2008, with permission from Elsevier.

a . < 2 5 years

b. 25-34 years C.

> 3 5 years

d. None o f the above

5 2 -5 . When screening for gestational diabetes with a 50 g oral glucose challenge test, what minimum threshold value is used to identify women needing to undergo further testing?

a. 165 mg/dL b. 160 mg/dL c. 155 mg/dL d. 140 m g/dL

29 2

Medical and Surgical Complications 5 2 -6 . Results shown below from the Hyperglycemia and Adverse Pregnancy Outcome (HAPO) study suggest which o f the following?

5 2 - 9 . Which o f the following is true o f oral hypoglycemic agent use?

SECTION 8

a . An excellent substitute for insulin in all insulinresistant gestational diabetics

3. There is a threshold level o f fasting glucose above which fetal morbidity rates increases,

b. Inadequate in women with fasting glucose levels

b. Primary cesarean delivery rates are increased when

> 1 1 0 mg/dL

maternal fasting glucose is less than 90 mg/dL.

C. Often associated with the need for supplemental insulin

C. Increasing plasma glucose levels at each epoch are

associated with increasing adverse outcome rates.

d . Recommended by the American College o f Obste tricians and Gynecologists for gestational diabetes treatment

d. We should lower our threshold for diagnosis and treatment o f gestational diabetes.

30 -r

V g h^c \

25 c <

,+--

CD

+

+ ai X d g

4-

a j ag

• F a s tin g

720

2 0 (2 4

3 + (3 /

3 0 (3 4

4 + (4 /

4 0 (4 4

1 hour

0 ,+0

,+1(,.-

,..(,00

,01(,2,

,2-(,4.

,2/(-,,

9,++ 9-,-

2 hour

74+

4, (, + 3

,+4(,-0

, - 1(,. 4

,/ +(,0 2

,03(,2 2

9,23

Adapted from The H APO Study Cooperative Research Group: Hyperglycemia and adverse pregnancy outcomes. N Eng! J M ed 358(19):1991—2002, with permission. © 2008 Massachusetts M edical Society. All rights reserved.

5 2 -7 . What is the percentage o f A l gestational diabetics that

5 2 -1 0 . In women with A l gestational diabetes, which o f the

have shoulder dystocia?

following is true?

a. 0.1%

a . Elective cesarean delivery for sonographically esti mated fetal weights (EFW) > 4 ,5 0 0 g has lowered rates o f brachial plexus injury.

b. 0.3% C. 3%

d. 10% 5 2 -8 . Which o f the following is true regarding women using daily self glucose monitoring compared with those using intermittent semiweekly monitoring at clinic visits?

b. Elective induction with suspected macrosomia rather than awaiting spontaneous labor does not reduce shoulder dystocia rates. C. Sonographic suspicion o f shoulder dystocia is ac

ceptably accurate,

d. None o f the above.

a . Equal risk o f macrosomic infants

b. Gained more weight after diagnosis C. Gained less weight after diagnosis d . H ad a higher risk o f delivering a macrosomic infant

5 2 - 1 1 . A history o f gestational diabetes increases the risk of developing which o f the following? 3. Overt diabetes b. Hypertension C. Dyslipidemia d. All o f the above

Diabetes 5 2 -1 2 . W hat is the rate of stillbirth in pregestational diabetics? a. 0.4%

5 2 - 1 6 . This image concerning birthweight distribution illus trates which of the following?

b. 1.0% r

c. 1.5%

d. 2.0%

2++

(1++

5 2 -1 3 . In women with overt diabetes, one supported hypoth

(0++

esis for unexplained fetal demise suggests which o f the following?

/++

a . Accelerated placental senescence

.++

b. Umbilical cord edema that constricts cord blood flow

( -++

C. Maternal hyperglycemia that leads to osmotically induced villous edema

-

d . Umbilical vein thrombosis

-0

5 2 -1 4 . W hat are the two main causes o f fetal death in women

B ir th w e ig h t (s ta n d a rd d e v ia tio n s )

with type I diabetes?

a . Respiratory distress syndrome (RD S) and unex plained fetal demise b. Unexplained fetal demise and congenital malforma tions C. Unexplained fetal demise and uteroplacental insuf ficiency

d . Congenital malformations and placental abruption

From Bradley and co-workers, 1988; redrawn from British Medical Journal, 1988, Vol. 297, N o. 6663, pp. 1583—1584, with permission from the BM J Publishing Group.

a . Infants o f diabetic mothers are “growth promoted. ”

b. Growth promotion from maternal hyperglycemia does not preclude fetal growth restriction as defined by — 2 standard deviations below the mean. C.

5 2 -1 5 . Proven causes for hydramnios in diabetic pregnancies include which of the following?

a . Increased amnionic fluid glucose concentration b. Fetal polyuria

,++

The birthweight distribution o f infants of diabetic mothers is skewed toward consistently heavier birthweights.

d. All o f the above. 5 2 -1 7 . Which of the following neonatal morbidities are

C. Fetal esophageal atresia

related to maternal hyperglycemia?

d. None ofthe above

a . Respiratory distress syndrome

b. Hypoglycemia C.

Hyperbilirubinemia and polycythemia

d. All o f the above 5 2 -1 8 . Both mother and father o f an infant have type I dia betes. W hat is the infant’s chance of developing type I diabetes? a. 1-3%

b. 6 % C.

20%

d.

None o f the above

52-1 9. Chronic hypertension with diabetic nephropathy is associated with what percent risk o f developing preeclampsia? a. 2 0% b. 40% C. 60% d. S 0 -:

293

294

Medical and Surgical Complications 5 2 - 2 0 . In this image, proliferative retinopathy and the results o f laser photocoagulation are shown. This procedure reduces the rate o f vision loss progression and blindness by what percentage?

5 2 - 2 3 . What percentage o f women with diabetes have infec tions during pregnancy?

a. 25% b. 40% c. 60%

d. 80% 5 2 - 2 4 . The American Diabetes Association (ADA) considers optimum preconceptional glucose levels to include which o f the following self-monitored values? a. Preprandial glucose levels 70—100 mg/dL

b . 2-hour postprandial levels < 120 mg/dL C. 1-hour postprandial levels < 1 4 0 mg/dL

d. All o f the above

Reproduced, with permission, from Elm an K D , Welch RA, Frank R N , et al: Diabetic retinopathy in pregnancy: A review. Obstet Gynecol 75:119, 1990.

5 2 -2 5 . Self monitoring o f capillary glucose levels using a glucometer is recommended for which o f the following reasons?

a. 20%

a . Insulin pumps increase the risk o f hyperglycemia.

b. 50%

b. It involves a woman in her own care.

c. 75%

C. It is the only way to ensure oral hypoglycemic efficacy.

d . Is curative 5 2 - 2 1 . According to this image, which o f the following is true?

d . It decreases the number o f necessary office visits. 5 2 - 2 6 . The A D A recommends what daily caloric intake for pregnant women with diabetes?

a . 40 kcal/kg/d for normal-weight women b. 30-35 kcal/kg/d for overweight women C. 30-35 kcal/kg/d for normal-weight women

d . 24 kcal/kg/d for underweight women 5 2 - 2 7 . Good pregnancy outcomes can be achieved with mean plasma glucose levels as high as what value? = ^VW h^X XaVgg

;

c8,1/

<

c8,-4

=

c8,2-

c8-1

LdhVa

c8/4,

Reproduced, with permission, from Cunningham F G , Leveno K J, Bloom SL, et al: W illiams Obstetrics, 23rd ed. N ew York, McGraw-Hill, 2010.

a . Empiric risks o f preeclampsia are related to the White classification.

b. Patients with Class F diabetes in Sweden have sim ilar risks for preeclampsia as those in the United States. C. The lowest risk o f preeclampsia is in patients with Class C diabetes.

d. All o f the above. 5 2 - 2 2 . What is the incidence o f fetal loss with ketoacidosis during pregnancy?

a. 5% b. 15% c. 20% d. 30%

a. 110 mg/dL b. 121 mg/dL C. 133 mg/dL

d. 143 mg/dL 5 2 - 2 8 . Which o f the following is true o f managing diabetic patients during the second trimester? a. Maternal serum alpha-fetoprotein (msAFP) should be obtained at 16—20 weeks.

b. Targeted sonography should be performed between 18 and 2 0 weeks. C. A stable period o f glycemic control ensues lasting until approximately 24 weeks,

d. All o f the above.

Diabetes

295

Chapter 52 Answer Key L e tte r answ er

Page c ite d

H e a d e r c ite d

Q u e s tio n num ber

L e tte r answ er

Page c ite d

H e a d e r c ite d

52-1

c

p. 1 1 0 4

In tro d u c tio n

5 2 -1 6

d

p. 1115

Figure 5 2 -5

5 2 -2

c

p. 1 1 0 4

In tro d u c tio n

5 2 -1 7

b

p. 1115

5 2 -3

a

p. 1105

Figure 52-1

N e o n a ta l M o rta lity and M o rb id ity

5 2 -4

c

p. 1105

O ve rt D iabetes

5 2 -1 8

c

p. 1116

In h e rita n ce o f D iabetes

5 2 -5

d

p. 1106

S creening

5 2 -1 9

c

p. 1116

D ia b e tic N e p h ro p a th y

5 2 -6

c

p. 11 0 4

Figure 52-2 and HAPO s tu d y

5 2 -2 0

b

p. 11 1 7

Figure 5 2 -4 and D iabetic R e tin o p a th y

5 2 -7

c

p. 1109

M a c ro so m ia

5 2-21

a

p. 1113

Figure 5 2 -4

5 2 -8

c

p. 11 1 0

Glucose M o n ito rin g

5 2 -2 2

c

p. 1118

D iabetic K etoacidosis

5 2 -9

b

p. 1111

Oral H yp o g ly c e m ic A g e n ts

5 2 -2 3

d

p. 1118

In fe ctio n s

5 2 -1 0

b

p. 1111

O b ste trica l M a n a g e m e n t

5 2 -2 4

d

p. 1119

P re co n ce p tio n a l Care

5 2 -1 1

d

p. 1112

P o s tp a rtu m E valuation

5 2 -2 5

b

p. 1119

Insulin T re a tm e n t

5 2 -1 2

b

p. 1113

Table 52-7

5 2 -2 6

c

p. 1120

D iet

5 2 -1 3

c

p. 11 1 4

U n e xp la in e d Fetal D e m ise

5 2 -2 7

d

p. 1120

H yp o g ly c e m ia

5 2 -1 4

b

p. 1113

Fetal Effects

5 2 -2 8

d

p. 1120

Second T rim e s te r

5 2 -1 5

c

p. 1115

H yd ra m n io s

CHAPTER 5

Q u e s tio n num ber

296

Thyroid and Endocrine Disorders 5 3 -1 . Which o f the following is true o f thyroid-stimulating hormone (TSH ) during pregnancy?

a . Decreased levels early b. Crosses the placenta and stimulates fetal thyroxine

5 3 - 5 . In this graph, what is the incidence o f antithyroid per oxidase antibodies in women with subclinical hypothy roidism? (SH = subclinical hypothyroidism; IM H = isolated maternal hypothyroxinemia)

production 1+(

d. None o f the above 5 3 -2 . Thyroid peroxidase antibodies are identified in approx imately what percentage o f pregnant women?

a. 2% b. 10% C.

20%

d. 30%

0+

' §! 5 m /+ ( c ° ® 0 Sr c tn I 03

c

5 fe na

Q.

g 'o >.

20-

IQ -

c. 0.5% d. 0.25% 5 3 -4 . Complications from thionamides include which o f the following?

a. Agranulocytosis b. Hepatotoxicity C.

Serious vasculitis

d. All ofthe above

/!

0!

c 8 2,0

c 8 ,,

- Q ___CL

E u th y ro id IM H (n = 1 6 ,4 0 7 ) (n = 2 3 3 )

curs in what percentage o f pregnancies?

b. 0.1%

.Vi.

., ! c 8 ,33

.+(

5 3 -3 . Symptomatic thyrotoxicosis or hyperthyroidism oc a. 1%

1, ! c 8 3-

701

C. Increased levels early because o f the effects o f human chorionic gonadotropin (hCG)

iiii

m

SH O v e rt (n = 5 9 8 ) h y p o th y ro id (n = 1 3 4 )

Reproduced, with permission, from Cunningham FG , Leveno KJ, Bloom SL, et al: Williams Obstetrics, 23rd ed. New York, M cGraw-Hill, 2010.

a. 4% b. 5% c. 31% d. 61% 5 3 - 6 . What is the reported perinatal mortality rate associated with uncontrolled hyperthyroidism in pregnancy?

a. 5% b. 10% c. 15% d. 20%

Thyroid and Endocrine Disorders 5 3 -7 . This sonogram image illustrates what morbidity con sistent with fetal thyrotoxicosis?

5 3 - 1 0 . Which o f the following is true o f the condition illus trated by this newborn, who was delivered by a mother with thyroid disease?

Reproduced, with permission, from Cunningham FG, Leveno KJ, Bloom SL, et al: Williams Obstetrics, 23rd ed. New York, McGraw-Hill, 2010.

a. Hepatomegaly b. Nonimmune hydrops C.

Fetal pleural effusion

d. All of the above

Reproduced, with permission, from Cunningham FG , Leveno KJ, Bloom SL, etal: Williams Obstetrics, 23rd ed. New York, McGraw-Hill, 2010.

a. Goitrous hypothyroidism b. A common adverse fetal effect of maternal propy lthiouracil (PTU) treatment

5 3 -8 . When treating maternal thyroid storm with heart failure, which o f the following is true? a . Initially, sodium iodide is administered and then continued every 8 hours.

b. If there is a history o f iodine-induced anaphylaxis, Lugol solution is given every 6 hours. C. Initially, 1 g o f a thionamide is given by nasogastric tube. d. Dexamethasone is contraindicated. 5 3 -9 . Concerning gestational thyrotoxicosis, which o f the following is true?

a. It should be treated with low-dose thionamides. b. There is abnormal massive thyrotropin release. C.

It should not be treated with antithyroid drugs,

d . None o f the above.

C.

Will not occur if the mother has had thyroid gland radioablation

d. A fetal arteriovenous malformation 5 3 - 1 1 . Subclinical hypothyroidism is characterized by which of the following?

a. High serum thyroxine level b. Low free thyroxine (FT 4 ) level C.

High serum thyrotropin and normal F T 4 levels

d. Low serum thyrotropin level 5 3 - 1 2 . Which o f the following is the most common cause of hypothyroidism in pregnancy? a. Graves disease

b. Paraneoplastic syndrome C.

Hashimoto thyroiditis

d. None o f the above

297

298

Medical and Surgical Complications 5 3 - 1 3 . Concerning treatment ofhypothyroidism, which of the following is true?

5 3 - 1 8 . Which o f the following is true o f 1,25-dihydroxyvitamin D?

SECTION 8

9. Approximately 100 |i,g thyroxine should be given daily.

a . Levels increase threefold during pregnancy.

b. Thyrotropin levels should be measured at 4- to

C. It is mostly o f maternal origin during pregnancy,

6 -week intervals. C.

The thyroxine dose should be adjusted in 25 -5 0 |!g increments to achieve T S H levels between 0.5 and 2.5 mU/L.

d. All o f the above. 5 3 - 1 4 . Which o f the following is true o f routine thyrotropin screening in pregnancy? a . It is advocated by the American College o f Obstetri cians and Gynecologists.

b. It leads to improved outcome o f neonates whose mothers have subclinical hypothyroidism. C. It is not advocated by the American Association o f Clinical Endocrinologists,

d. It should be performed on symptomatic women or those with a history o f thyroid disease. 5 3 - 1 5 . Which o f the following is true o f isolated maternal hypothyroxinemia? a. It is characterized by high T S H an dlow F T 4 levels.

b. It has a similar prevalence of anti thyroid antibodies as subclinical hypothyroidism. C.

In some women, it is associated with a twofold inci dence of macrosomia.

d . It has no apparent serious adverse effects on preg nancy outcome. 5 3 - 1 6 . Which of the following is true of postpartum thyroidi tis? a. It affects 5-10% o f women during the first year postpartum.

b. It is related to increasing levels o f thyroid autoanti bodies. C.

It occurs in 25% o f women with type I diabetes mellitus.

d. All o f the above. 5 3 - 1 7 . Which o f the following is true o f thyroid nodules dur ing pregnancy? a. When smaller than 0.5 cm, they can be detected by sonography.

b. They are not assessed properly with fine-needle aspiration. C.

If cancerous, they can confer a worse prognosis than if found in nonpregnant controls,

d. They can be safely removed before 24 -26 weeks.

b. It increases gastrointestinal calcium absorption. d. All o f the above. 5 3 - 1 9 . Which of the following is true o f hyperparathyroidism? a . It may be masked by pregnancy due to significant calcium shunting to the fetus.

b. It is caused mainly by hyperfunction of all four parathyroid glands. C. It has a reported prevalence o f 2 -3 per 1 0 , 0 0 0 women,

d. It is generally a disease o f young females. 5 3 - 2 0 . Ofwom en with hyperparathyroidism in pregnancy, which o f the following is true? a . If symptomatic, surgical removal o f the parathyroid adenoma should be delayed until postpartum.

b. Asymptomatic women may be treated with oral phosphate, 1 - 1 . 5 g daily in divided doses. C. If hypercalcemic crisis occurs, the patient should have fluids restricted,

d. Adjunctive therapy includes doxorubicin, which inhibits bone resorption. 5 3 - 2 1 . Which o f the following is true o f pheochromocytomas? a. They are found in 0.1% o f hypertensive patients during pregnancy.

b. They are called the 10-percent tumor. C. They are detected by a 24-hour urine collection for free catecholamines, metanephrines, or vanillylmandelic acid (VMA).

d. All o f the above.

Thyroid and Endocrine Disorders 5 3-2 2. The patient whose magnetic resonance imaging (MRI) is shown here has a history o f hypertension, palpita tions, and frequent flushing episodes. The most likely diagnosis is which o f the following?

5 3 - 2 5 . Concerning Addison disease, which ofthe following is true? a. The most frequent cause is tuberculosis.

b. Adrenal hypofunction does not usually affect fertil ity. C. Low cortisol levels should prompt adrenocorti cotropic hormone stimulation testing. d. Cortisone therapy may be discontinued postpartum. 5 3 - 2 6 . Concerning prolactinomas, which ofthe following is true? a . They are classified by location.

b. They are considered macroadenomas if larger than 5 mm. C.

They are treated with bromocriptine if smaller than 1 0 mm.

d. They should be surgically resected prior to preg nancy if suprasellar and > 1 0 mm. 5 3 - 2 7 . This MRI illustrates which ofthe following?

Reproduced, with permission, from Cunningham FG , Leveno K J, Bloom SL, et al: Williams Obstetrics, 23rd ed. N ew York, McGraw-Hill, 2010.

a. W ilms tumor b. Adrenal tumor C.

Renal medulloblastoma

d. Liver hepatoma 5 3 - 2 3 . Which o f the following is appropriate first-line medical therapy for hypertension with pheochromocytoma in pregnancy?

a. Calcium-channel blockers b. (3-Blockers C.

a-Blockers

d. Apresoline 5 3 - 2 4 . Concerning Cushing syndrome, which o f the follow ing is true? a . M ost common cause is adrenal adenoma.

b. Cases o f corticotropin-dependent Cushing syndrome in pregnancy occur frequently. C. Typical body habitus show moon facies, buffalo hump, and peripheral obesity,

d. Associated heart failure is rare during pregnancy.

Reproduced, with permission, from Cunningham FG , Leveno K J, Bloom SL, etal: Williams Obstetrics, 23rd ed. New York, McGraw-Hill, 2010.

a. Suprasellar tumor b. Sequelae o f an acute hemorrhagic obstetric event C. A condition that may lead to chronic hypertension d. A condition that responds to bromocriptine

299

Medical and Surgical Complications

Chapter 53 Answer Key Q u e s tio n num ber

L e tte r answ er

Page c ite d

H e a d e r c ite d

Q u e s tio n num ber

L e tte r answ er

Page c ite d

H e a d e r c ite d

53-1

a

p. 1 1 2 7

T h yro id P h y s io lo g y and P re g n a n cy

5 3 -1 4

d

p. 1133

T h y ro tro p in S creening in P re gnan cy

5 3 -2

b

p. 1 1 2 7

A u to im m u n ity and Thyroid Disease

5 3 -1 5

d

p. 1133

Isolate M a te rn a l H y p o th y ro x in e m ia

5 3 -3

b

p. 11 2 7

H y p e rth y ro id is m

5 3 -1 6

d

p. 11 3 4

P o s tp a rtu m T h y ro id itis

5 3 -4

d

p. 1128

T re a tm e n t

5 3 -1 7

d

p. 11 3 5

N o d u la r Thyroid Disease

5 3 -5

c

p. 11 2 7

Figure 53-2

5 3 -1 8

b

p. 1 1 3 5

P ara th yro id Diseases

5 3 -6

b

p. 1129

P re g n a n cy O u tco m e

5 3 -1 9

a

p. 1135

5 3 -7

b

p. 11 2 9

Fetal and N e o n a ta l Effects, Figure 2 9 -7

H y p e rp a ra th y ro id is m (in P regnancy)

5 3 -2 0

b

p. 11 3 5

M a n a g e m e n t in P re g n a n cy

5 3-2 1

d

p. 11 3 6

P h e o c h ro m o c y to m a

5 3 -2 2

b

p. 1 1 3 7

Figure 5 3 -4

5 3 -2 3

c

p. 1 1 3 7

M anagem ent

5 3 -2 4

d

p. 1 1 3 7

C ushing S yn d ro m e

5 3 -2 5

c

p. 11 3 9

A d re n a l D e ficie n cy

5 3 -2 6

d

p. 11 3 9

P ro la ctino m a s

5 3 -2 7

b

p. 11 4 0

Figure 53-5

5 3 -8

c

p. 11 3 0

T h yro id S torm and H eart Failure: M a n a g e m e n t

5 3 -9

c

p. 1 1 3 0

H yp e re m e s is G ra vida ru m and G e s ta tio n a l Th yro to xicosis

5 3 -1 0

a

p. 11 2 9

Figure 53-3

5 3-1 1

c

p. 11 3 0

S ubclinical H y p e rth y ro id is m

5 3 -1 2

c

p. 1131

O ve rt H y p o th y ro id is m and P re g n a n cy

5 3 -1 3

d

p. 1131

T re a tm e n t

Connective Tissue Disorders 5 4 - 5 . Which statement is true regarding immune-mediated diseases in general during pregnancy?

5 4 -1 . All EX C E P T which o f the following are inherited con nective tissue disorders?

a. Chondrodysplasia

a . They worsen during pregnancy.

b. Marfan syndrome

b. They improve during pregnancy.

C.

Rheumatoid arthritis

C.

They remain unchanged during pregnancy,

d . They are modulated by the effect o f pregnancy

d. Osteogenesis imperfecta

hormones.

5 4 -2 . Which statement is true regarding immune-mediated connective tissue disorders?

5 4 -6 . Fetal cells and D N A are present in maternal blood by what time?

a . They rarely show renal involvement. b. They have a clearly elucidated pathogenesis.

a. First trimester

C. They are always associated with rheumatoid factor,

b. Second trimester

d . They may or may not have an association with au

C. Third trimester

toantibody formation.

d. Postpartum period

5 4 -3 . Which statement is true regarding the innate phase of the immune system’s protection against nonself?

a . It is mediated through cytotoxic mechanisms of cell injury.

5 4 -7 . The heritable “autoimmunity gene,” which predisposes to lupus, rheumatoid arthritis, and Crohn disease, is located on which chromosome?

a. 6

b. It involves direct antibody attachment to a specific surface antigen. C.

b. 16

c. 21

It is broad and rapid and mediated through neu trophils, macrophages, and complement,

d. 22

d . It is precise and is mediated by antigen-specific

5 4 -8 . High titers o f which systemic lupus erythematosus (SLE)-specific antibody correlate with nephritis and vasculitis activity?

reactions through T and B lymphocytes. 5 4 -4 . The series o f genes shown here is located on which chromosome?

a. Anti-Ro

a . The long arm o f chromosome 6

b. Anti-La

b. The short arm of chromosome 6

C.

C.

The long arm o f chromosome 16

Antinuclear antibody

d. Anti-double-stranded D N A

d. Theshort arm of chromosome 16

View more...

Comments

Copyright ©2017 KUPDF Inc.
SUPPORT KUPDF